Download as pdf or txt
Download as pdf or txt
You are on page 1of 184

LỜI GIẢI ĐỀ THI CHỌN ĐỘI TUYỂN QUỐC GIA

DỰ THI IMO 2011


***********
Bài 1.

Trên mặt phẳng tọa độ có một con cào cào ở điểm (1;1) . Nó có thể nhảy từ điểm A sang điểm
1
B khi tam giác OAB có diện tích bằng và tọa độ của A, B nguyên dương.
2
1. Tìm các điểm (m, n) sao cho con cào cào có thể nhảy đến đó sau hữu hạn bước.
2. Chứng minh rằng con cào cào có thể nhảy đến (m, n) kể trên sau ít hơn m  n bước.

Lời giải.

Trước hết, ta cần chứng minh nhận xét sau:

Với (m, n) là cặp số nguyên tố cùng nhau thì tồn tại a và b nguyên tố cùng nhau sao cho

mb  na  1 và a  b  m  n 1 .
Thật vậy, không mất tính tổng quát, giả sử m  n .

Xét các số có dạng mb 1,1  b  n . Dễ thấy ta có n số như vậy và nếu b  b ' thì
mb 1  mb '1(mod n) nên các số này lập thành hệ thặng dư đầy đủ mod n, suy ra tồn tại số b
sao cho mb 1 n . Ta cũng thấy rằng b  n 1 vì b  n không thỏa mãn.
1
mb 1 mn
Đặt a  a  m và mb  an  1 .
n n

Ta có n(b  a)  (m  n)b 1  (m  n)n  (m  n)(b  n) 1  (m  n)n  b  a  m  n .

Do đó b  a  m  n 1 . Nhận xét được chứng minh.

Trở lại bài toán, ta thấy rằng, con cào cào đang ở đỉnh A(a, b) và muốn nhảy sang đỉnh B(c, d )
1 1 1
thì phải có SOAB   ad  bc   ad  bc  1 .
2 2 2

Từ đó suy ra xuất phát từ điểm (1;1) , con cào cào chỉ có thể nhảy đến các điểm (m; n) mà
(m, n)  1 . Hơn nữa, với (m, n)  1 thì theo nhận xét ở trên thì con cào cào có thể nhảy đến điểm
có tọa độ tương ứng là (m; n) . Vậy các điểm cần tìm là (m; n) với m, n nguyên tố cùng nhau.

2. Xét điểm nguyên dương (m; n) mà (m, n)  1, m  n  1 thì con cào cào có thể nhảy đến điểm
này chỉ sau một bước nhảy.

Ta lại xét điểm (m; n) mà (m, n)  1, m  n  1 thì theo nhận xét trên, tồn tại các số m ', n ' sao
cho mm ' nn '  1 và m ' n '  m  n 1 .

Lặp lại quá trình này, ta thấy rằng con cào cào có thể nhảy đến điểm (m; n) sau không quá
m  n bước.

Vậy ta có đpcm.

2
Bài 2.

Cho đường (O) và điểm A nằm ngoài đường tròn. Kẻ hai tiếp tuyến AB, AC tới (O) với A, B là
các tiếp điểm. Gọi P là điểm thuộc tia đối của tia BA, Q là điểm thuộc tia đối của tia CA sao
cho PQ tiếp xúc với (O). Qua P kẻ đường thẳng song song với AC và cắt đường thẳng BC tại
E. Qua Q kẻ đường thẳng song song với AB và cắt BC tại F.
1. Chứng minh rằng các đường thẳng QE , PF luôn đi qua điểm cố định lần lượt là M, N.
2. Chứng minh rằng tích PM  QN không đổi.
Lời giải.

Trước hết, ta chứng minh nhận xét sau: Cho tam giác ABC ngoại tiếp (I) có tiếp điểm của (I) lên
AB, AC lần lượt là E, F. Đường thẳng qua B, song song với AC cắt EF tại K; CK cắt AB tại G.
Khi đó, tam giác AGI vuông tại I.
Thật vậy:

Đặt AB  c, BC  a, CA  b và chu vi tam giác ABC là 2 p .


Do BK // AC nên tam giác BKF cân tại B, suy ra: AE  AF  p  a .

BG BK p b AB p bc
Theo định lí Thales thì:      AG  .
AG AC b AG b p
AF p( p  a)
Mà AF  p  a nên  .
AG bc
AF A AH A 1 cos A p ( p  a )
Ta cũng có: AI  , AH  AF .sin   sin 2   ..
A 2 AI 2 2 bc
sin
2
AF AH p( p  a )
Từ đó suy ra:   . Do đó: GI song song với EF, tức là AGI vuông tại I.
AG AI bc

3
Bổ đề được chứng minh. Trở lại bài toán đã cho.
1/ Gọi M, N lần lượt là giao điểm
của QE với AB và PF với AC.
Theo bổ đề trên, ta thấy rằng tam
giác OMA và ONA lần lượt
vuông tại O nên các điểm M, N
cố định.
2/ Trước hết, ta đặt
AB  AC  a, BP  x, CQ  y .
Chu vi của tam giác APQ là
p  2(a  x  y ) .
Theo bổ đề trên, ta tính được:

2 AP.PQ
PM  AP  
AP  AQ  PQ
AP ( AP  AQ  PQ ) (a  x ) x

AP  AQ  PQ a x y
(a  y ) y
Tương tự QN  .
ax y
xy (a  x )(a  y )
Cần chứng minh rằng tích PM  QN  không đổi. Thật vậy:
(a  x  y ) 2
Nếu gọi R là bán kính của (O) thì diện tích của tam giác APQ cùng bằng:

pR sin BAC. AP. AQ (a  x )(a  y ) R
   .
2 2 a x y 
sin BAC

(a  x )(a  y ) R
Suy ra tỉ số  k không đổi, với k  . Từ đó, ta có
a x y 
sin BAC

xy
(a  x)(a  y )  k (a  x  y )  a (a  x  y )  xy  k (a  x  y )  a   k , tức là tỉ
a x y
xy
số cũng không đổi.
a x y

xy (a  x )(a  y )
Vậy PM  QN  không đổi. Ta có đpcm.
(a  x  y ) 2

4
* Cách khác:

1. Ta sẽ chứng minh giao điểm


A
M của EQ và AB, giao điểm N
của FP và AC là các điểm cố
định.

Giả sử (O) tiếp xúc với BC tại


D. Gọi K, L lần lượt là giao
L điểm của QO và BC, PK và
OD theo thứ tự đó.

C F   90 0 ,suy
Dễ thấy rằng PKO
K ra: các cặp tam giác LKO, LDP
B N
O và LOP, QOA đồng dạng với
E M nhau. Suy ra,

OK LO OQ OK PB
   
PD LP AQ OQ AQ
Mặt khác dễ thấy là PE  PB
P D Q OK PE EM
nên   .
OQ AQ MQ

Áp dụng định lí thales đảo ta được: MO song song với BC là đoạn cố định mà M thuộc AB cố
định nên M cũng cố định. Tương tự, N là điểm cố định. Ta có đpcm.

2.Từ câu 1), ta có M , O, N thẳng hàng. Từ đó dễ có PMO  ONQ ( g .g ) .

Suy ra PM  QN  OM  ON  OM 2 là hằng số.

Ta cũng chứng minh được rằng M, N là tiếp điểm của đường tròn A-Mixitilinear trên các đoạn
AP, AQ của tam giác APQ.

5
Bài 3.

Cho cho n nguyên dương thỏa n  3 và n số thực x1 , x2 , x3 ,..., xn thỏa mãn đồng thời
(i) x1  x2  x3  ...  xn  0 .
(ii) x12  x22  x23  ...  xn2  n(n 1) .
(iii) x1  x2  x3  ...  xn .
Tìm giá trị lớn nhất và nhỏ nhất của f  x1  x2 .

Lời giải.

*Tìm giá trị lớn nhất.

Theo BĐT Bunhiacopski cho n  2 số thực x3 , x4 ,..., xn , ta có:

( x3  x4  ...  xn )2
(12 12  ...12 )( x32  ...  xn2 )  ( x3  x4  ...  xn ) 2  x32  ...  xn2  . Suy ra
n2

( x3  x4  ...  xn )2 ( x1  x2 ) 2 ( x1  x2 ) 2
n(n 1)  x  x  x  ...  x  x  x 
2 2 3 2 2 2
 
n2 n2
1 2 2 n 1 2
2
2n(n 1)(n  2)
 ( x1  x2 )2   2(n 1)(n  2)  x1  x2  2(n 1)(n  2)
(n  2  2)

Do đó, giá trị lớn nhất của f  x1  x2 là 2(n 1)(n  2) , đạt được khi

(n 1)(n  2) 2(n 1)


x1  x2 , x3  x4  ...  xn hay x1  x2  , x3  x4  x5  ...  xn   .
2 n2

*Tìm giá trị nhỏ nhất.


Ta xét các trường hợp:

- Nếu n  3 thì x1  2 x2  x1  x2  x3  0 và 2 x1  x2  x1  x2  x3  0 .

Thay x3  ( x1  x2 ) vào đẳng thức (ii), ta được x12  x22  ( x1  x2 )2  6  x12  x22  x1 x 2  3 .
Từ đánh giá (2 x1  x2 )( x1  2 x2 )  0  x12  x22  x1 x 2  (2 x1  x2 )( x1  2 x2 )  3  ( x1  x2 ) 2  1 .

Dễ thấy nếu x1  x2  1  x3  1  0  x1  x2 , mâu thuẫn nên x1  x2  1 .

Đẳng thức xảy ra khi x1  2, x2  x3  1

-Nếu n  4 , ta chú ý đến đánh giá sau: “Nếu a  b  c thì b 2  c 2  a 2  (b  c  a) 2 . (*) ”


6
Đánh giá này đúng vì

b 2  c 2  a 2  (b  c  a )2  2a 2  2a(b  c )  2bc  0  (a  b)(a  c)  0

Áp dụng đánh giá này với x2  x3  x4 , ta được: x32  x42  x22  ( x3  x4  x2 )2 .

Dễ thấy x2  x3  x4  x2 , x2  x5 nên áp dụng tiếp đánh giá (*), ta có:

x32  x42  x52  x22  ( x3  x4  x2 )2  x52  2 x22  ( x3  x4  2 x2 ) 2 .

Tiếp tục áp dụng đánh giá (*) với các số a6 , a7 ,..., an , ta được:

x12  x22  x23  ...  xn2  (n  3) x22   x3  x4  ...  xn  (n  3) x2   (n  3) x22   x1  (n  2) x2 


2 2

Suy ra x12  (n  2) x22   x1  (n  2) x2   n(n 1) .


2

Ta sẽ chứng minh rằng


n(n 1)( x1  x2 )2
x12  (n  2) x22   x1  (n  2) x2   2 x12  (n 2  3n  2) x22  2(n  2) x1 x2 
2
.
4

Bất đẳng thức này tương đương với: ( x1  x2 ) (n 2  n  8) x1  (n 1)(3n  8) x2   0 , mà bất đẳng
thức này đúng do x1  x2 và

(n 2  n  8) x1  (n 1)(3n  8) x2  (n 2  n  8) x1  (3n  8) x1  n(n  4) x1  0 .

n(n 1)( x1  x2 )2
Từ các điều này, ta có: n(n 1)   ( x1  x2 )2  4 , dễ thấy x1  x2  2 .
4

Trong trường hợp này, giá trị nhỏ nhất của biểu thức đã cho là 2, đạt được khi
x1  x2  x3  ...  xn1  1, xn  1 n .

Vậy giá trị lớn nhất của f  x1  x2 là 2(n 1)(n  2) , đạt được khi

(n 1)(n  2) 2(n 1)


x1  x2 , x3  x4  ...  xn hay x1  x2  , x3  x4  x5  ...  xn   .
2 n2

Giá trị nhỏ nhất của f  x1  x2 là 1 với n  3 , đạt được khi x1  2, x2  x3  1 và là 2 với
(n 1)(n  2) 2(n 1)
n  4 , đạt được khi x1  x2  , x3  x4  x5  ...  xn   .
2 n2

7
Bài 4.

 a2 
Cho dãy số (an ) thỏa mãn a0  1, a1  3 và an 2  1   n1  .
 a 
 n 
Chứng minh rằng với mọi số tự nhiên n thì an an 2  an21  2n .
Lời giải.

Dễ thấy mọi số hạng của dãy đều là số nguyên dương.

Trước hết, bằng quy nạp, ta sẽ chứng minh rằng đã cho cũng thỏa mãn hệ thức truy hồi
an 2  4an 1  2 an , n   .

Thật vậy, bằng tính toán trực tiếp, ta có a2  10, a3  34, a4  116 thỏa mãn cả hai hệ thức truy
hồi, tức là khẳng định đúng với n  0, 1, 2 .

Giả sử khẳng định đúng đến n  k , k  2 , tức là ak  2  4 ak 1  2ak .

Ta sẽ chứng minh rằng nó cũng đúng với n  k 1 .

 a2   ( 4 a  2 ak ) 2   4 ak2 
Ta có ak 3  1   k  2   1   k 1   1  16 a k 1  16 a k   .
 ak 1   ak  1   ak 1 

 4a 2 
Ta lại cần chứng minh  k   4 ak 1  1, k  2  * .
 ak 1 

Thật vậy,

4ak2 4ak2 1
(*)   1  4 ak 1  1   ak2  ak 1ak 1  ak2  ak 1
ak 1 ak 1 4

 a2  a2 a2
Hơn nữa, theo giả thiết thì ak 1  1   k   k  ak 1  1  k  ak2  ak 1ak 1  ak 1  ak2 .
 ak 1  ak 1 ak 1

Theo giả thiết quy nạp, ta có

1
ak 1  4ak  2 ak 1  4(4ak 1  2 ak 2 )  2 ak 1  6 ak 1  8(ak 1  ak 2 )  4ak 1  ak 1  ak 1 .
4

2 1 2  4ak2 
Do đó, ta có được a  ak 1ak 1  a  ak 1 hay (*) đúng. Từ
k k    4 ak 1  1, k  2 , ta có
4  ak 1 

8
ak 3  1  16ak 1  16 ak  4 ak 1  1  4(4 ak 1  2ak )  2(4ak  2 ak 1 )  4 ak 2  2ak 1 .

Suy ra khẳng định đúng với n  k 1 và theo giả thiết quy nạp thì khẳng định được chứng minh.

Ta sẽ chứng minh dãy số thỏa mãn a0  1, a1  3, an  4an 1  2an  2 , n  2 cũng thỏa mãn đẳng
thức an 2 an  an21  2 n với mọi n (**)

Thật vậy, ta có an 2 an  an21  (4an 1  2 an )an  an 1 (4an  2an 1 )  2(an 1an 1  an2 ) .

Lặp lại biến đổi này n lần với chú ý a0 a2  a12  1  10  32  1  2 0 ta thấy (**) đúng.

Vậy ta có đpcm.

9
Bài 5.

Tìm tất cả các số nguyên dương n sao cho biểu thức sau

A  2n 2 (2n 1)  8.3n  1

là một số chính phương.


Lời giải.
Giả sử tồn tại số n nguyên dương thỏa mãn đề bài.

Khi đó, tồn tại m sao cho:

m2  2 n2 (2 n 1)  8  3n 1  2 2 n 2  2  2n1 1 8  3n  (2n 1 1) 2  8.3n

(2n 1  1  m)(2n 1  1  m)
hay  2  3n .
4

(2n 1  1)  t (2n 1  1)  t
Đặt a  ,b  thì a  b  2 n 1  1, ab  2  3n .
2 2

Dễ thấy t là số lẻ nên a, b là các số nguyên dương. Do đó, a  3u , b  2  3v hoặc ngược lại.

Trong cả hai trường hợp, ta đều có 3u  2  3v  2n1  1 và u  v  n .

Ta xét các trường hợp sau:

- Nếu n  1 thì 3u  2  3v  3, u  v  1, hệ này không có nghiệm nguyên dương.

- Nếu n  2 thì 3u  2  3v  7, u  v  2 , hệ này cũng không có nghiệm nguyên dương.

- Nếu n  3 thì 3u  2  3v  15, u  v  3 , hệ này có nghiệm là u  2, v  1 .

- Nếu n  4 thì 3u  2  3v  31, u  v  4 , hệ này không có nghiệm nguyên dương dương.

- Nếu n  5 thì 3u  2  3v  63, u  v  5 , hệ này có nghiệm là u  2, v  3 .

- Nếu n  5 , ta sẽ chứng minh rằng hệ tương ứng trong trường hợp này sẽ vô nghiệm.
2 ( n 1)
2(n  1) 2(n  1) n2
Ta có 3u  2n 1  9u  8 3
u  nv  v 1.
3 3 3

n2 n2
Tương tự, ta cũng có u   1 . Đặt w  min{u, v}   1  w  2 . Suy ra
3 3
2 n1  1  0(mod 9) . Bằng cách thử trực tiếp, ta có được n  1  0(mod 6) hay n  1  6 k , k  * .

10
Từ đó, ta được 3u  2  3v  (2 k  1)(2 k  1)(42 k  4 k  1) .

Dễ dàng thấy rằng 42 k  4 k  1  1  1  1  0(mod 3) nhưng 42 k  4 k  1  2 k  4 k  1  0(mod 9)


n2 n 1
nên 3w1 là ước của (2 k  1)(2 k  1) . Suy ra 3w1  2 k  1  3k  1  w 1   n  11
3 6

Dễ thấy không tồn tại số nguyên dương n nào mà n  1  0(mod 6) và 5  n  11 nên trong
trường hợp này, hệ không có nghiệm.

Vậy có tất cả hai số nguyên dương n thỏa mãn đề bài là n  3, n  5 .

11
Bài 6.

Có n học sinh ngồi quanh một bàn tròn, trong tay mỗi học sinh có một số kẹo sao cho tổng số
kẹo của tất cả các học sinh này là một bội số của n. Ta thực hiện một quy tắc chuyển kẹo như
sau: nếu có một học sinh có số kẹo lớn hơn số kẹo của người bạn bên tay phải mình thì ta sẽ
lấy đi của người đó chuyển sang cho người bạn bên tay phải.
Chứng minh rằng với quy tắc trên, sau một số hữu hạn các bước với cách chuyển thích hợp,
số kẹo của mỗi học sinh đều bằng nhau.

Lời giải.

Xuất phát từ một học sinh nào đó, lần lượt theo chiều ngược chiều kim đồng hồ, ta đánh số các
em bởi 1, 2, 3,..., n . Khi đó, với mỗi i  1, n , ngồi ngay bên phải em i là em i 1 .

(với quy ước em n 1 là em 1).

Với t  * , ta định nghĩa thời điểm t là thời điểm nằm giữa lần chuyển kẹo thứ t và thứ t 1 .

Với mỗi i  1, n , kí hiệu x là số kẹo của em i tại mỗi thời điểm và gọi xi (t ) là số kẹo của

em i tại thời điểm t. Ta gọi mỗi bộ ( x1 , x2 , x3 ,..., xn ) là một trạng thái.

n
Xét đại lượng F   xi2 và xét một thời điểm t tùy ý. Giả sử, ở lần chuyển kẹo thứ t 1 , người
i 1

thực hiện việc chuyển kẹo là em i. Khi đó:

xi (t )  xi 1 (t ), x j (t  1)  x j (t ), j  i; i  1
xi (t  1)  xi (t )  1, xi 1 (t  1)  xi 1 (t )  1

2 2 2 2
Do đó F (t  1)  F (t )   xi (t )  1   xi 1 (t )  1   xi (t )    xi 1 (t )   2  xi 1 (t )  1  xi (t )   0

Và F (t  1)  F (t )  0  xi (t )  xi 1 (t )  1 (*) . ( F (t ) kí hiệu giá trị của F tại thời điểm t).

Như vậy, giá trị của F không tăng trong quá trình chuyển kẹo và F chỉ nhận cùng một giá trị ở 2
thời điểm liên tiếp là t , t 1 khi và chỉ khi ở lần chuyển kẹo thứ t + 1, em chuyển kẹo có nhiều
hơn em nhận kẹo đúng 1 chiếc kẹo. Hơn nữa, do mỗi trạng thái cho ta một nghiệm tự nhiên của
phương trình

x1  x2  x3  ...  xn  n

nên số trạng thái đôi một khác nhau là hữu hạn. Suy ra, trong quá trình chuyển kẹo, F chỉ nhận
một số hữu hạn các giá trị đôi một khác nhau.
12
Xảy ra một trong hai trường hợp sau:

* Trường hợp 1: F nhận mỗi giá trị chỉ tại một số hữu hạn thời điểm liên tiếp .

Trong trường hợp này, sau một số hữu hạn lần chuyển kẹo, F sẽ nhận giá trị nhỏ nhất có thể và
kể từ thời điểm (đầu tiên) F nhận giá trị nhỏ nhất đó, việc chuyển kẹo chỉ có thể thực hiện thêm
một số hữu hạn lần. Khi việc chuyển kẹo không thể thực hiện được, ta phải có

x1  x2  x3  ...  xn  x1 hay x1  x2  x3  ...  xn .

Khẳng định của bài toán được chứng minh.

* Trường hợp 2 : Tồn tại một giá trị mà F nhận giá trị đó tại vô hạn thời điểm liên tiếp .

Giả sử F (t )  F (t  1), t  t0 . Khi đó, theo (*), kể từ thời điểm t0 , mỗi lần chuyển kẹo chỉđược
thực hiện bởi em i mà tại thời điểm đó xi  xi 1  1 (**)

Do kể từ thời điểm t0 ta sẽ nhận được vô số trạng thái mà chỉ có hữu hạn trạng thái đôi một
khác nhau (chứng minh trên) nên phải tồn tại ít nhất một trạng thái xuất hiện tối thiểu 2 lần.

Xét một trạng thái trong số các trạng thái như vậy, gọi là A.

Giả sử A xuất hiện tại thời điểm t1  t0 ; gọi k là số nguyên dương nhỏ nhất sao cho tại thời điểm
t1  k , A lại xuất hiện một lần nữa.

Gọi (C) là quá trình chuyển kẹo kể từ thời điểm t1 đến thời điểm t1  k .

Do (C) khởi đầu và kết thúc bởi cùng một trạng thái nên trong quá trình đó nếu em i đã nhận kẹo
ở một lần chuyển kẹo nào đó thì sau đó i phải thực hiện việc chuyển kẹo cho bạn ngồi ngay bên
phải mình. Từ đó, do các em ngồi quanh bàn tròn, suy ra trong (C), mỗi em đều phải thực hiện
việc chuyển kẹo ít nhất một lần.

Hơn nữa, do tổng số kẹo của các em là một bội của n nên với ( x1 , x2 ,..., xn ) là một trạng thái tùy
ý, ta phải có hoặc x1  x2  x3  ...  xn , hoặc tồn tại i  j  1, 2, 3,..., n sao cho xi  x j  2 .
Suy ra, mỗi trạng thái trong (C) đều có tính chất: tồn tại i  j  1, 2, 3,..., n sao cho xi  x j  2 .

Gọi m là số nguyên dương nhỏ nhất sao cho trong (C) có ít nhất một trạng thái mà ở trạng thái đó
tồn tại i  1, 2, 3,..., n sao cho xi  xi  m  2 . Gọi B là trạng thái có tính chất như vậy và ở gần A
nhất (có thể B  A ). Giả sử B xuất hiện (trong (C)) tại thời điểm t2  t1 .

Xét hai khả năng sau:


13
- Khả năng 1: m  1 , ta lại xét tiếp các trường hợp:

+ Trường hợp 1.1: B  A . Khi đó, ở lần đầu tiên (trong (C)) chuyển kẹo cho em i 1 , em i sẽ có
nhiều hơn i + 1 ít nhất 2 chiếc kẹo; mâu thuẫn với (2).

+ Trường hợp 1.2 : B  A . Khi đó, ở trạng thái A, ta có xi  xi 1  1 . Do đó, để trở lại được
trạng thái A, sau thời điểm t2 , i phải chuyển kẹo cho em i 1 ít nhất 1 lần và ở lần đầu tiên trong
các lần như vậy i sẽ có nhiều hơn i 1 ít nhất 2 chiếc kẹo; mâu thuẫn với (2).

- Khả năng 2: m  1 . Khi đó, từ định nghĩa của m suy ra số kẹo của các em
i, i  1,..., i  m  1, i  m (trong trạng thái B) phải thỏa mãn

xi  xi 1  1, xi 1  xi 1  ...  xi  m1  xi  m  1.

+ Trường hợp 2.1: B  A . Trong trường hợp này, tại lần đầu tiên có một trong các em
i  1, i, i  1,..., i  m thực hiện việc chuyển kẹo; em chuyển kẹo chỉ có thể hoặc là i 1 hoặc i
hoặc i  m 1 hoặc i  m .

 Nếu i 1 thực hiện việc chuyển kẹo thì sau bước chuyển đó, ta sẽ có trạng thái mà trong
đó xi  xi 1  2 .

 Nếu i thực hiện việc chuyển kẹo thì sau bước chuyển đó, ta sẽ có trạng thái mà trong
xi 1  xi  m  2 .

 Nếu i  m 1 thực hiện việc chuyển kẹo thì sau bước chuyển đó, ta sẽ có trạng thái mà
xi  xi  m 1  2 .

 Nếu i  m thực hiện việc chuyển kẹo thì sau bước chuyển đó, ta sẽ có trạng thái mà
xi  m 1  xi  m  2 .

Trong cả 4 tình huống trên, ta đều nhận được điều mâu thuẫn với tính nhỏ nhất của m.

+ Trường hợp 2.2 : B  A . Khi đó, ở trạng thái A, các số xi , xi 1 ,..., xi  m1 , xi m sẽ không thỏa
mãn quan hệ (∗). Vì thế, để trở lại trạng thái A, sau thời điểm t2 phải có thời điểm mà ít nhất một
trong các sốđó thay đổi. Điều này chỉ có được khi có ít nhất một trong các em
i  1, i, i  1,..., i  m thực hiện việc chuyển kẹo. Lập luận hoàn toàn tương tự như trường hợp 2.1,
ta sẽ nhận được những điều mâu thuẫn với tính nhỏ nhất của m.

Tóm lại, các mâu thuẫn nhận được ở trên cho thấy trường hợp 2 không thể xảy ra.

Bài toán được chứng minh.


14
Vietnam TST 2012 – Lời giải và bình luận
Trần Nam Dũng & K0

Kỳ thi chọn đội tuyển Việt Nam tham dự IMO 2012 đã diễn ra trong 2 ngày 16 và
17/04/2012 tại Hà Nội. Mỗi ngày thí sinh phải giải quyết 3 bài toán trong vòng 4 giờ 30
phút. Theo đánh giá chung, đề thi năm nay thuộc loại khó. Về phân môn, 6 bài toán được
phân bố như sau:

Bài 1. Hình học phẳng (Quỹ tích và điểm cố định)

Bài 2. Tổ hợp (Phủ)

Bài 3. Số học (Hệ thặng dư)

Bài 4. Số học (Dãy số)

Bài 5. Đại số (Bất đẳng thức)

Bài 6. Tổ hợp (Lý thuyết đồ thị)

Nếu đi sâu vào lời giải thì có thể thấy bài 4 là một bài toán thuần túy đại số. Bài 3 là bài
số học nhưng mang đậm chất tổ hợp. Như thế, có thể thấy đề thi năm nay quá nặng về
Đại số và Tổ hợp, phần Số học và Hình yếu, dù bài hình là một bài toán tốt.

Về độ khó, chỉ có bài 4 là dễ chịu hơn cả, còn lại 5 bài đều là những bài toán khó, đều là
thách thức đáng kể đối với các thí sinh.

Một đặc điểm nữa trong đề thi năm nay là có nhiều bài toán sử dụng ý tưởng các định lý
mạnh như định lý Cauchy-Davenport (bài 3), định lý Dirac, định lý Tutte (bài 6). Điều
này một mặt là tích cực vì hướng học sinh đến việc làm quen với những vấn đề cơ sở của
toán cao cấp, mặt khác cũng tạo những bất lợi cho các học sinh chưa có điều kiện làm
quen với những kiến thức này. Đây là điều mà những người dẫn dắt phong trào HSG của
Việt Nam phải thảo luận kỹ để có một định hướng đúng.

Dưới đây chúng tôi trình bày lời giải chi tiết các bài toán của Vietnam TST 2012 cùng
các bình luận.

Bài viết này được hoàn thành với sự tham gia trực tiếp của các bạn: Võ Quốc Bá Cẩn
(ĐH Y Cần Thơ) và Lê Phúc Lữ (ĐH FPT), Lê Hồng Quý cũng như sự tham gia gián tiếp
của thầy Nguyễn Chu Gia Vượng (Viện Toán học), các thành viên mathscope.org như
chemthan, Mr_Stoke, kien10A1, novae, leviethai, lethanhtu, nghiepdu-socap, …
Bài 1.

Trên mặt phẳng, cho đường tròn (O) và hai điểm cố định B, C trên đường tròn này
sao cho BC không là đường kính của (O) . Gọi A là một điểm di động trên đường
tròn (O) và A không trùng với hai điểm B, C . Gọi D, K , J lần lượt là trung điểm của
BC , CA, AB và E , M , N lần lượt là hình chiếu vuông góc của A, B , C trên
BC , DJ , DK .

Chứng minh rằng các tiếp tuyến tại M , N của đường tròn ngoại tiếp tam giác
EMN luôn cắt nhau tại điểm T cố định khi điểm A thay đổi trên (O) .

Lời giải.

Đây là một bài toán khá thú vị với phát biểu nhẹ nhàng, cấu hình không quá phức tạp và
gợi ra nhiều ý tưởng nhưng việc xử lí không dễ, quan trọng là phải đoán được điểm cố
định được nêu ra. Dưới đây chúng ta sẽ xem xét một số hướng tiếp cận và xử lí mở rộng
của bài toán này.

Cách 1. (sử dụng hàng điểm điều hòa và tứ giác điều hòa)

Gọi H là trực tâm của tam giác ABC. Ta xét trường hợp H nằm trong tam giác, các trường
hợp còn lại chứng minh tương tự.

Trước hết, ta chứng minh rằng T nằm trên đường thẳng OD.

Dễ dàng thấy H cùng nằm trên các đường thẳng BM và CN nên các điểm D, M , N , H , E
cùng thuộc đường tròn đường kính HD.
Đường thẳng qua H, song song với BC cắt đường thẳng OD tại điểm S. Do HSD  900
nên S cũng thuộc đường tròn đường kính HD. Gọi X là hình chiếu của E lên AD thì X
cũng thuộc đường tròn này.

Ta sẽ chứng minh các tứ giác DMSN , XMEN là các tứ giác điều hòa.

Thật vậy, do HS  BC và D là trung điểm của BC nên theo tính chất về chùm điều hòa, ta
có ( HS , HD, HC , HB )  1 hay tứ giác DMSN tương ứng là tứ giác điều hòa. Theo tính
chất của tứ giác điều hòa, ta có T nằm trên đường thẳng DO.

Dễ thấy tứ giác DEJK là hình thang cân nên nên ENK  EMJ ( g .g ) .

EM EJ AB
Suy ra   . Hơn nữa,
EN EK AC

XM sin XNM sin XDM sin DAC AB


    .
XN sin XMN sin XDN sin DAB AC

EM AB
Do đó,  hay tứ giác XMEN điều hòa. Ta có được T nằm trên EX hay T chính
EN AC
là giao điểm của EX và AO.

Ta sẽ chứng minh rằng khoảng cách từ T đến D không đổi.

Gọi B  là hình chiếu của B trên AC. Do AHX  ADE nên

AX  AD  AH  AE  AB   AC
hay tứ giác CDXB  nội tiếp. Suy ra DXC  DB C  DCA  DX  DA  DC 2 .

AE  DX AD  DX DC 2
Theo định lí Thales thì DT    .
AX AH AH

Dễ thấy DC , AH đều không đổi nên độ dài đoạn DT không đổi hay T là điểm cố định.

Ta có đpcm.

Cách 2. (dùng phương tích, trục đẳng phương)

Gọi R, S lần lượt là trung điểm của DB, DC thì R, S lần lượt là tâm đường tròn ngoại
1 1 1
tiếp các tam giác BMD, CND . Ta có TM  TN , MR  DB   BC  DC   NS và
2 4 2
bằng biến đổi góc, ta thu được TMR  TNS hay TMR  TNS (c.g .c) .

Suy ra TR  TS hay T nằm trên đường trung trực của BC.

Gọi X là tâm đường tròn ngoại tiếp tam giác HBC thì X cố định. Ta sẽ chứng minh T nằm
trên trục đẳng phương của đường tròn (S) và (X). Gọi U là trung điểm của OD. Ta thấy

T /( X )  T /( S )  TX 2  XC 2  TS 2  SC 2
 TX 2  TS 2  XD 2  CD 2  SC 2  TD 2  SD 2  XD 2  CD 2  SC 2  TC 2  XD 2
 TD  XD   TC 2  XD 2  CD 2  2TD  XD  DS 2  DU  DT
2
Điều này tương đương với tam giác TSU vuông tại S. Hơn nữa, ta thấy

TSU  900  STU  SUT  900  RTS  BXC  1800


 MTN  MIN  1800

Đẳng thức cuối đúng nên suy ra T nằm trên trục đẳng phương của (S) và (X). Do hai
đường tròn này cố định nên trục đẳng phương của chúng cũng cố định. T là giao điểm của
hai đường thẳng cố định nên T là điểm cố định. Ta có đpcm.

Bình luận.

So sánh với các bài toán hình ở vị trí bài 1 nhiều năm trở lại đây thì bài này khó hơn hẳn.
Hướng giải theo con đường hình học thuần túy bắt buộc phải kẻ thêm khá nhiều đường
phụ và điều này sẽ khiến nhiều bạn phải bỏ cuộc. Có một cách giải quyết trong trường
hợp này là dùng phương pháp tọa độ do giả thiết cũng tương đối thuận lợi. Đôi khi cách
tiếp cận bằng đại số cũng đem lại hiệu quả cao. Chúng ta sẽ cùng tìm hiểu một cách làm
bằng biến đổi vector như sau:

Ta thấy các điểm M, N chính là trung điểm của các đường cao tương ứng của tam giác
ABC. Các điểm M , N , E , H , D cùng thuộc đường tròn đường kính HD. Gọi R là điểm đối
xứng với O qua đường thẳng BC và S là giao điểm của đường tròn ngoại tiếp tam giác
BCR với đường thẳng OD. Gọi F là chân đường cao kẻ từ C đến AB và T là trung điểm
của DS. Dễ thấy T là điểm cố định. Ta tính được
FH  AH  cos B  2OD  cos B  2 R cos A  cos B và
FCS  900 FCR  900  900 A  900 B   A  B  900 và

BC
DCS  900 RCD  900  900 A  A nên CS  .
2cos A

Do T , I , N lần lượt là trung điểm của các đoạn DS , HD, CF nên ta có:
     
2 NT  CS  FD, 2 NI  CD  FH .
             
  
Suy ra: 4NT NI  CS  FD CD  FH  CS  CD  CS  FH  FD  CD  FD  FH .

  BC 2
Ta tính được CS  CD  CD 2   R 2  sin 2 A và
4
 
FD  CD  DF  DC  cos CDF  CD 2 cos 2 B  R 2 sin 2 A cos 2 B .
  BC
FD  FH  FD  FH  cos DFH   2 R cos A cos B sin B  R 2  2sin A cos B sin B cos A .
2
  BC
CS  FH  CS  FH  cos FCS    2 R cos A cos B sin( A  B )
2cos A
 2 R 2 sin A cos B sin( A  B )

Do đó
 
4 NT NI  R 2 sin 2 A  sin 2 A cos 2 B  2sin A cos B sin B cos A  2sin A cos B sin( A  B )
 2 R 2 sin 2 A cos 2 B  sin A cos B sin B cos A  sin( A  B )  0

Từ đó suy ra NT  NI hay TN là tiếp tuyến của đường tròn ngoại tiếp tam giác MNE .
Chứng minh tương tự, ta có TM là tiếp tuyến của đường tròn ngoại tiếp tam giác này.

Do đó, hai tiếp tuyến kẻ từ M và N của đường tròn ngoại tiếp tam giác MNE cắt nhau tại
T là điểm cố định, ta có đpcm.

Bài toán này có nội dung tương tự với mở rộng của bài 2, IMO 2009:

Cho tam giác ABC nội tiếp đường tròn tâm O. Trên các cạnh AC và AB lần lượt lấy
các điểm P và Q. Gọi M, N, J lần lượt là trung điểm của BP, CQ và PQ. Đường tròn
ngoại tiếp tam giác MNJ cắt PQ tại R. Chứng minh rằng OR vuông góc với PQ.
Một kết quả quen thuộc khác cũng có được từ bài toán này là: tiếp tuyến tại H của đường
tròn ngoại tiếp tam giác EMN cắt AB, AC tại hai điểm đối xứng nhau qua H.

Cách giải thứ 2 ở trên khá thuần túy và đẹp mắt, có thể thay việc chứng minh tam giác
bằng nhau ở trên bằng phép quay. Trong trường hợp tam giác tù (tại B hoặc C), hình vẽ
và vị trí các điểm cũng có nhiều thay đổi, chúng ta có thể sử dụng góc định hướng để có
một lời giải tốt hơn!

Lời giải và bình luận của bài 1 được thực hiện bởi Lê Phúc Lữ, dựa trên cách giải của
Hoàng Đỗ Kiên, Phan Đức Minh, Lê Thanh Tú và bản thân người bình luận.

Bài 2.

Trên một cánh đồng hình chữ nhật kích thước m  n ô vuông gồm m hàng và n cột,
người ta đặt một số máy bơm nước vào các ô vuông. Biết rằng mỗi máy bơm nước có
thể tưới nước không những cho ô vuông chứa nó và các ô vuông có chung cạnh với ô
đó mà còn có thể tưới cho các ô vuông cùng cột với nó và cách nó đúng một ô vuông.
Tìm số nhỏ nhất các máy bơm nước cần đặt để các máy bơm đó có thể tưới hết cả cánh
đồng trong hai trường hợp:

1) m  4 .

2) m  3 .

Lời giải.

1) Với m  4 , ta sẽ chứng minh rằng số máy bơm nước nhỏ nhất thỏa mãn yêu cầu đề
bài là n.

Điều kiện đủ là hiển nhiên với cách đặt ở mỗi cột 1 máy bơm ở hàng thứ hai như sau:


X X X … X X X


Chú ý là một máy bơm chỉ tưới được tối đa 6 ô nên điều kiện cần rõ ràng đúng với n  1
và n  2 .

Ta chứng minh điều kiện cần bằng phản chứng. Giả sử tồn tại n sao cho cánh đồng kích
thước 4 n có thể tưới được bằng ít hơn n máy bơm nước. Gọi n0 là số nguyên dương n
nhỏ nhất như vậy. Theo chú ý ở trên ta phải có n0  3 .
Xét cánh đồng kích thước 4 n0 . Theo định nghĩa của n0 , tồn tại một cách xếp k  n0
máy bơm để tưới hết cánh đồng. Vì số máy bơm nhỏ hơn số cột nên phải tồn tại ít nhất
một cột không chứa máy bơm (ta gọi là cột trống).

Bước 1. Ta thấy cột trống không thể là cột ở biên vì nếu cột trống là cột biên, chẳng hạn
là cột thứ nhất thì để tưới được các ô ở cột trống, cột thứ hai phải chứa 4 máy bơm. Khi
đó, bằng cách thêm một máy bơm vào cột 3 hàng 2 (nếu ô này chưa có máy bơm), ta thấy
n0  2 cột còn lại (bỏ đi cột 1 và 2) sẽ được tưới bởi k  4  1  k  3  n0  2 máy bơm,
mâu thuẫn với cách chọn n0 .

Bước 2. Vì cột trống không nằm ở biên, ta xét cột trống đầu tiên từ bên trái sang. Ta giả
sử cột này là cột j. Để tưới được các ô ở cột trống này, tổng cộng ở hai cột hai bên cột
trống này phải có ít nhất 4 máy bơm (*).
Xét các trường hợp sau:
i) Cột j 1 chứa ít nhất 2 máy bơm. Khi đó do các cột từ 1 đến j  2 đều không
trống nên j cột đầu chứa ít nhất j máy bơm. Suy ra n0  j cột sau chứa nhiều
nhất k  j máy bơm. Vì được ngăn cách bởi 1 cột trống nên rõ ràng các máy
bơm này bơm được cho tất cả các ô của cách đồng kích thước 4 n0  j  . Vì
k  j  n0  j nên điều này mâu thuẫn với cách chọn n0 .
ii) Cột j 1 chỉ chứa 1 máy bơm, khi đó, do (*), cột j  1 phải chứa ít nhất 3
máy bơm. Khi đó, do j  1 cột đầu chứa ít nhất j 1  0  3  j  2 máy bơm
nên n0  k  j  2 , tức là bên cạnh cột j  1 còn ít nhất 2 cột nữa. Bây giờ,
bằng cách thêm vào cột 2 hàng j  2 một máy bơm nếu cần, ta thấy cánh đồng
gồm n0   j  1 cột còn lại sau khi bỏ đi j  1 cột đầu có thể được tưới bởi
k   j  2  1  k  j 1 máy bơm. Vì k  j 1  n0   j  1 nên ta nhận
được mâu thuẫn với cách chọn n0 .

Như vậy điều kiện cần được chứng minh. Ta có kết luận: Với cánh đồng 4 n, cần ít
nhất n máy bơm để tưới nước thỏa mãn yêu cầu bài toán.

2) Ta sẽ chứng minh rằng số máy bơm ít nhất để tưới được cánh đồng 3 n là
 n 1
n   . Trước hết ta chứng minh điều kiện đủ. Với n  5 điều kiện đủ là hiển nhiên,
 4 
ta xếp mỗi cột 1 máy bơm là được.

Với n  5, ta có cách xếp sau:


X X
X
X
Từ đây dễ dàng chỉ ra cách đặt máy bơm cho n bất kỳ. Chẳng hạn với n  20 , ta đặt 16
máy bơm như sau.

X X X X X X
X X X X X
X X X X X

Bây giờ ta chứng minh điều kiện cần. Chú ý là một máy bơm nước chỉ có thể tưới được
tối đa 5 ô nên với n  1, 2 , ta thấy rằng cần phải có ít nhất n máy bơm nước mới có thể
tưới được tất cả các ô của cánh đồng 3 n .

Tương tự phần 1), ta sẽ chứng minh bằng phương pháp phản chứng.

 n 1
Đặt f (n)  n    , n  1, 2,3,... .
 4 

Giả sử tồn tại số nguyên dương n sao cho cánh đồng kích thước 3 n có thể được tưới
bởi k  f  n máy bơm nước. Gọi n0 là số nhỏ nhất như vậy. Theo chú ý ở trên n0  3 .

Do f  n0   n0 nên từ đây ta suy ra k  n0 . Như vậy phải có ít nhất 1 cột trống.

Lý luận tương tự như ở phần 1, ta thấy cột trống không thể ở biên. Xét cột trống đầu tiên
từ bên trái sang. Giả sử đó là cột j. Khi đó, để tưới các ô của cột j, hai cột kề bên cột j
phải chứa ít nhất 3 máy bơm nước.

Xét các trường hợp sau:

i) Cột j 1 chứa ít nhất 2 máy bơm nước. Khi đó j cột đầu chứa ít nhất j máy bơm
nước (do các cột từ 1 đến j  2 chứa ít nhất 1, cột j 1 chứa ít nhất 2). Suy ra n0  j
cột còn lại chứa không quá k  j máy bơm nước và các máy bơm này tưới hết các ô của
cánh đồng kích thước 3n0  j  .

 n 1  n  j 1
Ta có k  j  f (n0 )  j  n0  j   0   n0  j   0   f ( n0  j ) nên từ đây
 4   4 
ta suy ra điều mâu thuẫn với cách chọn n0 .
ii) Cột j 1 chỉ chứa 1 máy bơm nước. Khi đó cột j  1 phải chứa ít nhất 2 máy
bơm nước. Như thế j  1 cột đầu chứa ít nhất j  1 máy bơm nước. Suy ra n0   j  1
cột tiếp theo chứa nhiều nhất k   j  1 máy bơm nước. Tiếp tục xét hai trường hợp:

Trường hợp 1. Cột j  2 là cột trống. Khi đó n0   j  2 cột còn lại sau khi bỏ j  2
cột đầu được tưới đủ bởi nhiều nhất k   j  1 máy bơm nước. Ta có

 n 1  n  4 1
k  ( j  1)  f ( n0 )  ( j  1)  n0  ( j  1)   0   n0  ( j  2)   0 
 4   4 
 n  ( j  2) 1
 n0  ( j  2)   0   f ( n0  ( j  2))
 4 

(do j  2 nên j  2  4 ).

Điều này mâu thuẫn với cách chọn n0.

Trường hợp 2. Cột j  2 có ít nhất 1 máy bơm. Khi đó các máy bơm từ cột j  2 đến
cột n0 tưới đủ các ô ở các cột này ( n0   j  1 cột). Theo tính toán ở trên, số máy bơm ở
các cột này không quá k   j  1 . Ta lại có đánh giá

 n 1  n  ( j  1) 1
k  ( j  1)  f ( n0 )  ( j  1)  n0  ( j  1)   0   n0  ( j  1)   0 
 4   4 
 f  n0  ( j  1)

mâu thuẫn với cách chọn n0 .

Bài toán được giải quyết hoàn toàn. Vậy số máy bơm nhỏ nhất để có thể tưới tất cả các ô
 n 1
của cánh đồng 3 n là n   .
 4 

Bình luận.

 Đây là một bài toán hay và thú vị theo nghĩa để giải nó không cần những kiến thức
cao siêu nhưng đòi hỏi những suy luận rất tinh tế. Những bài toán như vậy mang
đậm chất IMO.

 Câu 1) tương đối dễ chịu ngay ở kết quả (điều kiện cần và đủ) lẫn cách chứng
minh. Với câu 2), việc dự đoán đúng kết quả đóng một vai trò hết sức quan trọng.
Nhiều thí sinh TST và cả một số bạn ở ngoài đã có dự đoán sai rằng số máy bơm
cần thiết vẫn là n, từ đó đưa ra những lời giải sai.

 Phương pháp chứng minh được trình bày trong cả hai lời giải được gọi là phương
pháp phản ví dụ nhỏ nhất, nằm trong chủ đề Phương pháp chứng minh phản
chứng hoặc chủ đề Nguyên lý cực hạn.

 Một cách khác để trình bày lời giải bài toán là dùng phép quy nạp toán học.

 Bài này có nét giống với bài 3 trong VietnamTST 2010 nhưng có phần dễ hơn.

Bài 3.

Cho số nguyên tố p  17 . Chứng minh rằng t  3 là số nguyên dương lớn nhất thỏa
mãn điều kiện: Với các số nguyên bất kì a, b, c, d sao cho sao cho abc không chia hết
cho p và a  b  c chia hết cho p thì tồn tại các số nguyên x, y , z thuộc tập
  p  
0,1,...,   1 sao cho ax  by  cz chia hết cho p.
  t  

Lời giải.

Ta sẽ xử lí bài toán này theo các bước sau:

1. Trước hết ta chứng minh với t  3 thì luôn tồn tại x, y , z thỏa mãn bài toán.

 p
Đặt: L    1, S  {ax  by  cz | 0  x, y, z  L} . Yêu cầu bài toán tương đương với
 3 
việc chứng minh S chứa một hệ thặng dư đầy đủ mod p.

Kí hiệu a  b là a  b chia hết cho p, a  b nếu a không đồng dư với b mod p, a 1 là


số nghịch đảo của a theo mod p, S là số phần tử khác nhau của S theo mod p.

2. Bởi vì a  b  c  0 nên S  {ax  by  ( a  b) z | 0  x, y, z  L} . Việc nhân a, b với


cùng b1  0 không làm thay đổi số phần tử của tập S theo mod p. Do đó ta có thể xem
b  1 nên ta có S  {ax  y  ( a  1) z | 0  x, y , z  L} .

3. Do a  0, 1 mod p nên ta chỉ xét ở đây 1  a  p  2 . Chú ý là


S  {ax  y  ( a  1) z | 0  x, y , z  L}  S  {( p  a 1) z  y  ( p  a ) x | 0  x, y, z  L}
Do x, y , z có thể đổi chỗ cho nhau nên ta có vai trò của a và p  a 1 là như nhau nhau
p 1 p 1
nên ta có thể giả sử là a  . Với a  thì k  min  L, 2 L  a  0 .
2 2

4. Với mỗi 0  l  L , đặt X l  a  z  l   y  a  1 z | 0  y  L,0  z  L  l  và


L  L 
Yl  ax  y  a  1 x  l  | 0  x  L  l ,0  y  L thì S   X l    Yl  . Ta có
l 0  l0 

X l  a  z  l   y – a  1 z | 0  y  L, 0  z  L – l 
  y – z  al | 0  y  L, 0  z  L – l 
 l – L  al , l – L  1  al ,, L  al 

là tập gồm 2L – l số nguyên liên tiếp. Và tương tự,

Yl  ax  y – a  1 x  l  | 0  x  L  l , 0  y  L
  y – x – a  1 l | 0  x  L  l , 0  y  L
 l – L – a  1l , l – L – a  1 l  1,, L – a  1l 

là tập gồm 2L –l số nguyên liên tiếp. Dễ thấy với l  k  min  L, 2 L  a  thì

L  al  1  L  l  1  a l  1 và L  a  1l  1  1  L  1 ( a  1)l

L L
nên X
l 0
l chứa tất cả các số nguyên từ L đến L  ak và Y
l 0
l chứa tất cả các số từ

L  k  a  1 k đến L .

Do đó S chứa tất cả các số từ L  k  a  1 k đến L  ak . Dễ thấy vì a, k  1 nên

L  ak  L  k  ( a  1) k   1  2 L  2ak  1  2 L  2a  2k  2  1  2 L  2a  2k 1
Mặt khác do k  min L, 2 L  a nên:

Nếu k  L thì 2 L  2a  2k 1  2 L  2a  2 L 1  4 L  1  p . (Đây là chỗ sử dụng


điều kiện p  17 ).

Nếu k  2 L  a thì 2 L  2a  2k 1  2 L  4 L 1  6 L 1  p .
Vậy S chứa không ít hơn p số nguyên liên tiếp nên ta có S chứa một hệ thặng dư đầy đủ
theo mod p.

p 1
5. Với t  4 thì có thể lấy a  b  1, c  2 và d  thì do
2

  p   p 1 p 1   p   p 1
2   1   x  y  2z   2   1 
  4   2 2   4   2

3 p 1 5
Nên  x  y  2z   p không thể đồng dư 0 mod p, tức là không tồn tại
2 2 2
x, y , z thỏa mãn.

6. Kết hợp các lý luận trên ta có đpcm.

Bình luận.

 Bài toán này xứng đáng là bài toán số 3 của kì thi. Việc chứng minh cho trường
hợp t  4 khá dễ dàng; tuy nhiên, với trường hợp t  3 thì vấn đề phức tạp hơn
nhiều. Với cách giải trên thì ta có thể mở rộng bài toán ra như sau:

Bài toán 3.1.

Cho số nguyên tố p. Tìm số nguyên dương L nhỏ nhất sao cho với mọi bộ ba số nguyên
không đồng dư với nhau đôi một theo mod p và a  b  c chia hết cho p thì với mọi d
đều tồn tại 0  x, y , z  L sao cho ax  by  cz chia hết cho p.

 Quay trở lại với lời giải bài toán ban đầu.

Mấu chốt của vấn đề là tận dụng tính chất: khi nhân cả ba số a, b, c với cùng một số
m  0 theo mod p thì vai trò của a, b, c và ma, mb, mc là tương đương nhau ở trong bài
toán là tương đương nhau ở trong bài toán. Lại có a  b  c  0(mod p ) nên ta có thể
thay c bởi a  b và do đó giảm độ phức tạp đi. Một suy nghĩ tự nhiên là sẽ tìm m sao
cho ma hoặc mb bằng 1. Điều này đơn giản.

Sau các bước trên thì ta sẽ thu được S  ax  y  ( a  1) z | 0  x, y , z  L với


 p
L    1. Khi đến đây thì khi ta thử với a  1 thì
 3 

S   x  y  2 z | 0  x, y , z  L  2 L, 2 L  1,..., 2 L 1, 2 L


gồm các số nguyên liên tiếp.

Lại thử với a  2 thì S  2 x  y  3 z | 0  x, y, z  L  3L, 3L  1,...,3L 1,3L

cũng gồm các số nguyên liên tiếp.

Như vậy qua hai trường hợp a  1 và a  2 thì ta sẽ có ý tưởng là chứngminh S chứa
một tập con có dạng M , M  1,..., M 1, M  .

Chú ý là tập  y  z | 0  x, y, z  L  L, L  1,..., L 1, L nên ta sẽ nghĩ đến việc


đặt x  z  1 và đặt

X l  a  z  l   y – a  1 z | 0  y  L,0  z  L – l    y – z  al | 0  y  L,0  z  L – l 
là tập gồm các số nguyên liên tiếp. Tương tự ta có Y1 nếu đặt z  x  1 .

Đểcó S chứa các số nguyên liên tiếp thì ta cần có X 1 và X l 1 phải giao nhau, ta thu được
điều kiện cần và đủ là a  l  2 L . Vậy ta cần phải có a  2 L . Kiểm tra với Y1 ta thu
được điều kiện tương tự.

Vấn đề còn lại là phải có a  2 L . Quay trở lại ta chú ý ngay đến vai trò của a và
p 1
( a  1)  p  (a  1) là như nhau và một trong hai phải không vượt quá  2 L.
2

 Các cách giải khác :

Hai cách giải dưới đây đều sử dụng đến định lý thuộc lĩnh vực lý thuyết số cộng tính và
tổ hợp sau đây

Định lý Cauchy-Davenport

Cho hai tập các số nguyên A, B và số nguyên tố p. Không có hai phần tử nào của A đồng
dư với nhau theo modun p. Tương tự cho B. Khi đó tập C  a  b | a  A, b  B sẽ
chứa ít nhất min  A  B 1, p phần tử đôi một không đồng dư với nhau mod p.

Đặt A  0,1, 2,..., L thì ta có S  aA  bA  cA . Theo định lý Cauchy-Davenport ta có


 p
S  3 A  3   2 .
 3 

Tuy nhiên, nói chung đây chưa phải là điều ta cần, ta có thể vượt qua điều này bằng một
số cách như sau:
Cách 1. (dựa theo bạn chemthan)

Đặt B  ax  by  cz |1  x, y, z  L 1 và C  ax  by  cz | 1  x, y , z  1 .


Áp dụng định lý Cauchy-Davenport, ta có:

S  min  B  C 1, p và B  3 L 1  2 .

Do đó S  min 3L – 5  C –1,p  min 3L – 6  C ,p .

Nếu ta chứng minh được 3L  6  C  p thì ta chứng minh được bài toán cho t  3 .

 p
Ta có 3L  6  3    9  p 11 nên C phải ít nhất là 11 (ta cần chứng minh).
 3 

Đặt D  a, b, c, a, b, c và

E  a  b, b  c, c  a, a  b  c, b  c  a, c  a  b  a  b, b  c, c  a, 2a, 2b, 2c 

thì dễ thấy D  E  6 . Nếu D và E rời nhau thì ta có C  D  E  12 thỏa mãn.

Nếu D và E không rời nhau thì tồn tại.

- Trường hợp 1 : a  a  b (không xảy ra)

- Trường hợp 2 : a  b  c hay a  c  b  b  b (không xảy ra)

- Trường hợp 3 : a  2b thì có thể dễ dàng chỉ ra C  13 .

Vậy ta có C  11 và ta có đpcm.

Cách 2. (Dựa trên cách trên với cách đặt tập C)


L
Vì a  b  c  0 nên với K  thì ta có S  X , với
2
X  ax  by  cz | K  x, y , z  K   C  C  C  ...  C (K phiên bản C)
Áp dụng định lý Cauchy-Davenport K 1 lần thì X  min  K C  K  1, p .
Ta cần có K C  K  1  p . Ta đếm C . Dễ thấy rằng
D  0, a, b, c, a, b, c, a  b, b  c, c  a chứa các phần tử đôi một không đồng dư
modun p nên C  D  10 .
 p / 3 1
Ta có K C  K  1  9 K  1 . Ta cần chứng minh 9 K  1  p với K    . (*)
 2 
 

Đến đây thử với p  6l  1 hoặc p  6l 1 với l  3 ta thấy (*) luôn đúng.

 Lời giải và bình luận trên đây được thực hiện bởi bạn Lê Hồng Quý (Traum), huy
chương đồng Olympic Toán quốc tế năm 2006.

 Định lý Cauchy – Davenport có nhiều cách chứng minh khác nhau, trong đó có
cách chứng minh dựa vào định lý không điểm tổ hợp (Combinatorial
Nullstenlenzat, bài viết Đa thức và các bài toán tổ hợp đăng trên Tạp chí Toán
học và Tuổi trẻ (6/2009)). Một cách chứng minh khác dùng đa thức nhiều biến là :
http://planetmath.org/encyclopedia/ProofofCauchyDavenportTheorem.html

Dưới đây chúng tôi trình bày một cách chứng minh sơ cấp cho định lý này. Cảm ơn bạn
Nguyễn Ngọc Trung (chemthan), huy chương vàng Olympic Toán quốc tế năm 2010, đã
cung cấp tư liệu tiếng Anh.

Đặt A  a1 , a2 ,..., am  , B  b1 , b2 ,..., bn  . Ta chứng minh k  A  B  m  n 1 (theo


mod p). Ta giả sử m  n và sẽ chứng minh quy nạp theo m.

Với m  1 , mệnh đề đúng vì từ a1  bi  a1  b j (mod p ) nếu i  j nên

a1  B  B  n  1  n 1 .

Khi đó các bất đẳng thức 0  F  m  n  G , F  G  p và giả thiết quy nạp cho ta
mệnh đề đúng đối với các tập hợp F và G.

Giả sử mệnh đề đã đúng với mọi hai tập hợp X và Y sao cho X  m, X  Y và
X  Y  p . Giả sử A  m  1 và B  n , trong đó m  n và m  n  p . Khi đó
n  p và do đó tồn tại c  B .Ta chọn các phần tử a1 , a2 khác nhau thuộc A. Vì dãy
c  t a2  a1  (mod p ) với t  1,2,3,..., p 1 chứa tất cả các số dư, trừ c nên
b  c  t  a2  a1   B với t nào đó. Gọi t là số nhỏ nhất có tính chất này.

Tập hợp A  b  a2   A chứa phần tử b  a2  a1 , b  a2  a2  b . Chú ý rằng


b  a2  a1  c  t 1a2  a1   B . Vì A  B  b  a2   A  B nên ta chỉ cần
chứng minh rằng A  B  m  n 1 .
Đặt F  A  B và F  A  B . Vì b  F , b  a2  a1  F và b  a2  a1  A nên F là
tập con thực sự khác rỗng của A . Như vậy B là tập con thực sự của G.

Từ đây suy ra 0  F  m  n  G .

Mặt khác m  n  A  B  A  B  A  B  F  G .

Ta cũng chú ý rằng F  G  A  B (Với f  F , g  G , ta có thể giả sử rằng g  A và


khi đó f  F  B suy ra rằng f  g  A  B ). Suy ra A  B  F  G .

Khi đó các bất đẳng thức 0  F  m  n  G , F  G  p và giả thiết quy nạp cho ta
mệnh đề đúng đối với các tập hợp F và G. Do đó

A  B  A  B  F  G  F  G 1  A  B 1  m  n 1

và phép quy nạp được hoàn tất.

 Theo lời giải đầu tiên thì bài toán vẫn đúng cho p  13 .

 Đây là một bài toán khó và nó gần với một bài toán tổ hợp hơn là một bài toán số
học (về phương pháp chứng minh và lập luận)

 Các bạn có thể tham khảo một bài toán có ý tưởng giải tương tự như sau:

Bài toán 3.2. Cho p  3 là số nguyên tố và a1 , a2 , a3 ,..., a p2 là dãy các số tự nhiên sao
cho p không chia hết ak và akk 1 . Chứng minh rằng ta có thể chọn ra một số số hạng
của dãy số để tích của chúng có số dư là 2 khi chia cho p.

Hướng dẫn. Dùng căn nguyên thủy đưa về bài toán cộng tính.

Bài 4.

 x1  1, x2  2011,
Cho dãy số nguyên dương  xn  được xác định bởi 
 xn 2  4022 xn1  xn , n  
*

x2012  1
Chứng minh rằng là số chính phương.
2012

Lời giải.
Đây chính là bài toán nhẹ nhàng và quen thuộc nhất trong đề thi lần này. Để đơn giản hơn
trong việc dùng kí hiệu và hiểu rõ bản chất vấn đề, ta sẽ phát biểu và chứng minh bài toán
tổng quát như sau:

Cho p là số nguyên dương lẻ lớn hơn 1.

 x1  1, x2  p,
Xét dãy số nguyên dương  xn  được xác định bởi 
 xn 2  2 pxn 1  xn , n  
*

x p 1  1
Chứng minh rằng là số chính phương.
p 1

Bài toán này có một số lời giải như sau.

Cách 1 (dùng công thức tổng quát của dãy và biến đổi trực tiếp).

Phương trình đặc trưng của dãy số đã cho là t 2  2 pt  1  t 2  2 pt  1  0 có   p 2  1  0


nên phương trình có hai nghiệm là t1  p  p 2  1, t2  p  p 2  1 .

Công thức tổng quát của dãy đã cho là xn  At1n  Bt2n , n  1, 2,3,...

Thay n  1, 2 tương ứng với hai số hạng cho trước của dãy, ta được hệ phương trình sau:

 At1  Bt2  1
 2 2
 At1  Bt2  p

t2 t t n 1  t2n 1
Giải hệ này, ta thu được A  , B  1 hay xn  1 , n  1, 2,3,...
2 2 2
2
t1p  t2p  2  t1  t2 
p /2 p /2
x p 1  1
Suy ra   .
p 1 2( p  1) 2( p  1)

Chú ý rằng t1  t2  2 p, t1t2  1 nên t1  t2  t1  t2  2 t1t2  2( p  1) .

Hơn nữa, ta cũng có S n  t1n  t2n  , n  1 vì S1 , S 2   và S n 2  2 pS n1  S n .

Đặt t1  a, t2  b thì a  b  2( p  1), ab  1 và t1p /2  t2p /2  a p  b p .

p 1 p 1
p 1 i
p p
Ta có a  b  (a  b) (1) a b i i
 2( p  1)  (1)i a i b p 1i . Xét biến đổi sau:
i0 i0
p 3
p 1 2 p 1 p 1 p 1
i i p 1 i i i p 1 i i i p 1 i
 (1) a b
i 0
  (1) a b
i 0
 (1) 2
(ab) 2
  (1) a b
p 1
i
2
p 3 p 3
2 p 1 p 1 2 p 1 2i p 1 p 1 2 i p 1
  (1)i (ab)i b p 12i   (1)i ( ab)i a p 1 2i  ( 1) 2
  ( 1) i t2 2
  ( 1) i t1 2
 ( 1) 2

i 0 p 1 i 0 p 1
i i
2 2
p 3 p 3
2  p 1 2i p 1 2 i
 p 1 2 p 1
  (1)i t1 2  t2 2   (1) 2   (1)i S p 1 2i  (1) 2  N  
i 0   i0 2

2
Do đó t1p /2  t2p /2  N 2( p  1)   t1p /2  t2p /2   N 2  2( p  1) .

x p 1  1 N 2  2( p  1)
Vậy   N 2 là số chính phương. Ta có đpcm.
p 1 2( p  1)

Cách 2. (Xét dãy số phụ và dùng quy nạp).

Ta thấy rằng

x2  1 p  1
x2  p    1,
p 1 p 1
x4  1 4 p 3  3 p  1
x3  2 p 2  1, x4  4 p 3  3 p    (2 p  1) 2 ,
p 1 p 1
x 1
x5  8 p 4  8 p 2  1, x6  16 p5  20 p 3  5 p  6  (4 p 2  2 p  1) 2 .
p 1

Từ công thức truy hồi là xn 2  2 pxn1  xn , ta có 2 pxn 1  xn  xn  2 . Suy ra

xn  2  2 p  2 pxn  xn 1   xn  4 p 2 xn  2 pxn 1  xn  (4 p 2  2) xn  xn  2 , n  3 .

x2 n  1
Ta sẽ xây dựng công thức của dãy yn  , n  1, 2, 3,... và chứng minh các số hạng
p 1
của dãy này đều nguyên.

 y1  1, y2  2 p  1
Xét dãy  yn  thỏa mãn  với a, b được chọn sau.
 yn 2  ayn 1  byn , n  1

Do y3  4 p   2 p  1 nên a(2 p  1)  b  4 p 2  2 p  1 ; ta có thể chọn a  2 p, b  1 .


 y1  1, y2  2 p  1
Dãy số tương ứng là  .
 yn 2  2 pyn 1  yn , n  1

x2 n  1
Ta sẽ chứng minh bằng quy nạp rằng yn2  , n  1 . (*)
p 1

Với n  1, 2 , khẳng định (*) đúng.

x2n  1 2 x 1
Giả sử ta có yn2  , yn1  2 n 2 .
p 1 p 1

Ta có yn 2 yn  yn21  (2 pyn1  yn ) yn  yn1 (2 pyn  yn 1 )  yn 1 yn1  yn2 , n .

Hơn nữa y3 y1  y22  2 p  2 nên yn 2 yn  yn21  2 p  2, n .

Từ công thức xác định dãy thì

yn  yn  2  2 pyn1  yn2  yn2 2  2 yn yn  2  4 p 2 yn21  yn2  yn2 2  2  2 p  2  yn21   4 p 2 yn21 hay

x2n  2  1 x2 n  1
yn2 2  (4 p 2  2) yn21  yn2  4( p  1)  (4 p 2  2)   4( p  1)
p 1 p 1
(4 p 2  2) x2n  2  x2 n  1 x2 n 4  1
 
p 1 p 1

Khẳng định (*) cũng đúng với n  2 . Theo nguyên lí quy nạp, (*) được chứng minh.

xn  1
Do đó, ta đã chứng minh được với mọi n chẵn thì là số chính phương; nói riêng, ta
p 1
x p 1  1
cũng có cũng là số chính phương. Ta có đpcm.
p 1

Bình luận.

Cách thứ nhất có thể kết hợp thêm quy nạp để chứng minh t1p / 2  t2p /2  N 2( p  1) cho
đơn giản thay vì biến đổi trực tiếp.

Cách thứ hai của bài toán này dựa trên một định lí về dãy số gồm toàn số chính phương
như sau:

Cho dãy số  un  xác định như sau u1  a, u2  b, un 2  cun1  un , n  1 .


Xét dãy số mới là  vn  với v1  a 2 , v2  b 2 , vn  2  (c 2  2)vn1  vn  2  u1u3  u22  , n  1 .

Khi đó vn  un2 với mọi số nguyên dương n.

Ý tưởng để chứng minh định lí này hoàn toàn đã được áp dụng vào lời giải thứ hai ở trên.
Cũng chính cách này đã cho ta một kết quả mạnh hơn bài toán đã cho là khẳng định vẫn
đúng khi thay x p1 bởi xn với n chẵn bất kì.

Ngoài ra, ta có thể dựa trên tính chất của dãy số ( xn ) là xn 2 xn  xn21  p 2  1 để suy ra rằng

2
 xn  2  1  xn  1   xn 1  1 
    và dùng quy nạp để có được điều phải chứng minh.
 p  1  p  1   p  1 

Ta cũng có một hướng tiếp cận khác nữa là:

Xét phương trình Pell có dạng x 2  ( p 2 1) y 2  1 với p là số nguyên dương lẻ.

Ta xét đồng thời hai dãy số được xác định như sau:

 x1  1, x2  p, xn2  2 pxn1  xn
 , n  1, 2,3,...
 y1  0, y2  1, yn2  2 pyn1  yn

Khi đó, bằng quy nạp, ta chứng minh được rằng xn2   p 2 1 yn2  1 và  xn , yn  cũng
chính là tất cả các nghiệm của phương trình Pell nêu trên.

Bằng quy nạp, ta cũng chứng minh được rằng  x2 k 1 p 1 , x2 k  1 p  1 với mọi
 x 1  x2 k  1
k  0 nên có thể viết đẳng thức quan hệ giữa xn , yn thành  2 k     y2 .
 p 1   p  1 

x2 k 1 x2 k  1
Ta cũng chứng minh được rằng cả hai số , nguyên tố cùng nhau nên mỗi
p 1 p  1
số đều phải là số chính phương. Bài toán được giải quyết nhanh chóng và có lẽ đây chính
là cơ sở để xây dựng bài số 4 này!

x2012 1
Từ đây, ta cũng có thể thêm vào bài toán ban đầu một kết quả thú vị nữa là là
2010
một số chính phương.
Ngoài ra, công thức xác định của dãy đã cho có liên quan đến đa thức Chebyshev loại I
như sau:

T0 ( x )  1, T1 ( x)  x, Tn  2 ( x )  2 xTn 1 ( x )  Tn ( x ), n  0 .

Do đó, dựa vào các tính chất đã biết của loại đa thức này, bài toán đã cho có thể phát triển
theo nhiều hướng thú vị hơn.

Lời giải và bình luận bài 4 được thực hiện bởi Lê Phúc Lữ, tham khảo các lời giải của
các bạn Lê Việt Hải, Võ Anh Đức và Hoàng Đỗ Kiên, Nguyễn Huy Tùng.

Bài 5.

Chứng minh rằng C  10 24 là hằng số lớn nhất sao cho nếu có 17 số thực dương
a1 , a2 ,..., a17 thỏa mãn điều kiện a12  a22  ...  a172  24 và

a13  a23   a173  a1  a2    a17  C

thì với mọi 1  i  j  k  17, ta có ai , a j , ak là độ dài ba cạnh của một tam giác.

Lời giải.

Đặt ai  24 xi , i  1, 2, , 17, khi đó yêu cầu bài toán tương đương với:
Chứng minh rằng C  10 là hằng số lớn nhất sao cho nếu có 17 số thực dương
x1 , x2 , , x17 thỏa mãn x12  x22    x172  1 và
24( x13  x23    x173 )  ( x1  x2    x17 )  C
thì với mọi i, j , k thỏa mãn 1  i  j  k  17, ta có xi , x j , xk là độ dài ba cạnh của một
tam giác.

Chứng minh dưới đây gồm hai phần:

(a) Chứng minh hằng số C  10 thỏa mãn yêu cầu đề bài. Không mất tính tổng quát, ta
chỉ cần chứng minh x1 , x2 , x3 là độ dài ba cạnh của một tam giác. Để ý rằng với mọi
0  t  1, ta có
24t 3  t  (16t 4  9t 2 )  t (1  t )(4t 1) 2  0.
Do đó, từ giả thiết ta suy ra
16( x14  x24    x174 )  9( x12  x22    x172 )  10,
hay
16( x14  x24    x174 )  1  ( x12  x22    x172 ) 2 .
Bây giờ, sử dụng bất đẳng thức Cauchy-Schwarz, ta có
 
 1    1 ( x14  x24  x34 )  ( x44  x54    x174 )
16( x14  x24    x174 )  2  1
 14 so 1 

  2( x14  x24  x34 )  x42  x52    x172  .


2

 
Từ đây kết hợp với trên, ta thu được
2( x14  x24  x34 )  x12  x22  x32 ,
hay
2( x14  x24  x34 )  ( x12  x22  x32 ) 2 .

Đến đây, bằng cách sử dụng đồng nhất thức

( x12  x22  x32 ) 2  2( x14  x24  x34 )  2( x1  x2  x3 )( x1  x2  x3 )( x2  x3  x1 )( x3  x1  x2 ),

ta dễ dàng suy ra được x1 , x2 , x3 là độ dài ba cạnh của một tam giác.

(b) Chứng minh C  10 là hằng số lớn nhất thỏa mãn yêu cầu đề bài. Giả sử tồn tại hằng
số C   10 thỏa mãn yêu cầu đề bài. Khi đó, ta xét 17 số dương x1 , x2 , , x17 với
1 1 1   2 1
x1  , x2    , x3  , x4    x17   , 0 .
4 16 16 14 16
Lúc này, dễ thấy x12  x22    x172  1. Ngoài ra, ta cũng có
1 2
1
x1  x3         1     2  1    1    x2
4  4  16 2 16 2 16
nên x1 , x2 , x3 không phải là độ dài ba cạnh của một tam giác.

Bây giờ, ta sẽ chứng minh rằng, bằng cách chọn  thích hợp, các số x1 , x2 , , x17 sẽ
thỏa mãn bất đẳng thức
24( x13  x23    x173 )  ( x1  x2    x17 )  C .
Rõ ràng chứng minh được điều này cũng có nghĩa là ta đã chứng minh được C  10 là
hằng số lớn nhất thỏa mãn yêu cầu của bài toán.
Ta có bất đẳng thức trên tương đương với
  1    2 3  1
1 1 3 1 1   2

24  3      3  14            14   C .
4 16  16 14   4 16 16 14
 
 1
Dễ thấy vế trái là một hàm liên tục của  trên 0, . Ngoài ra, khi   0 thì
 16 
  1 
3
 1 
3 
1  1 1 1
VT  24  3     14  
16     14  10  C .
4 16   4 16 16
 
 1
Do đó, theo tính chất của hàm liên tục, ta thấy rằng tồn tại một giá trị 0  0,  sao
 16 
cho tại   0 thì VT  C . Lúc này, ta có bộ số
1 1 0  02 1 0  02 
 1
( x1 , x2 , , x17 )   ,  0 , 0 ,  , ,  
 4 16 16 14 16 14 
thỏa mãn đồng thời các điều kiện x12  x22    x127  1 và
24( x13  x23    x173 )  ( x1  x2    x17 )  C .
Nhưng trong chúng có ba số x1 , x2 , x3 không lập thành độ dài ba cạnh của một tam giác.
Điều này mâu thuẫn với giả thiết C   10 là hằng số thỏa mãn yêu cầu đề bài. Vậy
Cmax  10.

Bình luận.
Lời giải ở trên đã sử dụng đánh giá để đưa về xét bất đẳng thức
16( x14  x24    x174 )  ( x12  x22    x172 ) 2 ,
rồi từ đó suy ra xi , x j , xk (1  i  j  k  17) là độ dài ba cạnh của một tam giác.

Như vậy, từ việc giải bài toán đã cho, ta thu được một bài toán khác là:

Bài toán 5.1. Cho các số dương x1 , x2 , , x17 thỏa mãn


( x12  x22    x172 ) 2  16( x14  x24    x174 ).
Chứng minh rằng, với mọi 1  i  j  k  16, ta có xi , x j , xk là độ dài ba cạnh của một
tam giác.

Một điều thú vị là kết quả tổng quát của bài toán 5.1 vẫn đúng và trên thực tế, trong lịch
sử các kỳ thi Olympic, bài toán này đã từng xuất hiện trong một đề thi, đó là một trong
các câu hỏi của đề thi Olympic Toán Trung Quốc năm 1988:
Bài toán 5.2. Cho số tự nhiên n  3 và a1 , a2 , , an là các số thực dương thỏa mãn
(a12  a22   an2 ) 2  ( n 1)( a14  a24    an4 ).
Chứng minh rằng, với mọi 1  i  j  k  n, ta có ai , a j , ak là độ dài ba cạnh của một
tam giác.

Cách giải của bài toán tổng quát hoàn toán giống với bài toán 5.1, đó là từ giả thiết, ta sẽ
tìm cách đánh giá đưa về ba biến rồi sau đó sử dụng khai triển quen thuộc
(a 2  b 2  c 2 ) 2  2( a 4  b 4  c 4 )  ( a  b  c)(b  c  a )(c  a  b)(a  b  c )
để suy ra điều phải chứng minh.

Qua lời giải được trình bày ở trên, có thể thấy được đánh giá
24t 3  t  16t 4  9t 2 , t  (0, 1)
chính là mấu chốt, là chìa khóa quan trọng để giải bài toán. Và khi giải bài toán này,
chúng tôi có cảm giác rằng tác giả bài toán đã phát hiện ra đánh giá
24t 3  t  16t 4  9t 2 , t  (0, 1) trước rồi sau đó đi ngược từ bài toán 5.2 để đi đến bài
toán đã cho. Rất có thể đây chính là cách mà tác giả đã tạo ra bài toán.

Việc nhận ra được bài toán gốc cũng như bất đẳng thức trung gian
24t 3  t  16t 4  9t 2 , t  (0, 1)
là điều không hề dễ dàng gì, và chính điều này đã gây khó khăn cho không ít thí sinh
trong kỳ thi vừa qua. Tuy nhiên, theo quan điểm cá nhân của mình, chúng tôi nghĩ rằng
việc sử dụng “mẹo” che giấu đề làm khó bài toán như thế là không nên. Nên có những
bài toán với ý tưởng tự nhiên hơn.

Lời giải và bình luận bài 5 này được thực hiện bởi Võ Quốc Bá Cẩn.

Bài 6.

Có 42 học sinh tham dự kì thi chọn đội tuyển Olympic toán quốc tế. Biết rằng một học
sinh bất kì quen đúng 20 học sinh khác.

Chứng minh rằng ta có thể chia 42 học sinh thành 2 nhóm hoặc 21 nhóm sao cho số
học sinh trong các nhóm bằng nhau và 2 học sinh bất kì nào trong cùng nhóm thì
quen nhau.

Lời giải.

Cách 1. (Của thầy Nguyễn Chu Gia Vượng)


Bài toán có thể phát biểu dưới dạng đồ thị một cách hiển nhiên: một đồ thị trên 2n  2
đỉnh, bậc mỗi đỉnh bằng n thì hoặc là hợp rời của 2 đồ thị đầy đủ trên n  1 đỉnh hoặc có
thể tìm được n  1 cạnh với đầu mút đôi một rời nhau.

Ta xét bổ đề sau: Một đồ thị liên thông trên N đỉnh, bậc nhỏ nhất bằng d (bậc mỗi đỉnh
không ít hơn d) luôn có một đường đi có độ dài không ít hơn min 2d , N 1 .

Quay trở lại bài toán, nếu G không liên thông thì hiển nhiên điều kiện về bậc cho thấy G
được tạo thành từ hai đồ thị đầy đủ trên n  1 đỉnh.

Giả sử G liên thông, thế thì theo bổ đề trên, G có một đường đi có độ dài 2n hoặc 2n  1 .

Nói rằng G có đường đi độ dài 2n  1 nghĩa là nói G có đường đi Hamilton (đi qua
2n  2 đỉnh, mỗi đỉnh 1 lần), trường hợp này hiển nhiên cho ta lời giải bài toán.

Giả sử G có đường đi độ dài 2n: A1B1 A2 B2 ... An Bn An1 . Gọi đỉnh còn lại là An2 . Nếu An2
kề Ai với i nào đó thì các cạnh An2 Ai , Aj B j , j  i và B j A j1 , j  i thỏa mãn bài toán.
Giả sử An2 không kề với bất kì Ai , thế thì do bậc của An2 bằng n nên ta suy ra An2 kề
Bi với mọi i.

Lập luận như trên với đường đi An2 B1 A2 ... An Bn An1 (thay đổi vai trò của A1 với An2 ) ta
suy ra A1 kề Bi với mọi i.

Tương tự, suy luận với đường đi A1B1 An2 B2 A3 B3 ... An Bn An1 (thay đổi vai trò của A2 với
An2 ) ta suy ra A2 kề Bi với mọi i.

Bằng cách này, ta dễ dàng suy ra Aj kề Bi với mọi i, j. Thế nhưng khi đó mỗi đỉnh Bi có
bậc bằng n  2 , vô lý.

Ta có điều phải chứng minh.

Để kết thúc phép chứng minh, ta sẽ chứng minh bổ đề.

Gọi G là đồ thị có liên thông có N đỉnh và bậc mỗi đỉnh không nhỏ hơn d. Với X là đồ thị
bất kỳ ta ký hiệu V ( X ), E ( X ) tương ứng là tập các đỉnh, các cạnh của X.

Trước hết ta có nhận xét sau:


Nếu P là một đường đi trong G. Nếu tồn tại phần tử x  V (G ) \ V ( P ) thì tồn tại
y  V (G ) \ V ( P ) kề với P.
Thật vậy, giả sử x  V (G ) \ V ( P ) và v  V ( P ) . Do tính liên thông, tồn tại đường đi Q từ x
đến v. Lấy đỉnh y là đỉnh kề trước của v trong Q ta được điều phải chứng minh.

Bây giờ gọi P  a...b là đường đi dài nhất trong G. Gọi k là độ dài của P và giả sử
k  min 2d , N 1 .
Nếu ab  E (G ) thì yzPabPz * là đường đi trong G dài hơn P (đường đi từ y đến z, từ z
theo P đến a , đến b, từ b theo P đến z* là đỉnh kề với z trong P).

Nếu ab  E (G ) , gọi K  V ( P ) là tập các đỉnh trước của các đỉnh kề a trong P (với P
được sắp thứ tự từ a đến b). Vì tất cả các đỉnh kề của a phải nằm trong P, ta có K  d .
Vì k  2 d , ta có V ( P) \ K  \ b  d . Như vậy tồn tại q  K kề với b. Bây giờ ta xét chu
trình C  aPqbPq*a , khi đó yzCz* (theo chiều bất kỳ của C) là đường đi trong G dài hơn
P, mâu thuẫn.

Từ mâu thuẫn ở trên, ta suy ra k  min 2d , N 1 . Bổ đề được chứng minh.

Cách 2. (Của Traum@ dựa trên ý tưởng của mnnn@ )

Nhắc lại một cặp ghép (matching) hay tập cạnh độc lập của một đồ thị G là một tập các
cạnh không có đỉnh chung. Cặp ghép cực đại (maximal matching) của G là cặp ghép mà
không thể mở rộng được nữa.

1. Gọi S là một cặp ghép cực đại của G. Nếu S  21 thì ta có điều phải chứng minh.

2. Nếu S  20 , gọi V(S) là tập các đỉnh trong S.

3. Gọi A và B là hai đỉnh bất kì không thuộc V(S). Vì S là một cặp ghép cực đại nên A và
B không kề nhau.

4. Gọi N ( A), N ( B) là tập các đỉnh kề với A, B trong V(S) tương ứng thì ta có
N ( A)  N ( B )  20 , do bậc mỗi đỉnh là 20.

5. Với mỗi C thuộc N(A), ta gọi đỉnh còn lại của cạnh chứa C trong S là C(S). Đặt M(A)
là tập các đỉnh C(S) trên thì ta có M ( A)  N ( A)  20 .

6. Nhận thấy rằng mỗi đỉnh trong M(A) không kề với B. Thật vậy nếu có C(S) trong M(A)
kề với B thì ta có thể thay cạnh C&C(S) bởi 2 cạnh A&C và C(S)&B. Ta thu được một
cặp ghép mới lớn hơn S, điều này mâu thuẫn với giả thiết lớn nhất của S.
Vậy M ( A)  N ( B)  0 .

7. Bởi vì M ( A)  20 và N ( B )  20 suy ra V ( S )  M ( A)  N ( B)  40 , mà ở trên ta


có 40  V ( S ) nên V ( S )  40 hay S  20 .

8. Từ 6. và 7, ta thấy rằng nếu C kề với A mà C(S) không kề với A thì ta có C kề với B và


S(C) không kề với B.

9. Gọi X(A) là tập các đỉnh C trong V(S) mà C và C(S) đều kề với A. Tương tự ta có định
nghĩa cho X(B). Gọi Y là tập các đỉnh kề với cả A và B và Z là các đỉnh không kề với cả A
và B. Theo các nhận xét 6, 7, 8, ta có Y  Z và X ( A)  X ( B ) và X ( A)  Y  20 .

10. Ta có các điều sau:

a. Các đỉnh thuộc X(A) chỉ có thể kề với các đỉnh thuộc X(A) hoặc Y.

b. Các đỉnh thuộc X(B) chỉ có thể kề với các đỉnh thuộc X(B) hoặc Y.

c. Các đỉnh thuộc Z chỉ có thể kề với các đỉnh thuộc Y.

11. Các đỉnh thuộc Z chỉ kề với các đỉnh thuộc Y. Vì các đỉnh thuộc Z chỉ kề với các đỉnh
thuộc Y nên ta có 20 Z  18 Y , mà Y  Z nên Y  Z  0 .

Vậy ta có X ( A)  X ( B )  20 . Mà theo 10. thì các đỉnh thuộc X(A) chỉ kề với A và các
đỉnh thuộc X(A). Nên ta có 21 đỉnh gồm A và các đỉnh trong X(A) tạo thành một K 21 .
Tương tự ta có một K 21 nữa tương ứng với B.

12. Bài toán được chứng minh xong.

Bình luận.

 Đây là một bài toán về sự tồn tại cặp ghép hoàn hảo (perfect matching) trong một
đồ thị có bậc chẵn. Vì thế xu hướng sử dụng các suy luận liên quan đến cặp ghép
là rất rõ ràng.

 Lời giải 1 được định hướng bởi điều kiện rất “Dirac” của đề bài. Chú ý là theo
phiên bản cơ sở của định lý Dirac thì đồ thị trên n đỉnh có bậc của mỗi đỉnh không
n
nhỏ hơn sẽ Hamilton, và như thế nếu n chẵn sẽ có gặp ghép hoàn hảo. Tuy
2
nhiên, nếu sử dụng đúng định lý Dirac thì không giúp ích. Ở đây, ta cần đến một
biến thể của định lý Dirac, chính là bổ đề trong lời giải. Bổ để này thực chất là một
bài tập trong các tài liệu về lý thuyết Graph. Chứng minh nêu trên của bổ đề được
tham khảo tại đây:
http://www.mpi-inf.mpg.de/departments/d1/teaching/ss11/graph_theory
/exercises/Graph_Theory_sol_02.pdf
 Ngoài cách phát biểu như ở định lý Dirac nguyên thủy, ta còn có một cách phát
biểu khác cho định lý Dirac, tổng quát hơn và rất gần với bổ đề:
Định lý (Dirac). Cho G là đồ thị 2-liên thông với bậc mỗi đỉnh không nhỏ hơn d.
Khi đó G chứa chu trình độ dài ít nhất là min 2d , V (G )  .

Có thể tham khảo về chứng minh định lý này tại:


http://math.fau.edu/locke/Dirac.htm

 Lời giải 2 cũng khá tự nhiên, theo hướng chứng minh nếu không tồn tại cặp ghép
hoàn chỉnh thì G phải là hợp của 2 phiên bản K 21 rời nhau. Cũng như trong lời
giải 1, việc sử dụng nguyên lý cực hạn (cặp ghép cực đại) đem lại những thông tin
bổ sung quan trọng (A, B thuộc V (G ) \ S không kề nhau) dẫn đến lời giải bài toán.
 Ngoài hai cách giải trên còn một cách giải khác sử dụng đến một kết quả về cặp
ghép hoàn hảo là định lý Tutte sau:
Định lý Tutte. Đồ thị G = (V, E) có cặp ghép hoàn hảo khi và chỉ khi với mọi tập
con U của V, số thành phần liên thông có số đỉnh lẻ trong đồ thị con cảm sinh bởi
V \ U nhỏ hơn hay bằng U .

Chứng minh định lý Tutte có thể tham khảo tại:


http://en.wikipedia.org/wiki/Tutte_theorem
Lời giải bài 6 sử dụng định lý Tutte tham khảo tại:
http://forum.mathscope.org/showthread.php?t=30511&page=3

 Đây là một bài toán khó đối với các thí sinh không quen thuộc với lý thuyết đồ thị,
nhưng đối với các bạn đã biết về cặp ghép hoàn hảo hoặc định lý Dirac (đặc biệt là
phương pháp suy luận đường đi dài nhất – longest path argument) thì bài này
không quá khó. Đây là điểm khiến chúng tôi đánh giá tính phân loại của bài này
không cao bằng bài 2, và không những thế, tạo nhiều lợi thế cho những bạn “biết
đúng chỗ”.
LỜI GIẢI VÀ BÌNH LUẬN
ĐỀ CHỌN ĐỘI TUYỂN QUỐC GIA
DỰ THI IMO NĂM 2013

Những người thực hiện: Trần Nam Dũng


Trần Quang Hùng
Võ Quốc Bá Cẩn
Lê Phúc Lữ

Xin chân thành cảm ơn thầy Nguyễn Tăng Vũ, thầy Trần Quốc Luật, các bạn Nguyễn
Văn Quý, Võ Anh Đức, Hoàng Đỗ Kiên, Phạm Tuấn Huy, Nguyễn Huy Tùng cùng
nhiều thành viên của các diễn đàn đã đóng góp ý kiến để chúng tôi hoàn tất tài liệu này!

Thành phố Hồ Chí Minh, ngày 06 tháng 05 năm 2013


1
Phần 1.

ĐỀ THI CHÍNH THỨC

Ngày thi thứ nhất.

Bài 1.

Cho tứ giác ABCD có các cạnh đối không song song nội tiếp đường tròn (O , R) .
Gọi E là giao điểm hai đường chéo và đường phân giác góc AEB cắt các đường thẳng
AB, BC , CD , DA lần lượt tại các điểm M , N , P , Q .
1. Chứng minh rằng các đường tròn ( AQM ),( BMN ),(CNP),( DPQ) cùng đi qua một
điểm duy nhất. Gọi điểm đó là K .
2 R2
2. Đặt min  AC , BD  m . Chứng minh rằng OK  .
4R2  m2
Bài 2.

1. Chứng minh rằng tồn tại vô số số nguyên dương t sao cho 2012t  1 và 2013t  1 đều
là các số chính phương.

2. Xét m, n là các số nguyên dương sao cho mn  1 và ( m  1)n  1 đều là các số chính
phương. Chứng minh rằng n chia hết cho 8(2m  1) .

Bài 3.

Với mỗi số n nguyên dương, đặt Sn  0,1, 2,..., 2n  1 . Xét hàm số f : (  Sn )  [0;1]
thỏa mãn đồng thời các điều kiện sau:

i/ f ( x ,0)  f ( x , 2n  1)  0 với mọi số nguyên x.

ii/ f ( x  1, y)  f ( x  1, y )  f ( x , y  1)  f ( x , y  1)  1 với x , y   và 1  y  2n .

Gọi F là tập hợp tất cả các hàm số f thỏa mãn.

1. Chứng minh rằng F là vô hạn.

2. Với mỗi hàm số f  F , đặt v f là tập hợp ảnh của f . Chứng minh rằng v f hữu hạn.

3. Tìm giá trị lớn nhất của v f với f  F .

2
Ngày thi thứ hai.

Bài 4.

Tìm hằng số k nguyên dương lớn nhất thỏa mãn: Với mọi a , b , c dương mà abc  1 thì
ta có bất đẳng thức sau

1 1 1 k k
    3.
a b c abc1 4

Bài 5.

Cho tam giác ABC nhọn không cân có A bằng 45 . Các đường cao AD , BE, CF đồng
quy tại trực tâm H . Đường thẳng EF cắt đường thẳng BC tại P . Gọi I là trung điểm
của BC ; đường thẳng IF cắt PH tại Q.

1. Chứng minh rằng IQH  AIE .

2. Gọi K là trực tâm của tam giác AEF và ( J ) là đường tròn ngoại tiếp tam giác KPD .
Đường thẳng CK cắt ( J ) tại G , đường thẳng IG cắt ( J ) tại M , đường thẳng JC cắt
đường tròn đường kính BC tại N . Chứng minh rằng các điểm G , M , N , C cùng thuộc
một đường tròn.

Bài 6.

Cho một khối lập phương 10  10  10 gồm 1000 ô vuông đơn vị màu trắng. An và Bình
chơi một trò chơi. An chọn một số dải 1  1  10 sao cho hai dải bất kì không có chung
đỉnh hoặc cạnh rồi đổi tất cả các ô sang màu đen. Bình thì được chọn một số ô bất kì của
hình lập phương rồi hỏi An các ô này có màu gì. Hỏi Bình phải chọn ít nhất bao nhiêu ô
để với mọi câu trả lời của An thì Bình luôn xác định được những ô nào là màu đen?

3
Phần 2.

LỜI GIẢI CHI TIẾT VÀ BÌNH LUẬN

Bài 1.

Cho tứ giác ABCD lồi có các cạnh đối không song song nội tiếp đường tròn (O , R) .
Gọi E là giao điểm hai đường chéo và đường phân giác góc AEB cắt các đường thẳng
AB, BC , CD , DA lần lượt tại các điểm M , N , P , Q .

1. Chứng minh rằng các đường tròn ( AQM ),( BMN ),(CNP),( DPQ) cùng đi qua một điểm
duy nhất. Gọi điểm đó là K .

2 R2
2. Đặt min  AC , BD  m . Chứng minh rằng OK  .
4R2  m2

Lời giải.

N K

A
B
M
E
O
S
D P C

1. Gọi R là giao điểm của AD , BC và S là giao điểm của AB, CD (do các cạnh đối của
tứ giác ABCD không song song nên các điểm này hoàn toàn xác định). Giả sử B nằm

4
giữa A , S và nằm giữa C , R như hình vẽ. Gọi K là giao điểm của đường tròn ngoại tiếp
các tam giác RAB, SBC thì

BKR  BKS  BAD  BCD  180 hay R , K , S thẳng hàng.

Suy ra RK  RS  RB  RC  RA  RD và SK  SR  SB  SA  SC  SD nên các tứ giác ADSK


và CDRK cũng nội tiếp hay K cũng thuộc về các đường tròn ( RCD) và (SDA).

Do đó, ta có

AKD  ASD  BSC  BKC và ADK  ASK  BSK  BCK

KA AD AE AM
nên các tam giác KAD và KBC đồng dạng. Suy ra    , theo tính
KB BC BE BM
chất đường phân giác thì KM là phân giác của góc AKB.

Mặt khác, ta có RNQ  BNE  CBD  BEN  CAD  AEQ  RQN nên ta được

ARB  2BNM .

1 1
Từ đó suy ra BKM  AKB  ARB  BNM hay tứ giác BMNK nội tiếp, tức là
2 2
K thuộc đường tròn ( BMN ) .

Chứng minh tương tự, ta cũng có K thuộc các đường tròn ( AQM ),(CNP),( DPQ) .

Tiếp theo, ta sẽ chứng minh rằng K là điểm chung duy nhất của các đường tròn này.
Thật vậy, các đường tròn ( AMP),( BMQ) có hai điểm chung là K , M còn các đường tròn
( DNP),(CNP) có hai điểm chung là K , N . Do đó, nếu bốn đường tròn này có hai điểm
chung thì M , N trùng nhau, vô lí.

Ta có đpcm.

2. Theo tính chất phương tích thì

RK  RS  RB  RC  RO 2  R2 , SK  SR  SB  SA  SO 2  R 2 nên

RO 2  SO 2  RK  RS  SK  SR  RK 2  SK 2 .

Từ đó suy ra OK  RS.

Hơn nữa, theo định lí Brocard trong tứ giác nội tiếp ABCD thì E chính là trực tâm của
tam giác ORS , suy ra OE  RS.

Do đó, O , E, K thẳng hàng.


5
Ta lại có RKA  SKC  RBA  SBC  2ADC  AOC nên AKC  AOC  180
hay tứ giác AOCK nội tiếp. Suy ra

2 2 2 R2
EO  EK  EA  EC  R  OE  EO( EO  EK )  R hay OK  .
EO
Mặt khác, theo bất đẳng thức cơ bản về đường xiên và đường vuông góc thì

1 1  1
EO  max d(O , AC ), d(O , BC )  max  4 R 2  AC 2 , 4 R2  BD 2   4R2  m2 .
 2 2  2

2R2
Vậy ta được OK  . Đây chính là đpcm.
4 R2  m 2

Nhận xét.

Lời giải của bài toán dựa trên hai định lý rất cơ bản của hình học là điểm Miquel của tứ
giác toàn phần và định lý Brocard.

Nói đến định lý Miquel thì nhiều người đã quen thuộc nó ở dạng phát biểu sau

Định lý Miquel. Cho tam giác ABC và các điểm D , E, F lần lượt thuộc các đường thẳng
BC , CA , AB . Khi đó các đường tròn ngoại tiếp các tam giác ( AEF ),( BFD),(CDE) có một
điểm chung M gọi là điểm Miquel.

Hệ quả 1. Các điểm D , E, F thẳng hàng khi và chỉ khi điểm Miquel M thuộc đường
tròn ( ABC ) .

Hệ quả trên rất có ý nghĩa nếu ta coi một tam giác và một đường thẳng là một tứ giác
toàn phần khi đó ta có thể phát biểu lại hệ quả đó như sau

Hệ quả 2. Cho tứ giác ABCD có AB giao CD tại E , AD giao BC tại F . Khi đó các
đường tròn ( EAD),( EBC ),( FAB),( FCD) có một điểm chung M. Điểm M gọi là điểm
Miquel của tứ giác ABCD .

Điểm Miquel M đặc biệt có rất nhiều tính chất thú vị khi tứ giác ABCD nội tiếp. Một
trong những tính chất quan trọng là như sau

Hệ quả 3. Cho tứ giác ABCD nội tiếp đường tròn (O) . Giả sử AB giao CD tại E , AD
giao BC tại F , AC giao BD tại G . Khi đó các đường tròn ( EAD),( EBC ),( FAB),( FCD)
có một điểm chung M và O , G , M thẳng hàng.

Định lý Brocard cũng là một định lý đã khá quen thuộc.


6
Định lý Brocard. Cho tứ giác ABCD nội tiếp đường tròn (O) . Giả sử AB giao CD tại E ,
AD giao BC tại F , AC giao BD tại G . Khi đó O là trực tâm tam giác EFG .

Một trong những bài toán ứng dụng hay của định lý này chính là bài thi quốc gia Việt
Nam năm 2012 vừa qua.

Chứng minh của định lý Miquel và các hệ quả có thể coi là một trong những thể hiện
quan trọng nhất của việc phải sử dụng góc định hướng trong hình học. Ở bài này, nếu
không dùng độ dài đại số cũng như góc định hướng, ta phải quy ước vị trí các điểm
như đã trình bày để lời giải được chặt chẽ.

Bài thi là một trong những ý tưởng hay cho việc kết hợp định lý Miquel và định lý
Brocard đặc biệt là câu 2). Ý tưởng của câu này có lẽ bắt nguồn từ đẳng thức
 
OK  OE  OR  OS  R 2 ; tuy nhiên, để giấu đi sự hiện diện điểm E thì bài thi đã đưa về
một bất đẳng thức liên hệ giữa đường xiên và hình chiếu, đó là

OE  max d(O , AC ), d(O , BD)  R 


2

min AC 2 , BD 2 .
4
Thực sự ý tưởng muốn giấu đi điểm E khá hay xong việc phải dùng đến một bất đẳng
thức hình học đã khiến cho bài toán mất đi khá nhiều vẻ đẹp của nó.

Bài toán này cũng có thể giải bằng cách sử dụng bổ đề sau liên quan đến phép biến
hình như sau:

Cho hai đoạn thẳng AB, CD sao cho ABCD không phải là hình thang. Khi đó, tồn tại
một phép vị tự quay tâm O biến AB thành CD. Nếu P là giao điểm của AB và CD ,
Q là giao điểm của AD và BC thì các tứ giác ADPK , BCPK , ABQK , CDQK nội tiếp.

Trong bài toán đã cho, điểm K cũng chính là tâm của phép vị tự quay đó.

Ý tưởng sử dụng bất đẳng thức ở đây khá giống với bài hình số 5 trong đề thi TST 2006,
cũng là sự so sánh giữa đường xiên và đường vuông góc. Tuy nhiên, ở đây ta không
cần chỉ ra với trường hợp nào thì đẳng thức xảy ra.

Nếu tổng quát lên, thay đường phân giác góc E thành đường thẳng bất kì qua E , ta có
bài toán sau:

Cho tứ giác ABCD lồi có các cạnh đối không song song nội tiếp đường tròn (O , R) .
Gọi E là giao điểm hai đường chéo và một đường thẳng bất kì đi qua E cắt các đường

7
thẳng AB, BC , CD , DA lần lượt tại các điểm M , N , P , Q . Chứng minh rằng giao điểm
khác M , N , P , Q của các cặp đường tròn ( AQM ),( BMN ) ; ( BMN ),(CNP) ; (CNP ),( DPQ)
và ( DPQ),( AQM ) cùng thuộc một đường tròn qua điểm Miquel K của tứ giác ABCD.

Q
A
B
M
E

O C S
D
P

Trong trường hợp đường thẳng bất kì trên trở thành phân giác thì bốn giao điểm trên
trùng nhau và trùng với K.

Bài 2.

1. Chứng minh rằng tồn tại vô số số nguyên dương t sao cho 2012t  1 và 2013t  1 đều
là các số chính phương.

2. Xét m, n là các số nguyên dương sao cho mn  1 và ( m  1)n  1 đều là các số chính
phương. Chứng minh rằng n chia hết cho 8(2m  1) .

Lời giải.
8
1. Đặt d  (2012t  1, 2013t  1) thì dễ thấy d  1 . Do đó, 2012t  1 và 2013t  1 đều là các
số chính phương khi và chỉ khi (2012t  1)(2013t  1)  y 2 với y là số nguyên dương nào
đó. Ta biến đổi đẳng thức trên

(2012t  1)(2013t  1)  y 2
 4  2012 2  20132 t 2  4  2012  2013  4025t  4  2012  2013  4  2012  2013  y 2
 (2  2012  2013t  4025) 2  1  4  2012  2013  y 2

Đặt x  2  2012  2013t  4025 thì ta có phương trình x 2  4  2012  2013 y 2  1 .

Dễ thấy 4  2012  2013 không phải là số chính phương nên phương trình Pell loại 1 này
có vô số nghiệm. Nghiệm nhỏ nhất của phương trình này là ( x , y)  (4025,1) nên các
nghiệm của nó được cho bởi công thức

x0  1, x1  4025, xn 2  8050 xn1  xn


 ,n  0 .
y
 0  1, y 1
 1, y n 2
 8050 y n1
 y n

Bằng quy nạp, ta chứng minh được x2 i 1 chia 2  2012  2013 dư 4025 với mọi i và mỗi
x2 i 1  4025
giá trị nguyên dương sẽ cho ta một giá trị t thỏa mãn đề bài.
2  2012  2013
Vậy tồn tại vô số giá trị nguyên dương t sao cho 2012t  1 và 2013t  1 đều là các số
chính phương. Ta có đpcm.

2. Đặt d  ( mn  1, mn  n  1) thì

d  mn  n  1  mn  1 hay d n , suy ra d (mn  1  mn) hay d 1 .

Do đó d  1 hay các số mn  1,( m  1)n  1 nguyên tố cùng nhau.

Khi đó, mn  1 và ( m  1)n  1 đều là các số chính phương khi và chỉ khi

( mn  1)  (m  1)n  1 là số chính phương.

Giả sử ( mn  1)  (m  1)n  1  y 2 với y    . Biến đổi biểu thức này, ta thu được

m( m  1)n2  (2 m  1)n  1  y 2
 4 m2 ( m  1)2 n2  4m( m  1)(2m  1)n  4m(m  1)  4m( m  1) y 2
2
  2m(m  1)n  (2 m  1)   1  4m( m  1)y 2

9
Đặt x  2 m( m  1)n  (2 m  1) thì ta có phương trình sau

x 2  4m(m  1) y 2  1 (*)

Đây chính là phương trình Pell loại 1 và do 4m( m  1) không là số chính phương với
mọi m nguyên dương nên (*) có vô số nghiệm.

Phương trình (*) có nghiệm nhỏ nhất là ( x , y)  (2 m  1,1) nên công thức nghiệm ( xi , yi )
của nó có thể được viết dưới dạng

x0  1, x1  2 m  1, xi  2  2(2m  1)xi 1  xi


 ,i  0 .
y
 0  0, y1
 1, y i 2
 2(2 m  1) y i 1
 y i

Bằng quy nạp, ta sẽ chứng minh rằng x2i chia 2m( m  1) dư 1 và x2 i 1 chia 2m( m  1) dư
2m  1 với mọi i  0,1, 2,... (**)

Thật vậy,

- Với i  0, theo công thức truy hồi của dãy ( xi ) thì ta thấy khẳng định (**) đúng.

- Giả sử (**) đúng đến i , tức là x2i chia 2m( m  1) dư 1 và x2 i 1 chia 2m( m  1) dư 2m  1 .
Ta có

x2 i  2  2(2 m  1)x2 i 1  x2 i  2(2m  1)(2 m  1)  1  8 m(m  1)  1  1 (mod 2m( m  1)) và

x2 i  3  2(2m  1)x2 i  2  x2 i 1  2(2 m  1)  (2 m  1)  2 m  1 (mod 2m(m  1)) .

Do đó, (**) cũng đúng với i  1 .

Theo nguyên lí quy nạp, (**) đúng với mọi số tự nhiên i.

Tiếp theo, ta sẽ xây dựng công thức truy hồi cho dãy ri  x2 i 1 với i  0,1, 2,...

Ta có

ri  2  x2 i  5  2(2m  1)x2 i  4  x2 i  3  2(2m  1)  2(2m  1)x2 i  3  x2 i  2   x2 i  3


   
 4(2m  1)2  1 x2 i  3  2(2 m  1)x2 i  2  4(2m  1) 2  1 x2 i  3  ( x2 i  3  x2 i 1 )
  4(2m  1) 2
 2 x 2 i3  
 x2 i 1  4(2 m  1)2  2 ri 1  ri

Đặt ri  2 m( m  1)si  (2m  1) thì dãy ( si ), i  0 nguyên dương và xác định duy nhất.

Thay vào công thức truy hồi của (ri ), ta được

10
 
2m(m  1)si  2  (2 m  1)  4(2 m  1)2  2  2m(m  1)si 1  (2 m  1)    2m( m  1)si  (2 m  1) 

  
 2 m(m  1)si  2  2 m( m  1) 4(2 m  1)  2 si 1  2 m( m  1)si  4(2 m  1) (2m  1) 2  1
2

 
 si  2  4(2 m  1)2  2 si 1  si  8(2m  1)

Ta tính được r0  x1  2m  1 nên s0  0 và

 
r1  x3  2(2m  1) 2(2m  1)2  1  (2m  1)  16 m( m  1)(2 m  1)  2 m  1 nên s1  8(2 m  1) .

s0  0, s1  8(2 m  1),


Ta có công thức truy hồi của dãy số ( si ) là 
 2

si  2  4(2 m  1)  2 si 1  si  8(2 m  1), i  0

Từ đó suy ra dãy số ( si ) có các số hạng chia hết cho 8(2m  1) với mọi i  0,1, 2,...

Hơn nữa, với cách đặt x  2 m( m  1)n  (2 m  1) thì dễ dàng thấy rằng n thỏa mãn đề bài
khi và chỉ khi n  si , i  1, 2, 3,... (do s0  0 không phải là số nguyên dương).

Vậy tất cả các giá trị n đều chia hết cho 8(2m  1). Đây chính là đpcm.

Nhận xét.

Ý thứ 1 của bài toán thực ra là một trường hợp đặc biệt và cũng là một sự dẫn dắt cho ý
thứ 2. Ta có thể giải ý 1 theo cách ít tính toán hơn bằng nhận xét:

Nếu x , y  1 là nghiệm của phương trình 2013x 2  2012 y 2  1 thì t  y 2  x 2   * và các


số 2012t  1, 2013t  1 đều chính phương.

Hơn nữa, mỗi nghiệm ( x , y) của phương trình x 2  2012  2013 y 2  1 cho ta một nghiệm
( x  2012 y , x  2013 y ) của phương trình 2013x 2  2012 y 2  1 nên dễ thấy đpcm.

Cách tiếp cận bằng phương trình Pell hoàn toàn tự nhiên và khi đã xây dựng thành
công thì ta chỉ còn cần thao tác trên các dãy số nguyên. Trong những năm gần đây, các
bài toán về dãy số nguyên khá được ưa chuộng, đặc biệt dãy số nguyên liên quan đến
công thức nghiệm của phương trình nghiệm nguyên. Chẳng hạn như

(Việt Nam TST 2012) Cho dãy số nguyên dương ( xn ) được xác định bởi

x1  1, x2  2011,

xn  2  4022 xn 1  xn , n  1, 2, 3,...
11
x2012  1
Chứng minh rằng là số chính phương.
2012
Trong việc giải quyết ý thứ 2, nếu ta lập luận như sau:

Do mn  1 và ( m  1)n  1 đều là các số chính phương nên ta có thể đặt

mn  1  a 2 ,( m  1)n  1  b2 .

Từ đó ta có phương trình ( m  1)a 2  mb 2  1 . Đổi biến a  x  my , b  x  ( m  1) y để đưa


về phương trình Pell liên kết là x 2  m( m  1)y 2  1 .

Cách giải này cũng hoàn toàn phù hợp nhưng chú ý rằng phương trình Pell tổng quát
dạng Ax 2  By 2  n nói chung có nhiều hơn một nghiệm cơ sở. Ta cần chứng minh
trong trường hợp đặc biệt này ( A  B  1 và n  1 ) thì nó chỉ có duy nhất một nghiệm cơ
sở. Nếu không thì lời giải cũng thực sự vẫn còn thiếu sót.

Ngoài cách sử dụng phương trình Pell như trên, ta có thể lập luận như sau:

Do ( mm  1, mn  n  1)  1 nên ta cần có ( mn  1)( mn  n  1) là số chính phương hay tồn


tại a   sao cho 4m(m  1)n2  4(2m  1)n  4  a 2 , đưa về
2
(2m  1)n  2   n2  a 2

Ta chứng minh được rằng n phải là số chẵn và đây là phương trình Pythagore nên phải
tồn tại các số p , q , k sao cho

n  2 kpq ,
 2 2
(2 m  1)n  2  k( p  q )

trong đó k (n,(2m  1)n  2)  2 nên k  1 hoặc k  2.

Nếu k  1 thì dễ thấy p 2  q 2 chẵn, suy ra p , q cùng tính chẵn lẻ. Nếu p , q cùng lẻ thì n
chia 4 dư 2 và (2 m  1)n  2 chia hết cho 4 trong khi p 2  q 2 chia 4 dư 2, mâu thuẫn. Nếu
p , q cùng chẵn thì n chia hết cho 4 dẫn đến (2 m  1)n  2 chia 4 dư 2 trong khi p 2  q 2
chia hết cho 4, cũng mâu thuẫn.

Nếu k  2 thì n  4 pq và (2 m  1)n  2  2( p 2  q 2 ) , suy ra

p 2  q 2  2(2 m  1) pq  1  0 .

12
Đến đây ta chứng minh được (sử dụng phương pháp Viète Jumping) p , q là hai số hạng
liên tiếp của dãy số cho bởi công thức

x1  1, x2  2(2 m  1),


 .
x
 i 2  2(2 m  1) x i 1
 x i
, i  1

Từ đó suy ra pq chia hết cho 2(2m  1) hay n chia hết cho 8(2m  1) .

Phương pháp xây dựng nghiệm ở đây có nét giống với bài 6 đề VMO 2012.

Xét các số tự nhiên lẻ a , b mà a là ước số của b2  2 và b là ước số của a2  2 . Chứng


minh rằng a , b là các số hạng của dãy số tự nhiên ( vn ) xác định bởi

v1  v2  1 và vn  4 vn 1  vn 2 với mọi n  2 .

Cách giải này có phần ngắn gọn hơn và xử lí nhẹ nhàng hơn cách đã nêu ở lời giải ban
đầu nhưng để trình bày chặt chẽ không phải là điều đơn giản.

Bài 3.

Với mỗi số n nguyên dương, đặt Sn  0,1, 2,..., 2n  1 . Xét hàm số f : (  Sn )  [0;1]
thỏa mãn đồng thời các điều kiện sau:

i/ f ( x ,0)  f ( x , 2n  1)  0 với mọi số nguyên x.

ii/ f ( x  1, y)  f ( x  1, y )  f ( x , y  1)  f ( x , y  1)  1 với x , y   và 1  y  2n .

Gọi F là tập hợp tất cả các hàm số f thỏa mãn.

1. Chứng minh rằng F là vô hạn.

2. Với mỗi hàm số f  F , đặt v f là tập hợp ảnh của f . Chứng minh rằng v f hữu hạn.

3. Tìm giá trị lớn nhất của v f với f  F .

Lời giải.

1. Trong đẳng thức ii/ đã cho, ta thấy rằng


( x  1)  y  ( x  1)  y  x  ( y  1)  x  ( y  1) (mod 2)

13
Điều này có nghĩa là các giá trị của hàm f ( x , y) với x , y cùng tính chẵn lẻ và x , y khác
tính chẵn lẻ là không có liên hệ với nhau.

Ta sẽ tìm các xác định hàm f trong cả hai trường hợp.

Trong mặt phẳng tọa độ Oxy , ta xét một lưới nguyên nằm ngang có chiều cao là 2n  1 ,
chiều rộng vô hạn và điểm có tọa độ (i , j ) sẽ được gán giá trị f (i , j ) .

Điều kiện i/ được hiểu là tất cả các số thuộc hai biên (trên và dưới) của lưới đều gán số
0; còn các điểm nguyên bên trong lưới đều được gán giá trị thuộc [0; 1] .

Điều kiện ii/ chính là với mọi hình vuông con nằm nghiêng (các đường chéo song song
với các trục tọa độ) có các đỉnh nguyên và cạnh là 2 đều có tổng các số gán cho các
đỉnh bằng 1.

Với mỗi điểm có tọa độ A nguyên thuộc lưới nguyên đang xét, ta đặt f1 ( A), f 2 ( A) là giá
trị gán cho các điểm có cùng tung độ với A , lần lượt có hoành độ lớn hơn và nhỏ hơn
hoành độ của A đúng 2 đơn vị.

Tiếp tục đặt ak  f ( k , k ) với k  1, 2, 3,..., 2n là giá trị gán cho điểm Ak ( k , k ) . Theo điều
kiện xác định hàm số thì

a1  a2  f1 ( A1 )  0  1  f1 ( A1 )  1  a1  a2 ,
a2  a3  f1 ( A2 )  f1 ( A1 )  1  f1 ( A2 )  a1  a3 ,
...
a2 n1  a2 n  f1 ( A2 n 2 )  f1 ( A2 n1 )  1  f1 ( A2 n1 )  a1  a2 n ,
a2 n  0  f1 ( A2 n1 )  f1 ( A2 n )  1  f1 ( A2 n )  a1

Tương tự, ta cũng có

f2 ( A2 n )  1  a2 n1  a2 n
f 2 ( A2 n  1 )  a 2 n  a 2 n  2
...
f2 ( A2 n1 )  1  a1  a2 n ,
f2 ( A2 n )  a2 n

Từ đây ta thấy rằng nếu dãy ak xác định thì các giá trị f1 ( Ak ), f2 ( Ak ) cũng hoàn toàn
xác định. Ta sẽ chọn các giá trị ak sao cho f1 ( Ak ), f2 ( Ak ) đều thuộc [0; 1] .

Ta chọn a1  a3  a5  ...a2 n1 , a2  a4  a6  ...  a2 n , a1  a2 n  1 thì f1 ( Ak ), f2 ( Ak )  [0; 1] .


14
Hơn nữa, ta cũng thấy rằng

f1 ( A1 )  f1 ( A3 )  f1 ( A5 )  ...  f1 ( A2 n1 ), f1 ( A2 )  f1 ( A4 )  f1 ( A6 )  ...  f1 ( A2 n ), f1 ( A1 )  f1 ( A2 n )  1,


f2 ( A1 )  f2 ( A3 )  f2 ( A5 )  ...  f2 ( A2 n1 ), f2 ( A2 )  f 2 ( A4 )  f2 ( A6 )  ...  f 2 ( A2 n ), f 2 ( A1 )  f2 ( A2 n )  1
Do đó, tính đơn điệu của hai dãy con có chỉ số chẵn và lẻ vẫn được bảo toàn.

Tương tự, từ dãy f1 ( Ak ) , ta xác định được f1 ( f1 ( Ak )) thỏa mãn, từ dãy f 2 ( Ak ), ta cũng
xác định được f2 ( f2 ( Ak )) và cứ thế, tức là xây dựng được tất cả giá trị cho hàm số
f ( x , y) trong trường hợp x  y chẵn.

Tiếp tục đặt bk  f ( k  1, k ) với k  1, 2, 3,..., 2n là giá trị gán cho điểm Bk ( k  1, k ) thì
hoàn toàn tương tự như trên, ta xây dựng được tất cả các giá trị của f ( x , y) với x  y lẻ.

Dễ thấy có vô số cách chọn dãy ( ak ),(bk ) thỏa mãn các điều kiện trên nên có vô số hàm
f thỏa mãn đề bài, tức là F vô hạn. Ta có đpcm.

2. Trong đẳng thức f ( x  1, y)  f ( x  1, y )  f ( x , y  1)  f ( x , y  1)  1 , ta thay x , y lần lượt


bởi x  1, y  1 , ta được f ( x , y  1)  f ( x  2, y  1)  f ( x  1, y)  f ( x  1, y  2)  1 .

Từ đó suy ra f ( x  1, y)  f ( x , y  1)  f ( x  1, y  2)  f ( x  2, y  1) (*) hay

f (1,1)  f (2,0)  f (3, 3)  f (4, 2)  ...  f (2 n  1, 2 n  1)  f (2 n  2, 2 n) ,

f (3,1)  f (4,0)  f (5, 3)  f (6, 2)  ...  f (2 n  3, 2 n  1)  f (2 n  4, 2 n) .

Do đó f (1,1)  f (2n  2, 2n), f (3,1)  f (2n  4, 2n) .

15
Tương tự, ta có được f (2, 2)  f (2n  2, 2n  1), f (4, 2)  f (2n  5, 2n  1) .

Tiếp tục áp dụng nhiều lần đẳng thức (*), suy ra

f ( k , k )  f (2n  1  k ,2n  1  k ), f ( k  2, k )  f (2n  3  k , 2n  1  k ) .

Chứng minh tương tự, ta có

f (2n  k , 2n  1  k )  f (4n  2  k , k ), f (2n  3  k , 2n  1  k )  f (4n  4  k , k ) .

Do đó, f ( k , k )  f (2n  1  k , 2n  1  k )  f (4n  2  k , k ) với mọi k  1, 2, 3,..., 2n và như thế,


bằng quy nạp, ta có được

f ( k , k )  f ((2n  1)i  k , 2n  1  k )  f ((4n  2)i  k , k ) với i   và k  1, 2, 3,..., 2n .

Hơn nữa, sự xác định các giá trị trong đường chéo tiếp theo (nằm về phía phải) hoàn
toàn giống nhau ở dải các điểm ( k , k ) và ((2n  1)i  k , 2n  1  k ) nên các giá trị trên đó
cũng tương ứng bằng nhau. Suy ra giá trị của f có sự tuần hoàn và được minh họa như
hình bên dưới, tức là các giá trị của f ( x , y) với x  y chẵn là sự lặp lại các giá trị được
gán cho các điểm nguyên trong tam giác  có tọa độ các đỉnh (1,1),(2n  1, 2n),(4n  1,1) .

2n+1
2n

O 1 2n+1 2n+2 4n+1 4n+2 4n+3 4n+3 6n+3

Tuy nhiên, các giá trị này là hữu hạn và ta tính được tổng cộng có không quá

1  2  3  ...  2n  n(2n  1) giá trị.

Tương tự với giá trị của f ( x , y) mà x  y lẻ, ta cũng có thêm không quá n(2n  1) giá trị
nữa. Kết hợp với 0, ta có v f  2n(2n  1)  1 .

Do đó, v f hữu hạn với mọi f  F , ta có đpcm.

16
3. Ta sẽ xây dựng một hàm số f có v f  2n(2n  1)  1 và chỉ ra đây chính là giá trị lớn

nhất của v f .

Bằng quy nạp, ta chứng minh được rằng

f (i  2 k , i) 
1  (1) 1  (1)   (1) a
i k
k
 ( 1) k  i ak
ki
4
với mọi i  1, 2,..., 2n và k   mà 0  k  i  2n  1 . (**)

Thật vậy, với k  0 thì (**) hiển nhiên đúng.

Giả sử (**) đúng với mọi (i , k ) mà k  m và k  m  1, i  j . Từ điều kiện ii/, ta có

f ( j  1  2( m  1), j  1)  f ( j  1  2m , j  1)  f ( j  2( m  1), j )  f ( j  2  2 m , j  2)  1 .

Sử dụng giả thiết quy nạp, ta tính được

f ( j  1  2( m  1), j  1) 
 1  (1)  1  (1)   (1)
j 1 m 1
m 1
am  j  2  (1) m  j  2 am 1 .
4
Suy ra (**) cũng đúng với i  j  1, k  m  1 .

Như thế, các giá trị được gán cho các điểm trong tam giác  đều có dạng  ij  ai  a j với

 ij  0,1 , trong đó giá trị  ij cũng như dấu của ai , a j xác định duy nhất theo i , j .

1 1
Tiếp theo, ta chọn a2 k 1  2 k 1
, a2 k  2( n  1 k )
với k  1, 2,..., n thì do trong hệ cơ số 3, một
3 3
r
i
số nguyên bất kì có duy nhất một cách biểu diễn dưới dạng tổng  3
i 0
i
với  r  0 và

ei  1,0,1 ) nên suy ra tất cả các giá trị của f ( x , y) dùng để đánh số cho các điểm nằm
trong tam giác  là đôi một phân biệt và khác 0, tức là có đúng n(2n  1) giá trị như thế,
tức là ta đã xây dựng được giá trị cho hàm số f ( x , y) với x  y chẵn.

Tương tự, để xây dựng cho f ( x , y) với x  y lẻ, ta chọn dãy số

1 1
b2 k 1  , b2 k  với k  1, 2,..., 2n .
3 2 k 1 3 32( n1 k ) 3

thì các giá trị dạng  ij  bi  bj cũng đôi một phân biệt và khác với các giá trị  ij  ai  a j .

17
Từ đó, ta xây dựng được toàn bộ giá trị cho hàm số f ( x , y) và có tất cả 2n(2n  1)  1 giá
trị đôi một khác nhau.

Vậy giá trị lớn nhất cần tìm của v f trên miền F là 2n(2n  1)  1 .

Nhận xét.

Đây là một bài toán đòi hỏi những kiến thức tổng hợp và một phong cách làm việc từ
tốn, bài bản. Mô hình lưới hoặc bảng là một mô hình tự nhiên mà ta nghĩ đến, và hàm
số có thể được xác định theo hàng, cột hoặc đường chéo.

Trong các bài toán có chứa tham số n , ta nên bắt đầu từ những giá trị n nhỏ để hình
dung bài toán một cách tốt nhất, cụ thể nhất, từ đó tìm cách tiếp cận tổng quát. Nếu
chọn cách này thì có lẽ câu 1) và 2), thậm chí cả phần chặn trên đúng ở câu 3) không
phải là quá khó. Điểm khó ở câu 1) là cách xây dựng hàm phải thỏa mãn điều kiện
f ( x , y)  [0; 1] . Để có được điều này, ta phải tìm được một điều kiện có tính bất biến đối
với dãy giá trị trên đường chéo (không chỉ đề dãy kề nó thuộc [0; 1] , mà còn để các dãy
tiếp theo cũng như thế).

Để làm câu 3), ta phải có những nhận xét tinh tế hơn, đòi hỏi nhiều thời gian hơn. Có lẽ
câu này chỉ dành cho những bạn đã giải quyết tốt hai bài 1, 2 trong vòng 1,5 - 2 giờ, còn
khá nhiều thời gian để tập trung toàn lực cho bài này. Ý tưởng cơ bản ở đây là dự đoán
và chứng minh được công thức tổng quát của hàm dựa theo các phần tử thuộc một
đường chéo. Từ công thức tổng quát này ta mới tìm cách chọn các “giá trị khởi tạo” để
tất cả các giá trị nằm trong “tam giác tuần hoàn” đôi một khác nhau.

Để chọn các giá trị khác nhau, ta có một số định hướng cơ bản sau:

+ Dùng bất đẳng thức

+ Dùng tính “độc lập tuyến tính” của các số vô tỷ, cụ thể là: Nếu p1 , p2 ,..., pn là các số
nguyên tố phân biệt thì không tồn tại các số hữu tỉ c1 , c 2 ,..., c n không đồng thời bằng 0
sao cho c1 p1  c 2 p2  ...  cn pn  0 .

Tuy nhiên, việc chứng minh bổ đề này là không đơn giản. Ta cũng có thể thay các số
pi này bằng các số vô tỷ khác dựa vào tính không đếm được của  , nhưng đây cũng
là kiến thức nằm ngoài khuôn khổ chương trình phổ thông.
18
Một cách tiếp cận thú vị và sơ cấp cho vấn đề này đã được trình bày ở trên là sử dụng
tính chất của hệ đếm cơ số 3: Mọi số nguyên N  0 bất kỳ đều có thể biểu diễn duy nhất
dưới dạng

N   i 3i   i 1 3i 1  ...   0 trong đó  i  1,  j  1; 0;1 , j  0,1,.., i  1.

Mệnh đề này có thể chứng minh khá dễ dàng bằng quy nạp toán học.

Việc f ( x , y)  [0; 1] không thành vấn đề vì ta có thể chia các giá trị khởi tạo cho một
hằng số đủ lớn để đạt được điều này.

Ngoài cách xây dựng theo đường chéo với 4n giá trị khởi tạo như trên ( 2n cho trường
hợp f ( x , y) mà x  y chẵn và 2n cho trường hợp f ( x , y) mà x  y lẻ), ta cũng có thể xây
dựng theo hai cạnh dọc hoặc theo hai cạnh ngang có độ dài 2n .

Chẳng hạn, trong trường hợp xây dựng theo cạnh dọc, song song với trục Oy , ta có thể
làm như sau:

Bằng quy nạp, ta chứng minh được rằng

f ( x , 2i )  i   f ( x  2i  1,1)  f ( x  2i  3,1)  ...  f ( x  2i  1,1) 


f ( x , 2i  1)   f ( x  2i ,1)  f ( x  2i  2,1)  ...  f ( x ,1)  ...  f ( x  2i  2,1)  f ( x  2i ,1)   i

Ta cần chọn f ( x ,1), x  [2n  1; 2n] sao cho:

i/ i  1  f ( x  2i  1,1)  f ( x  2i  3,1)  ...  f ( x  2i  1,1)  i .

ii/ i  f ( x  2i ,1)  f ( x  2i  2,1)  ...  f ( x ,1)  ...  f ( x  2i  2,1)  f ( x  2i ,1)  i  1 .

iii/ Các giá trị f ( x ,1) là phân biệt với x  [2n  1; 2n] .

iv/ f ( x , 2i  1)  f ( x  2i ,1)  f ( x  2i  2,1)  ...  f ( x ,1)  ...  f ( x  2i  2,1)  f ( x  2i ,1)  i


với i  n thì luôn bằng n.

Từ đó đưa lên đường tròn và kết hợp với bất đẳng thức để chọn, tức là xét một số giá trị
đủ nhỏ thích hợp để cho chúng đôi một phân biệt. Cách xây dựng cho trường hợp cạnh
ngang cũng thực hiện tương tự.

Bài này nhắc ta nhớ đến một phương trình hàm tương tự xuất hiện trong kì thi chọn đội
tuyển cách đây 10 năm:

19
(Việt Nam TST 2003) Cho hàm số f :     thỏa mãn đồng thời các điều kiện sau:
i) f (0,0)  52003 , f (0, n)  0 với mọi n là số nguyên khác 0.
 f (m  1, n)   f ( m  1, n  1)   f (m  1, n  1) 
ii) f ( m , n)  f ( m  1, n)  2    
 2   2   2 
với mọi số nguyên dương m và mọi số nguyên n.
Chứng minh rằng tồn tại số nguyên dương M sao cho f ( M , n)  1 với mọi số nguyên n
52003  1 52003  1
thỏa n  và f ( M , n)  0 với mọi số nguyên n thỏa n  .
2 2
Tuy nhiên bài toán này lại là một câu chuyện khác !

Bài 4.

Tìm hằng số k nguyên dương lớn nhất thỏa mãn: Với mọi a , b , c dương mà abc  1 thì ta
có bất đẳng thức sau

1 1 1 k k
     3 (*)
a b c abc1 4

Lời giải.

Giả sử k là số nguyên dương sao cho bất đẳng thức đã cho đúng với mọi a , b , c mà
2 9
abc  1. Thay b  c  , a  vào (*), ta được
3 4
3 4 k k 880
2    3 k   14 .
2 9 2 2 9 4 63
  1
3 3 4
Hơn nữa, vì k là số nguyên dương nên từ đánh giá trên, ta có k  13. Ta sẽ chứng minh
rằng với k  13 thì bất đẳng thức (*) đúng.

1 1 1 13 25
Thật vậy, với k  13 ta có bất đẳng thức     . (**)
a b c a bc 1 4

1 1 1 1
Đặt f ( a , b, c )     . Không mất tính tổng quát, ta có thể giả sử
a b c abc 1
a  max a , b , c , khi đó

20
1 1 2   1 1 
 
f ( a , b , c )  f a , bc , bc    
b c bc 
  13   
 a  b  c  1 a  2 bc  1 
 
2 1 13

 b c   .

 bc (a  b  c  1) a  2 bc  1 
  
1
Do a  max a , b , c và giả thiết abc  1 nên ta có bc  1, suy ra  1.
bc
Mặt khác, sử dụng bất đẳng thức AM-GM và biến đổi cho biểu thức trong ngoặc ở đẳng
thức trên, ta có
13 13 13
   1,

( a  b  c  1) a  2 bc  1   3 abc  1 3 abc  1
3 3 16

nên hiển nhiên f ( a , b , c )  f  a , bc , bc  . Ta đưa được bài toán về chứng minh

 1  25
f  2 , x, x   với x  bc , 0  x  1.
x  4
Nếu x  1 thì bất đẳng thức trên trở thành đẳng thức. Trong trường hợp 0  x  1, bằng
cách sử dụng lại biến đổi đã thực hiện trong quá trình tìm điều kiện cần cho k , ta thấy
bất đẳng thức tương đương với
( x  2)(2 x 3  x 2  1) 13
  4( x  2)(2 x 3  x 2  1)  13x(2 x  1)
x(2 x  1) 4
 4(2 x 4  5x 3  2 x 2  x  2)  26 x 2  13 x  8 x 4  20 x 3  18 x 2  9 x  8  0.
Ta có
8 x 4  20 x 3  18 x 2  9 x  8  (8 x4  8 x 2  2)  (20 x 3  20 x 2  5 x)  (10 x 2  14 x  6)
 2(2 x 2  1)2  5 x(2 x  1) 2  2(5x 2  7 x  3)  0
do 2(2 x 2  1)2  0, 5x(2 x  1) 2  0 và 5x 2  7 x  3  0 (tam thức bậc hai có hệ số cao nhất
dương và biệt thức   11  0 ). Như vậy, bất đẳng thức cuối hiển nhiên đúng. Ta đi
đến kết luận cuối cùng k  13 là giá trị cần tìm.

Nhận xét.

Bài toán này có dạng phát biểu khá giống với các bài toán tìm hằng số tốt thông thường
nên rất tự nhiên, ta nghĩ đến lối tiếp cận tương tự như các dạng toán này: Tìm một bộ số
thích hợp thay vào để dự đoán điều kiện cần cho k rồi sau đó đi chứng minh đó cũng
là điều kiện đủ.
21
Do bất đẳng thức có dạng đối xứng nên ta nghĩ đến việc chọn một bộ số mà ở đó có hai
1
biến nhận giá trị bằng nhau. Điều này lý giải cho việc chọn a  b  x và c  như
x2
trong lời giải ở trên. Một cách khác để chứng minh bất đẳng thức (**) dùng phương
pháp dồn biến như sau:
25
Biến đổi (**) về dạng đa thức ( ab  bc  ca)(a  b  c  1)  13  (a  b  c) .
4
Giả sử a  max a , b , c thì a  1 và đặt x  b  c thì bất đẳng thức trên có thể viết lại
25
thành ( ax  bc )( a  x  1)  13  ( a  x) . Bất đẳng thức này tương đương với
4
25 25a
ax 2  (bc  a 2  a  )x  14  0
4 4
2 2
 2 25   bc  a 2  a  25 
 bc  a   4 
 a x  4     14 
25a
0
 2a  2a 4
 
2
Do x  b  c  2 bc  nên
a
25 2a 2  bc  a 2  25 1 25
bc  a 2  4 a   a  a2 
4  a 4 a 4 0
x 
2a 2a 2a
2 2
 1 2 25   1  a 2  a  25 
4 a   a  a  
 a 4    a 4  25a
Ta đưa về chứng minh a   14  0.
 2 a  2 a 4
 
2 2
 1 2 4 25   1  t 4  t 2  25 
 4t  t 2  t  t  4   t 2 4  25t 2
Đặt a  t  0 thì ta đưa về t 2  2   2
 14  0
 2 t  2 t 4
 
1
 
hay (t  1)2 8t 4  9t 3  18t 2  20t  8  0 . Đặt t 
u
thì ta đưa được về bất đẳng thức đã

đề cập trong cách ban đầu.

1 1 1 13 25
Ta cũng có thể chứng minh bất đẳng thức     không dùng đến
a b c abc 1 4
dồn biến như sau:
22
x2 y2 z2
Do abc  1 nên tồn tại các số thực dương x , y , z sao cho a  ,b  ,c  , ta cần
yz zx xy
chứng minh rằng
yz zx xy 13 xyz 13
2
 2 2  3 3 3
 3 .
x y z x  y  z  xyz 4
Do x , y , z bình đẳng nên ta có thể giả sử x  y  z . Viết bất đẳng thức trên thành dạng
tương đương
x 3 y 3  y 3 z 3  z 3 x 3  3 x2 y 2 z 2 13( x 3  y 3  z 3  3xyz )
 .
x2 y 2 z 2 4( x 3  y 3  z 3  xyz)
Ta có các đánh giá x 3  y 3  z 3  3 xyz  ( x  y  z) ( x  y )2  ( z  x)( z  y ) và

x 3 y 3  y 3 z 3  z 3 x 3  3 x 2 y 2 z 2  ( xy  yz  zx)  z 2 ( x  y )2  xy( z  x)( z  y )


 xy( xy  yz  zx) ( x  y )2  ( z  x)( z  y )
Ta cần chứng minh
xy( xy  yz  zx) ( x  y )2  ( z  x)( z  y)  13( x  y  z ) ( x  y) 2  ( z  x)( z  y) 
 hay
xyz 2 4( x 3  y 3  z 3  xyz )
4( xy  yz  zx)( x 3  y 3  z 3  xyz)  13 xyz 2 ( x  y  z) .
3
 x y 3 3
Do tính thuần nhất nên ta có thể giả sử x  y  2 thì dẫn đến x  y  2   2.
 2 
Suy ra
( xy  yz  zx)( x 3  y 3  z 3  xyz )  ( xy  2 z)(2  z 3  xyz)  xy(2  z 3 )  2 z(2  z 3  xyz) .
Ta đưa về
4 xy(2  z 3 )  8 z(2  z 3  xyz)  13 xyz 2 ( z  2)  8 z(2  z 3 )  xy(9 z 3  18 z 2  8)
 8 z 4  9 z 3  18 z 2  16 z  8  (1  xy)(9 z 3  18 z 2  8)  0

Chú ý rằng xy  1 và z  1 nên (1  xy)(9 z 3  18 z 2  8)  0 ; tiếp tục đặt z  1  t với t  0


và thay vào 8 z 4  9 z 3  18 z 2  16 z  8  8t 4  23t 3  3t 2  15t  5 với t  0 .
Dễ dàng chứng minh được biểu thức này không âm nên bất đẳng thức cần chứng minh
ở trên là đúng. Từ đó ta có đpcm.
Ta thấy rằng việc xử lí bất đẳng thức (**) không quá khó nhưng vấn đề là tại sao lại
2 9
nghĩ ra cách chọn b  c  , a  để có được k  14 . Điểm tinh tế và cũng là khó nhất
3 4
của bài toán chính là ở đây.
23
Trong bất đẳng thức đã cho, đẳng thức xảy ra khi a  b  c  1 nên ta dự đoán là giảm số
1  1 
biến bằng cách đặt a  1, b  x , c  thì thu được ngay k  4  x   2  , tiếp tục cho
x  x 
x  1 thì có k  16 . Tuy nhiều điều kiện này còn chưa chặt!

1
Ta tiếp tục cho b  c  , a  x 2 thì thay vào bất đẳng thức ban đầu, ta được
x
4(2 x 4  5 x3  2 x 2  x  2) 4( x  2)(2 x 3  x 2  1)
k   f ( x) với x  0 . (***)
2 x2  x 2 x2  x
Khi đó, ta phải có k  min f ( x) .
x 0

Dưới đây ta sẽ phân tích một số cách để từ (***) có thể suy ra k  14.

(1) Ta có f ( x)  0 tương đương với một phương trình bậc 5:

g( x)  4 x 5  8 x 4  5 x 3  4 x  1  0 .

Ta không giải được phương trình này, vì thế không thể tìm được chặn trên chính xác
cho k . Tuy nhiên, do bài toán yêu cầu tìm k nguyên dương lớn nhất, nên ta cũng
không cần đi tìm giá trị min, mà chỉ đánh giá nó nhằm tìm ra một chặn trên cho k .

4 1
Chú ý phương trình g( x)  0 có thể viết dưới dạng 4 x 2  8 x  5  2
 3.
x x
Trên miền (0; ) thì vế trái là hàm tăng, vế phải là hàm giảm nên g( x)  0 có nhiều
nhất 1 nghiệm dương.

 1 7
Chú ý rằng g      0, g(1)  12  0 nên nghiệm x0 của phương trình này nằm giữa
2 4
1
và 1. Ta không có ý định đi tìm x0 mà thông tin này chỉ dùng để có định hướng để
2
4( x  2)(2 x 3  x 2  1)
chọn giá trị x để thay vào bất đẳng thức k.
2 x2  x
1 
Ta thay các giá trị đặc biệt nằm trong  ;1 .
2 

 Thay x  1 , ta được k  16 .
1
 Thay x  , ta được k  15 .
2

24
2
 Thay x  , ta được k  13,98 .
3
3
 Thay x  , ta được k  14, 2.
4
Đến đây, ta chỉ ra được k  14.

(2) Ở cách tiếp cận này, ta sẽ chứng minh rằng k  14 không đúng (do các đánh giá đơn
giản dễ đưa về trường hợp k  14 nhiều hơn).

Với k  14 , ta có bất đẳng thức 4 x 4  10 x 3  10 x 2  5 x  4  0 . Ta sẽ chứng minh rằng bất


đẳng thức này không đúng với mọi x  0. Đặt h( x) là vế trái của nó, ta có

h( x)  16 x 3  30 x 2  20 x  5, h( x)  48 x 2  60 x  20 .

Ta thấy h( x)  0 có nghiệm duy nhất và f (0)  0  f (1) nên khảo sát hàm số h( x), ta
được phương trình h( x)  0 có nghiệm duy nhất x0 thỏa x0  (0;1) và min h( x)  h( x0 ) .
x 0

 3 3 3 
Chú ý rằng f     0  a0   a0   ;1  và nên 16 x03  30 x02  20 x0  5  0 và ta có
4 4 4 

7 f ( x0 )  7(4 x04  10 x03  10 x02  5x0  4)  x0 (16 x03  30 x02  20 x0  5)


 ( x0  1)(12 x03  52 x02  2 x0  28)
3 2
3 2  3 3
Ta có 12 x  52 x  2 x0  28  12    52    30  0 và x0  1  0 nên f ( x0 )  0 , mâu
0 0
4 4
3 
thuẫn (chú ý rằng 12 x 3  52 x 2  2 x  28 đồng biến trên  ; 1  ).
4 

Do đó k  14 không thỏa mãn.

(3) Do f ( x) có dạng khá cồng kềnh nên nếu để nguyên như vậy mà khảo sát thì sẽ khá
phức tạp. Do đó, ta nghĩ đến việc biến đổi f ( x) về dạng đơn giản hơn để dễ tính đạo
hàm và vì f ( x) có dạng phân thức nên ta nghĩ đến việc tách và chia đa thức:

2 3
f ( x)  x 2  2 x   .
x 2x  1
Đến đây thì việc lấy đạo hàm đã trở nên khá dễ dàng, ta tính được

25
 1 3 
f ( x)  2  x  1  2  2 
.
 x (2 x  1) 
Ta cần giải phương trình f ( x)  0. Quan sát một chút, ta phát hiện được :
1 x 3  1 ( x  1)( x 2  x  1) 3 4( x 2  x  1)
x 2   , 1  .
x x2 x2 (2 x  1)2 (2 x  1)2
Như vậy, ta có thể phân tích được nhân tử:
x 1 4 
f ( x)  ( x 2  x  1)  2  2 
,
 x (2 x  1) 
từ đó ta đưa được việc xét f ( x)  0 về xét một phương trình đơn giản hơn:
x 1 4
2
 2
 0  4 x 3  4 x 2  3 x  1  0.
x (2 x  1)
Đến đây, ta gặp phải một khó khăn khá lớn, đó là phương trình 4 x 3  4 x 2  3 x  1  0
không có “nghiệm đẹp”. Khi khảo sát một phương trình, ta quan tâm đến hai việc:
phương trình đó có bao nhiêu nghiệm và đó là những giá trị nào. Việc thứ nhất có thể
dễ dàng thực hiện bằng cách viết lại phương trình dưới dạng:
1
4x2  4x  3   0.
x
Vế trái của phương trình trên là một hàm liên tục và đồng biến với mọi x  0, đồng thời
bằng kiểm tra trực tiếp ta dễ thấy phương trình có ít nhất một nghiệm thuộc (0, 1) nên
bằng cách kết hợp 2 điều này lại, ta có thể khẳng định phương trình f ( x)  0 có nghiệm
duy nhất thuộc (0, 1).

Việc thứ hai thực sự rất khó khăn trong trường hợp của bài toán này. Rõ ràng với việc
chỉ bằng tính tay, ta rất khó tính được giá trị chính xác nghiệm của phương trình
4 x 3  4 x 2  3 x  1  0. Vả lại, nếu tính được thì chắc chắn trong công thức nghiệm sẽ có
căn thức chằng chịt và khi thay vào f ( x), ta sẽ rất khó để đánh giá xấp xỉ sang dạng
thập phân để từ đó đưa ra nhận định về k. Có vẻ như ta đang đi vào ngõ cụt… Tuy
nhiên, các bạn hãy chú ý rằng ta đang cần tìm giá trị nguyên dương lớn nhất của k , do đó
ta hoàn toàn không cần phải tính ra đúng giá trị nhỏ nhất của f ( x) làm gì. Thay vào đó,
ta có thể nghĩ đến việc tìm một giá trị “sát” với giá trị nhỏ nhất đó cũng được. Muốn
vậy, ta cần tìm một giá trị x sát với nghiệm của phương trình f ( x)  0 để thay vào tính
toán. Từ nhu cầu mới này, ta nghĩ đến việc đánh giá chặn miền cho nghiệm của phương
26
trình 4 x 3  4 x 2  3 x  1  0. Gọi x0  (0, 1) là nghiệm của phương trình. Ý tưởng của ta là
làm sao khử được dạng bậc ba để có được một phương trình bậc thấp có thể giải được
nhanh chóng bằng tính tay. Đầu tiên ta có để ý rằng với x0  (0,1) thì 4 x03  4 x02 , do đó

8 x02  3 x0  1  4 x03  4 x02  3 x0  1  0.

3  41 1
Từ đây, ta tìm được chặn dưới cho x0 là x0   Tiếp theo, ta sẽ tìm chặn trên
16 2
cho x0 . Bằng chú ý nhỏ rằng với x0  (0,1) thì các lũy thừa của nó khá nhỏ và “xấp xỉ”
với nhau nên để khử bậc ba, ta mạnh dạn sử dụng bất đẳng thức AM-GM như sau:

0  4 x03  4 x02  3 x0  1  (4 x03  x0 )  4 x02  4 x0  1  4 x02  4 x02  4 x0  1  8 x02  4 x0  1.

3 1 3  41 3 1
Từ đây, ta tìm được chặn trên cho x0 là x0  . Như vậy  x0  . Vì
4 16 4
3 1
các đánh giá khá sát nên ta mạnh dạn chọn x  để thay vào f ( x). Để ý rằng x là
4
4x  1 1 3
nghiệm của phương trình 8 x 2  4 x  1  0, do đó x 2  ,  8 x  4,  4  4 x.
8 x 2x  1

Suy ra
4x  1 45 95 45 3  1 95 45 3  50
f ( x)   2 x  2(8 x  4)  (4  4 x)  x     .
8 2 8 2 4 8 8

45 3  50 3
Với kết quả này, ta thu được k   45   25  45  0,866  25  13,97.
2 2

Tất nhiên, cũng có thể lựa chọn một giá trị nào đó đẹp hơn cho x để thuận tiện hơn nữa
cho việc tính toán nhưng ở đây ta nên thận trọng vì chỉ cần chọn một số x lệch hơi xa
so với x0 thôi là có thể đưa đến việc k sẽ bị lệch đi mấy đơn vị sang 14, 15 thậm chí là
16. Tốt nhất, ta vẫn cứ nên sử dụng giá trị nào đó mà ta đã biết chắc rằng nó “sát” với
x0 dù lẻ một chút cũng không sao, bù lại ta sẽ có thể yên tâm hơn về kết quả.

Sau khi đã tìm được k  13, ta có thể thử đi chứng minh bất đẳng thức đúng với k  13
bằng nhiều cách nhưng vì độ chặt của bất đẳng thức ( k là hằng số tốt nhất) nên tất
nhiên ta sẽ chọn giải pháp nào an toàn mà hiệu quả nhất và phương pháp mà ta chọn ở
đây là làm giảm số biến bằng dồn biến.
27
Cuối cùng, ta sẽ chỉ ra một cách để tìm hằng số thực k rất gần với hằng số tốt nhất để
bất đẳng thức (*) đúng.

Đặt F( x)  4(2 x 4  5 x 3  2 x 2  x  2)  k(2 x 2  2)  0, x    . Vì với hằng số tốt nhất thì


2
điểm cực tiểu x0 của đa thức gần với nên ta giả sử rằng với k cần tìm thì điểm cực
3
2
tiểu x0 bằng đúng , tức là đạo hàm F ( x)  32 x 3  60 x 2  16 x  4  k(4 x  1) bằng 0 tại
3
2 1372 2
x , ta tính được k  . Với giá trị k này thì F( x) đạt được cực tiểu tại nên
3 99 3
 2  152
F ( x)  F    0.
 3  891

1372
Do đó, bất đẳng thức (*) đúng với k   13, 85859 .
99

880
Chú ý rằng với k  k1   13,96825 và với k2  13,85859 thì (*) cũng đúng nên hằng
63
số thực tốt nhất k thỏa mãn k2  k  k1 (ta tính được k  13,96764 ). Một điều thú vị ở
đây là dù k2 dù lớn hơn 13 khá nhiều nhưng bất đẳng thức trong trường hợp tương
ứng lại dễ chứng minh hơn.

Nhận xét chung, bài toán này có hình thức khá đơn giản nhưng lại đòi hỏi nhiều xử lí
trung gian tinh tế, nhất là phần tính toán trong điều kiện thời gian có giới hạn và không
có máy tính hỗ trợ. Nhiều bạn chủ quan khi gặp bài này, đánh giá k vội vàng để ra
k  8 hoặc k  16 rồi từ đó “kết luận” luôn và để mất điểm đáng tiếc. Một suy nghĩ
thường thấy là việc chứng minh bất đẳng thức một biến là chuyện đơn giản (các bài
dồn biến thường đưa về các bất đẳng thức 1 biến hiển nhiên đúng) nhưng thực ra
không phải vậy; cũng như các phương trình đại số, bất đẳng thức một biến cũng có thể
khó và thậm chí là rất khó nếu như trong quá trình xử lí, chúng ta không thu được
nghiệm đặc biệt nào.

Có 2 bài toán có cách giải khá giống với bài 4 (phần chứng minh), tuy nhiên đơn giản
hơn. Có lẽ bài toán 4 được phát triển từ các bài toán dưới đây

(Vasile Cirtoaje, Algebraic Inequalities, Chapter 1, bài toán 50 và 55)

28
1. Cho a , b , c là các số thực dương sao cho abc  1 . Chứng minh rằng

( a  b)(b  c)(c  a)  7  5( a  b  c ) .

2. Cho a , b , c là các số thực dương sao cho abc  1 . Chứng minh rằng

2 1 3
  .
a  b  c 3 ab  bc  ca

Bài 5.

Cho tam giác ABC nhọn không cân có góc A bằng 45 . Các đường cao AD , BE, CF
đồng quy tại trực tâm H . Đường thẳng EF cắt đường thẳng BC tại P . Gọi I là trung
điểm của BC ; đường thẳng IF cắt PH tại Q.

1. Chứng minh rằng IQH  AIE .

2. Gọi K là trực tâm của tam giác AEF và ( J ) là đường tròn ngoại tiếp tam giác KPD .
Đường thẳng CK cắt ( J ) tại G , đường thẳng IG cắt ( J ) tại M , đường thẳng JC cắt
đường tròn đường kính BC tại N . Chứng minh rằng các điểm G , M , N , C cùng thuộc
một đường tròn.

Lời giải.

1. Giả sử AB  AC , khi đó B sẽ nằm giữa P , C . Trường hợp AB  AC được chứng


minh hoàn toàn tương tự.

Trước hết, ta sẽ chứng minh rằng PH vuông góc với AI .

Thật vậy, gọi U , V lần lượt là trung điểm của AH , IH thì ta có UV  AI.

Dễ thấy ( P , D , B, C )  1 nên theo tính chất của hàng điểm điều hòa thì PB  PC  PD  PI .

Ta cũng có PE  PF  PB  PC nên PE  PF  PD  PI hay P nằm trên trục đẳng phương


của đường tròn đường kính AH (tâm U ) và IH (tâm V ).

Hơn nữa H cũng nằm trên trục đẳng phương của hai đường tròn này nên PH  UV .

Do đó PH  AI .

Vì BAC  45 nên EIF  90 , suy ra IQH  90  AIF  EIF  AIF  AIE.

Vậy ta có IQH  AIE .


29
G A

K
J E
N

F H
M
Q
P T B D I C

2. Ta thấy rằng EKF  ECF  EKF  EAF  180 nên K thuộc đường tròn đường
kính BC. Do hàng điểm D , P , B, C điều hòa nên ta có ID  IP  IC 2 , mà IM  IG  ID  IP
(cùng bằng phương tích của I đến ( J ) ) nên IM  IG  IC 2 hay

IMC  ICG(c.g.c ) nên IMC  ICG  ICK  45. (1)

Gọi T là trung điểm PD thì theo hệ thức Maclaurin, ta có CB  CT  CD  CP  CK  CG


hay tứ giác GTBK nội tiếp và do BKG  90 nên cũng có GTD  90 hay GT  PD .

Tam giác JPD cân tại J và có T là trung điểm PD nên JT vuông góc với PD.

Do đó, G , J , T thẳng hàng và KGJ  45 .

Mặt khác CN  CJ  CB  CT  CK  CG nên tứ giác KNJG nội tiếp và dẫn đến

JNG  JKG  JGK  45 . (2)

Từ (1) và (2) suy ra GMC  GNC  135 hay các điểm G , M , N , C cùng thuộc một
đường tròn. Đây chính là đpcm.

30
Nhận xét.

Ở ý thứ 1, việc chứng minh PH vuông góc với AI có thể được thực hiện bằng nhiều
cách. Chẳng hạn, xét tứ giác BCEF nội tiếp đường tròn tâm I . Theo định lí Brocard thì
H chính là trực tâm của tam giác AIP nên PH vuông góc với AI .

AD PD
Cũng có thể dùng cách tính toán chứng minh  để chứng minh kết quả này.
ID HD
Ý thứ 2 là một ứng dụng rất đẹp của hang điểm điều hòa trong đó có kết hợp cả hai hệ
thức cơ bản là hệ thức Newton và hệ thức Maclaurin. Số bài toán về hàng điểm điều
hòa vận dụng đến cả hai hệ thức này trong một bài toán là không nhiều, nên câu 2) là
một ý chứng minh rất hay và thú vị. Ngoài cách giải ở trên, ta có thể sử dụng hướng
tiếp cận khác như sau:

Gọi L là giao điểm thứ hai khác K của hai đường tròn ( I ),( J ) . Khi đó, để chứng minh
G , M , N , C cùng thuộc một đường tròn, ta đưa về chứng minh GI , CJ , LK đồng quy.

Trước hết, ta có thể đi theo hướng hoàn toàn thuần túy như sau:

G A

K
J E
N

F
H M

P B D I C
L

31
Chú ý ( P , D , B, C )  1 và BKC  90 nên KB, KC lần lượt là phân giác trong và ngoài
của góc PKD . Từ đó dẫn đến DKI  DPK hay IK tiếp xúc với đường tròn ( J ) , điều
này có nghĩa là các đường tròn ( I ),( J ) trực giao và dẫn đến ILJ  GLC  90 .

Hơn nữa, GI và CJ lần lượt là các đường đối trung trong các tam giác GKL, CKL . Gọi
U là trung điểm KL thì GI , CJ đối xứng với GU , CU qua phân giác các góc
LGC , LCG của tam giác LGC .

Do các đường thẳng LK , GU , CU đồng quy tại U và LK là phân giác góc GLC nên các
đường thẳng LK , GI , CJ đồng quy.

Từ đó lời giải bài toán hoàn tất.

Tuy nhiên, ngay từ đầu, ta có thể phát biểu lại bài toán thành:

Cho tam giác vuông KIJ ở K và dựng phía ngoài các tam giác KJG vuông cân đỉnh J
và KIC vuông cân đỉnh I . Chứng minh GI , CJ đồng quy tại một điểm nằm trên đường
cao đỉnh K của tam giác KIJ .

(Chú ý rằng các tam giác GJK , CIK vuông cân là có thể chứng minh hoàn toàn dễ dàng).

Để giải bài toán này, ta chỉ cần biến đổi đại số thông qua định lí Ceva, Thales là có thể
xử lí nhanh chóng.

Tuy rằng một điều chưa hoàn hảo lắm trong bài toán này là câu 1) và câu 2) hầu như
không liên quan gì tới nhau.

Thực chất vai trò của giả thiết góc A bằng 45 cũng không thật sự cần thiết lắm trong
bài toán này. Các bạn có thể tìm hiểu một mở rộng đơn giản nhất như sau

Cho tam giác ABC đường cao AD , BE, CF đồng quy tại H , EF cắt BC tại G . Gọi ( K )
là đường tròn đường kính BC . Trung trực của BC cắt ( K ) tại điểm L sao cho A , L nằm
cùng phía đối với BC . Gọi ( N ) là đường tròn ngoại tiếp tam giác GDL . Đường thẳng
CL cắt ( N ) tại M khác L . Đường thẳng MK cắt ( N ) tại P khác M . Đường thẳng
CN cắt ( K ) tại Q khác C . Chứng minh rằng M , P , Q , C cùng thuộc một đường tròn.

Cách chứng minh hoàn toàn tương tự như lời giải bài toán gốc.

Thực sự bài toán này được đánh giá rất cao không những vì phát biểu đẹp và chứng
minh hay của nó mà còn bởi vì tính phát triển của nó. Trong bài toán này, còn có rất
nhiều những khám phá mới đang đợi các bạn cùng suy ngẫm.
32
Bài 6.

Cho một khối lập phương 10  10  10 gồm 1000 ô vuông đơn vị màu trắng. An và Bình
chơi một trò chơi. An chọn một số dải 1  1  10 sao cho hai dải bất kì không có chung
đỉnh hoặc cạnh rồi đổi tất cả các ô sang màu đen. Bình thì được chọn một số ô bất kì
của hình lập phương rồi hỏi An các ô này màu gì. Hỏi Bình phải chọn ít nhất bao nhiêu
ô để với mọi câu trả lời của An thì Bình luôn xác định được những ô nào là màu đen?

Lời giải.

Trước hết, ta sẽ chứng minh nhận xét tổng quát:

Cho một khối lập phương 2n  2n  2n gồm 8n3 ô vuông đơn vị màu trắng. An và Bình chơi
một trò chơi. An chọn một số dải 1  1  n sao cho với hai dải bất kì thì chúng không có chung
đỉnh hoặc cạnh rồi đổi tất cả các ô sang màu đen. Bình thì được chọn một số ô bất kì của hình lập
phương rồi hỏi An các ô này có màu gì. Khi đó, Bình cần chọn ít nhất 6n2 ô mới có thể xác định
được ô nào có màu đen.

Thật vậy, gọi Sn là tập hợp các khối mà Bình cần phải chọn để hỏi An và với mỗi ô u
được Bình chọn, đặt Ru là hợp của các ô thuộc ba dải ngang, dọc, chéo đi qua u . Ta
thấy rằng:
33
Do điều kiện hai dãy được chọn bất kì không chung cạnh và đỉnh nên với mỗi câu trả
lời về màu cho mỗi ô u mà Bình chọn. Ta thấy rằng nếu gọi Ru là hợp của các ô được
chọn thì khi ô đó màu đen, sẽ có đúng một trong ba dải ngang, dọc, chéo đi qua ô đó
được tô màu đen. Trong trường hợp này, ta cần chọn thêm một số ô thuộc Ru để biết
chính xác dải đó. Nếu chỉ chọn thêm một ô thôi thì khi An trả lời ô đó màu trắng, Bình
sẽ không xác định được dải nào trong hai dải còn lại được tô đen. Do đó, Bình phải
chọn thêm ít nhất hai ô nữa trong Ru mới có khả năng trả lời được. Thêm vào đó, hai ô
đó phải thuộc hai dải khác nhau vì nếu chúng cùng thuộc một dải thì cũng tương tự
như trường hợp chọn một ô nêu trên.

Với nhận xét này, ta gán cho mỗi ô u của hình lập phương một bộ số ( a , b , c ) với định
nghĩa như sau:

 a  2 nếu dải hình hộp theo chiều ngang đi qua u không có thêm điểm nào
thuộc Sn và a  1 nếu ngược lại.

 b  2 nếu dải hình hộp theo chiều dọc đi qua u không có thêm điểm nào thuộc
Sn và b  1 nếu ngược lại.
34
 c  2 nếu dải hình hộp theo chiều chéo đi qua u không có thêm điểm nào thuộc
Sn và c  1 nếu ngược lại.

Khi đó, có hai trong ba số a , b , c có giá trị bằng 1 và số còn lại không vượt quá 2 nên
abc  4.

Đặt T là tổng các số dùng để gán cho các ô của hình lập phương. Khi đó, ta có

T  ( a  b  c)  4 S
uSn
n
.

Mặt khác, với mỗi dãy 1  1  n của khối lập phương (theo cả ba chiều) đều có ít nhất
một khối thuộc tập hợp Sn vì nếu không, Bình sẽ không có thông tin gì về dải đó và
trong trường hợp An trả lời rằng tất cả các ô được Bình chọn đều được tô màu trắng thì
Bình sẽ không biết được dải còn lại đó có được tô màu đen hay không.

Dễ thấy rằng có tất cả (2n)2 dải 1  1  2n nằm ngang và tất cả các dải này sẽ đóng góp ít
nhất 2(2n)2 đơn vị vào T (đóng góp vào các số a theo định nghĩa như trên).

Tương tự với (2n)2 dải 1  1  2n dọc và (2n)2 dải 1  1  2n chéo nên ta suy ra

T  3  2(2n) 2  24n2 .

Từ đó, ta được 4 Sn  24n2 hay Sn  6n2 , n   

Đến đây, ta suy ra hai điều sau:

- Trong hình lập phương 2  2  2 , Bình cần chọn ít nhất 6 ô.

- Trong hình lập phương 10  10  10 , Bình cần chọn ít nhất 150 ô.

Ta sẽ chỉ ra cách tô màu 6 ô thỏa mãn đề bài và từ đó chỉ ra cách xây dựng cho hình lập
phương 10  10  10 đã cho. (Trên thực tế, ta hoàn toàn có thể xây dựng cho trường hợp
tổng quát cho hình lập phương 2n  2n  2n ).

Thật vậy, trong hình lập phương 2  2  2 , trừ hai ô nào đó đối xứng nhau qua tâm, Bình
chọn 6 ô còn lại như hình bên dưới.

Ta sẽ chứng minh cách chọn này thỏa mãn yêu cầu.

35
Dễ thấy rằng trong hình lập phương 2  2  2 , chỉ có không quá 1 dải được tô màu nên
hoặc không có ô nào hoặc có 2 ô của hình lập phương được tô màu và 2 ô đó phải thuộc
cùng 1 dải. Do đó, trong 6 ô được chọn, ta có 3 trường hợp:

- Nếu không có ô nào được tô đen thì cả hình lập phương không có.

- Nếu có đúng 1 ô được tô đen thì ô đen còn lại sẽ thuộc trong 2 ô không được chọn có
cùng 1 dải với ô đen đã biết.

- Nếu có 2 ô được tô đen thì đó chính là tất cả các ô đen của hình lập phương.

Do đó, cách chọn này với hình lập phương 2  2  2 thỏa mãn điều kiện đề bài.

Tiếp theo, ta xây dựng cho hình lập phương 10  10  10 như sau:

Ta chia hình lập phương thành 5 lớp 10  10  2 và ta chia nó thành 25 phần, mỗi phần là
một hình lập phương 2  2  2 rồi đánh số như hình bên dưới:

1 2 3 4 5

5 1 2 3 4

4 5 1 2 3

3 4 5 1 2

2 3 4 5 1

36
Ở lớp thứ i với i  1, 2,3, 4, 5 , ta chọn các khối được đánh số i và với mỗi khối đó, ta bỏ
đi 2 ô bất kì đối xứng nhau qua tâm như cách đã nêu ở trên rồi chọn 6 còn lại.

Dễ thấy rằng với cách tô như vậy, ta chiếu các hình lập phương đã được chọn xuống
một mặt nào đó thì các hình chiếu sẽ phủ kín mặt đó. Điều này có nghĩa là với một dải
bất kì mà An đã chọn thì nó đều đi qua một trong các hình lập phương 2  2  2 mà Bình
chọn như trên. Khi đó, như đã chứng minh ở trên, ta sẽ xác định được rằng dải đó có
được tô màu hay không, tức là xác định được màu của tất cả các ô được tô màu đen của
hình lập phương ban đầu. Do đó, cách chọn các ô này thỏa mãn đề bài.

Vậy số ô ít nhất mà Bình cần chọn là 150.

Nhận xét.

Có thể nói đây là bài toán khó nhất của kỳ thi, tuy nhiên, cái khó của nó không nặng
tính kỹ thuật như bài 3 mà là cái khó về mặt phương pháp. Thực tế nhiều thí sinh đã
nói rằng “Em thực sự không biết phải xoay sở thế nào!”.

Để làm được bài toán này, trước hết phải thật tập trung để hiểu rõ yêu cầu của bài toán
và phải dùng trường hợp n  2 hoặc xét bài toán 2 chiều để hình dung bài toán một
cách cụ thể nhất. Trong các nhận xét ở trên thì nhật xét rằng mỗi dải đều có một ô được
chọn là khá hiển nhiên (và nhiều thí sinh đã phát biểu được nhận xét này), còn nhận xét
rằng ngoài khối u thì trong hợp Ru còn có ít nhất 2 khối nữa được chọn thì tinh tế hơn.
Khi đã có hai nhận xét này thì việc áp dụng kỹ thuật đếm bằng hai cách để đánh giá là
khá tự nhiên.

Việc xây dựng cấu hình cách chọn cũng là một thách thức.

Ở đây, trường hợp n  2 đóng một vai trò quan trọng, như những viên gạch để ta xây
dựng. Rõ ràng các cấu trúc cơ bản như hình vuông Latin ở trong bài giải đã giúp chúng
ta giải quyết bài toán. Các mô hình, cấu trúc, cách sắp xếp cơ bản vì vậy luôn đóng một
vai trò quan trọng trong các bài toán xây dựng ví dụ, phản ví dụ. Nếu có làm quen
nhiều với các bài toán về cực trị rời rạc thì có thể xử lí bài này, ít nhất là việc xây dựng
một cách chủ động hơn.

Trong các kỳ TST trước đây, đã có một số bài toán sử dụng phương pháp đếm bằng hai
cách để đánh giá, chẳng hạn như:

37
(Việt Nam TST 2000) Trong mặt phẳng cho 2000 đường tròn bán kính 1 sao cho không có
hai đường tròn nào tiếp xúc nhau và hợp của các đường tròn này tạo thành một tập hợp
liên thông. Chứng minh rằng số các điểm thuộc ít nhất hai đường tròn trong các đường
tròn đã cho không nhỏ hơn 2000.

(Việt Nam TST 2010) Gọi một hình chữ nhật có kích thước 1  2 là hình chữ nhật đơn và
một hình chữ nhật có kích thước 2  3 , bỏ đi 2 ô ở góc chéo nhau (tức có có 4 ô vuông
con) là hình chữ nhật kép. Người ta ghép khít các hình chữ nhật đơn và hình chữ nhật
kép này lại với nhau được một bảng hình chữ nhật có kích thước là 2008  2010 . Tìm số
nhỏ nhất các hình chữ nhật đơn có thể dùng để ghép.

Một lần nữa thì trong bài toán cực trị ở kì thi chọn đội tuyển, dấu bằng xảy ra có thể xây
dựng không quá khó nhưng để chỉ ra đó là giá trị tốt nhất thì là điều hoàn toàn không
dễ dàng, thậm chí rất khó!

Một vấn đề thú vị đặt ra là nếu kích thước của hình lập phương là lẻ thì ta sẽ có kết quả
thế nào? Áp dụng cách đánh giá chặn dưới như trường hợp n chẵn, ta được nếu
n  2 k  1 thì số hình vuông cần được đánh dấu phải thỏa mãn bất đẳng thức
3(2 k  1)2
T  . Từ đây suy ra T  6 k 2  6 k  2 . Tuy nhiên, việc xây dựng cách chọn
2
6 k 2  6 k  2 hình lập phương đơn vị thỏa mãn yêu cầu bài toán vẫn là một câu hỏi mở.
Với n  3 , tức là k  1 , hiện nay chưa tìm được cách chọn 14 hình lập phương thỏa mãn
yêu cầu bài toán và cũng chưa chứng minh được là cần nhiều hơn. (Ta có thể chỉ được
15 hình lập phương thỏa mãn yêu cầu).

Ta thử thay giả thiết đã nêu trong đề bài một chút để có bài toán mới: “Bình chọn một ô
bất kì của hình lập phương rồi hỏi An ô này có màu gì rồi sau đó tiếp tục như thế” và
các dữ kiện khác vẫn giữ tương tự.

Hoặc bài toán trong trường hợp hai chiều: Thay khối lập phương bởi bảng ô vuông và
thay các dải ô vuông được chọn bởi các hàng, cột của bảng.

Khai thác các trường hợp này sẽ giúp ta thu được nhiều kết quả thú vị khác!

38
Trần Nam Dũng (chủ biên)
Võ Quốc Bá Cẩn – Trần Quang Hùng – Lê Phúc Lữ
Hoàng Đỗ Kiên – Nguyễn Huy Tùng

LỜI GIẢI VÀ BÌNH LUẬN


ĐỀ CHỌN ĐỘI TUYỂN QUỐC GIA
DỰ THI IMO 2014

Thành phố Hồ Chí Minh, ngày 05 tháng 06 năm 2014


Việt Nam Team Selection Test 2014 2

Đề thi chính thức


Ngày thi thứ nhất - 25/03/2014
Bài 1. Tìm tất cả các hàm số f : Z → Z thỏa mãn

f 2m + f (m) + f (m) f (n) = n · f (m) + m,



∀m, n ∈ Z.

Bài 2. Trong mặt phẳng tọa độ vuông góc Ox y, xét các điểm nguyên có tọa độ thuộc tập
hợp sau
T = (x ; y) : −20 ¶ x , y ¶ 20, (x ; y) 6= (0; 0) .


Tô màu các điểm thuộc T sao cho với mọi điểm có tọa độ (x , y) ∈ T thì có đúng một trong
hai điểm (x ; y) và (−x ; −y) được tô màu. Với mỗi cách tô như thế, gọi N là số các bộ
(x 1 ; y1 ), (x 2 ; y2 ) mà cả hai điểm này cùng được tô màu và

x 1 ≡ 2x 2 , y1 ≡ 2y2 (mod 41).

Tìm tất cả các giá trị có thể có của N.


Bài 3. Cho tam giác ABC nội tiếp trong đường tròn (O). Trên cung BC không chứa A của
(O) lấy điểm D. Giả sử C D cắt AB ở E và BD cắt AC ở F . Gọi (K ) đường tròn nằm trong
tam giác EBD, tiếp xúc với EB, ED và tiếp xúc với đường tròn (O). Gọi (L) là tâm đường
tròn nằm trong tam giác F C D, tiếp xúc với F C , F D và tiếp xúc với đường tròn (O).

a) Gọi M là tiếp điểm của (K ) với BE và N là tiếp điểm của (L) với C F . Chứng minh
rằng đường tròn đường kính M N luôn đi qua một điểm cố định khi D di chuyển.

b) Đường thẳng qua M và song song với C E cắt AC ở P, đường thẳng qua N và song
song với BF cắt AB ở Q. Chứng minh rằng đường tròn ngoại tiếp tam giác AM P, ANQ
cùng tiếp xúc với một đường tròn cố định khi D di chuyển.

Ngày thi thứ hai - 26/03/2014


Bài 4. Cho tam giác ABC nhọn, không cân có đường cao AD và P thuộc AD. Các đường
thẳng P B, PC lần lượt cắt C A, AB tại E, F.
PA A
a) Giả sử tứ giác AEDF nội tiếp. Chứng minh rằng = (tan B + tan C ) cot .
PD 2
b) Gọi C P cắt đường thẳng qua B vuông góc AB tại M. BP cắt đường thẳng qua C vuông
góc AC tại N. K là hình chiếu của A lên M N. Chứng minh rằng ∠BKC + ∠M AN
không đổi khi P di chuyển trên AD.

Bài 5. Tìm tất cả đa thức P(x ), Q(x ) có hệ số nguyên và thỏa mãn điều kiện: Với dãy số
(x n ) xác định bởi

x 0 = 2014, x 2n+1 = P(x 2n ), x 2n = Q(x 2n−1 ) với n ¾ 1.

thì mỗi số nguyên dương m là ước của một số hạng khác 0 nào đó của dãy (x n ).
Việt Nam Team Selection Test 2014 3

Bài 6. Cho m, n, p là các số tự nhiên không đồng thời bằng 0. Không gian tọa độ được chia
thành các mặt phẳng song song cách đều nhau. Một cách điền vào mỗi khối lập phương
đơn vị một trong các số từ 1 đến 60 được gọi là cách điền Điện Biên nếu thỏa mãn: trong
mỗi hình hộp chữ nhật với các mặt trên hệ mặt đã cho và tập hợp kích thước ba cạnh (số
hình lập phương trên trên cạnh) xuất phát từ một đỉnh là 2m + 1, 2n + 1, 2p + 1, khối lập
phương đơn vị có tâm trùng với tâm của hình hộp chữ nhật được điền số bằng trung bình
cộng của các số điền ở tâm của 8 hình lập phương ở các góc của hình hộp đó. Hỏi có tất cả
bao nhiêu cách điền Điện Biên?
Những cách điền là giống nhau nếu trong mỗi cách, ta chọn một hình lập phương làm tâm
thì các số được điền vào các khối lập phương đơn vị có cùng vị trí tương đối với tâm trong
hai cách điền là bằng nhau.
Việt Nam Team Selection Test 2014 4

Nhận xét chung


Xét về cấu trúc, đề thi chọn đội tuyển thi IMO năm nay gồm 6 bài toán, trong đó có 2 bài
hình học phẳng, 2 bài đại số và 2 bài tổ hợp. Cụ thể như sau:

• Bài 1: Đại số (phương trình hàm trên tập số nguyên).

• Bài 2: Tổ hợp (bài toán đếm).

• Bài 3: Hình học phẳng (đường và điểm cố định).

• Bài 4: Hình học phẳng (hình học tính toán).

• Bài 5: Đại số (phương trình hàm đa thức trên tập số nguyên).

• Bài 6: Tổ hợp (bài toán đếm)

Ngoài hai bài hình ra thì tính chất số học xuất hiện ở cả bốn bài còn lại nhưng chỉ đóng
các vai trò làm nền để xây dựng các vấn đề chứ không thực sự là các bài số học. Trong mỗi
ngày, các bài toán được sắp xếp một cách tương đối theo độ khó tăng dần. Bài 1 và 4 có thể
coi là hai bài dễ. Tiếp theo là bài 2, bài 5 ở mức độ trung bình và khó nhất là bài 3 và bài
6. Theo ý kiến của chúng tôi, để lọt vào đội tuyển, các thí sinh chắc chắn sẽ phải giải quyết
được bài 1, 4 và ít nhất làm được hoàn chỉnh 1 bài trong các bài 2, 5 (cũng là các bài quyết
định), còn lại khó có thí sinh nào giải quyết được trọn vẹn bài 3 và 6.
Hai bài hình học năm nay, xét về tính phân loại thì ổn, có một bài dễ và một bài khó, trong
đó câu 3b sẽ là thách thức đáng kể. Bài 4 khá dễ khi xét lại các bài cùng vị trí ở kỳ thi các
năm trước. Tuy nhiên, nếu đặt vào chung với các bài 5 và 6 trong một ngày thi thì lại khá
hợp lý vì sẽ tạo một thử thách nhỏ cho thí sinh trước khi bước vào hai thử thách lớn hơn. Về
bốn bài còn lại, kiến thức được sử dụng nói chung rất quen thuộc, đòi hỏi tư duy thuần túy
và trình bày cẩn thận chứ không dùng các định lý khó. Bài 1 là phương trình hàm trên tập
số tự nhiên được tính vào đại số, nếu nắm vững cách xử lý phương trình hàm thì sẽ vượt
qua bài này không mấy khó khăn. Bài 5 về đa thức, cũng là đại số nhưng có liên quan đến
các tính chất số học, giải tích. Bài 2 và 6 đều là các bài tổ hợp nhưng được đặt trên nền số
học, xu hướng thường thấy trong các đề VMO, TST những năm gần đây.
Ta có thể thấy một điểm khá trùng hợp ở ba bài 2, 5 và 6 là mỗi bài đều có khai thác yếu
tố chiều và vì vậy mà bài toán có thể hình dung dễ hơn khi đưa về trường hợp 1 chiều. Ba
bài này cũng có nhiều điểm khó, cả về cách tiếp cận, lập luận chặt chẽ trong trình bày lời
giải lẫn đáp số cuối cùng của bài toán. So với đề thi các năm trước thì có thể thấy đề TST
2014 có phần nhẹ hơn, không có bài quá khó và hai bài dễ của kỳ thi cũng dễ hơn. Tuy vậy,
đề thi vẫn đảm bảo tính phân loại cao, các bài hình đòi hỏi nắm các kỹ thuật nhất định để
xử lý còn các bài khác thì lại đòi hỏi tư duy là chính, hạn chế sử dụng các định lý, công cụ
mạnh là một đặc điểm nổi bật và cũng là ưu điểm của đề thi này.
Đề thi, lời giải và bình luận dưới đây có tham khảo từ bình luận của GS. Nguyễn Tiến
Dũng trên page SputnikEducation và sự đóng góp của các bạn: traum (Lê Hồng Quý),
chemthan (Nguyễn Ngọc Trung), LTL (Nguyễn Văn Linh), blackholes (Huỳnh Việc Trung),
haojack123 (Trần Anh Hào) cùng một số thành viên khác của diễn đàn mathscope.org.
Việt Nam Team Selection Test 2014 5

Lời giải chi tiết và bình luận

Bài 1.
Tìm tất cả các hàm f : Z → Z thỏa mãn

f 2m + f (m) + f (m) f (n) = n f (m) + m, ∀m, n ∈ Z. (∗)




Lời giải. Đặt a = f (0). Rõ ràng f ≡ 0 không phải là nghiệm của phương trình, do đó tồn
tại m 0 ∈ Z sao cho f (m 0 ) 6= 0. Từ đó, bằng cách thay m = m 0 vào đẳng thức (∗) ta dễ
dàng suy ra f đơn ánh. Mặt khác, thay n = 0 vào (∗), ta được

f 2m + (a + 1) · f (m) = m, ∀m ∈ Z. (∗∗)


Kết quả này chứng tỏ f là toàn ánh. Do đó, ∃b ∈ Z sao cho f (b) = −1. Thay m = n = b
vào (1), ta được f (2b) = 0. Mặt khác, thay m = n = 0 vào (∗), ta cũng có f (a2 + a) = 0.
Từ đây, kết hợp với tính đơn ánh của f , ta suy ra

a2 + a
b= .
2
Bây giờ, thay n = b vào (∗), ta được

a2 + a
f (2m) = · f (m) + m, ∀m ∈ Z. (∗ ∗ ∗)
2

Tiếp tục, thay m = 0 vào (∗), ta được

f a · f (n) + a = an,

∀n ∈ Z.

Đến đây, bằng cách thay m = an vào (∗∗) rồi so sánh với đẳng thức trên, ta suy ra

(a + 1) · f (an) + 2an = a · f (n) + a, ∀n ∈ Z.

a(a2 + a)
Còn nếu thay n = b, ta được = f (0) = a, từ đó suy ra a ∈ {0, 1, −2}. Ta xét các
2
trường hợp sau:

• Nếu a = 1 thì dễ dàng suy ra f (n) = 1 − 2n, ∀n ∈ Z. Tuy nhiên, khi thử lại ta thấy
hàm này không thỏa mãn yêu cầu đề bài.

• Nếu a = 0 thì từ (∗ ∗ ∗), ta suy ra f (2m) = m, ∀m ∈ Z. So sánh kết quả này với (∗∗),
ta được f (m) = 0, ∀m ∈ Z, vô lý vì f phải là một hàm khác hằng.

• Nếu a = −2, ta có

f (−2n) + 4n = 2 · f (n) + 2, ∀n ∈ Z.
Việt Nam Team Selection Test 2014 6

Mặt khác, theo (∗ ∗ ∗), ta lại có f (−2n) = f (−n) − n. Do đó, ta được

f (−n) + 3n = 2 · f (n) + 2, ∀n ∈ Z.

Thay n bởi −n vào đẳng thức trên, ta cũng có f (n) − 3n = 2 · f (−n) + 2, ∀n ∈ Z.


Kết hợp hai điều này lại, ta suy ra f (n) = n − 2, ∀n ∈ Z. Thử lại, ta thấy hàm này
thỏa mãn yêu cầu đề bài.

Vậy hàm số cần tìm là f (n) = n − 2, ∀n ∈ Z.

Nhận xét.
Ở bài toán trên, tính đơn ánh – toàn ánh được nhận thấy khá dễ dàng và nó còn đúng với
mọi số thực chứ không chỉ trên tập số nguyên. Đây cũng là điều tiên quyết cần có để việc
thay thế các giá trị có thể thực hiện dễ dàng.
Ngoài cách giải trên, ta cũng có thể dự đoán ra biểu thức của hàm số và bằng cách chọn
các số đặc biệt, ta có thể xử lý nhanh chóng được “nhu cầu” này. Việc phát hiện ra hàm số
f (n) = n − 2 nói chung không khó vì nếu hàm số cần tìm là đa thức thì dễ thấy bậc của nó
là 1 và từ đó thay vào đồng nhất hệ số. Bằng cách thay các giá trị thích hợp, ta tính được

f (4) = 2, f (3) = 1, f (2) = 0, f (1) = −1, f (0) = −2.

Thay m = 3 vào công thức đã cho, ta có ngay

f 7 + f (n) = n + 3 với mọi n.




Từ đây có thể quy nạp để có kết quả trên. Một cách khác cũng khá nhanh như sau:
Đặt a = f (0) và u, v, w là các số nguyên thỏa f (u) = 0, f (v) = −1, f (w ) = 1. Ta thực
 biểu thức của hàm số đã cho như sau: Nếu m = w, n = u − w
hiện thay các giá trị thích vào
thì f 2w + 1 + f (u − w ) = u.
Nếu m = u, n = 0 thì f (2u) = u. Suy ra

2w + 1 + f (u − w ) = 2u ⇒ f (u − w ) = 2(u − w ) − 1.

Tiếp tục thay m = x ∈ Z, n = u − w thì f 2(x + (u − w ) f (x )) = x + (u − w ) f (x ).




Cuối cùng, thay m = x + (u − w ) f (x ), n = v thì


u−x
x + (u − w ) f (x ) = u hay f (x ) = .
u−w

Đây là biểu thức tuyến tính theo x , ta dễ dàng chọn được các giá trị thích hợp.
Ngoài ra, ta còn có thể tiếp cận theo cách sau: Ta chứng minh được rằng f (2m− f (m)) = m
và do f song ánh nên f (m)+ f −1 (m) = 2m. Đây là bài xuất hiện trong kỳ thi APMO 1989:
Xác định tất cả các hàm số f : R → R thỏa mãn

• f đơn điệu tăng.

• f (x ) + g (x ) = 2x với g là hàm ngược của f .


Việt Nam Team Selection Test 2014 7

Bài 2.
Trong mặt phẳng tọa độ vuông góc Ox y, xét các điểm nguyên có tọa độ thuộc tập
hợp sau
T = (x ; y) : −20 ¶ x , y ¶ 20, (x ; y) 6= (0; 0) .


Tô màu các điểm thuộc T sao cho với mọi điểm có tọa độ (x , y) ∈ T thì có đúng một
trong hai điểm (x ; y) và (−x ; −y) được tô màu. Với mỗi cách tô như thế, gọi N là số
các bộ (x 1 ; y1 ), (x 2 ; y2 ) mà cả hai điểm này cùng được tô màu và

x 1 ≡ 2x 2 , y1 ≡ 2y2 (mod 41).

Tìm tất cả các giá trị có thể có của N.

Lời giải. Trước hết, ta chuyển bài toán từ 2 chiều thành 1 chiều.
Ta có 210 ≡ −1(mod 41) nên 40 số nguyên khác 0 có giá trị tuyệt đối không vượt quá 20
có thể chia thành 2 dãy, mỗi dãy có độ dài 20 sao cho nếu số hạng đầu chia 41 dư x thì số
hạng sau chia 41 dư 2x .
Tô màu các số của dãy này sao cho trong 2 số đối nhau thì có đúng 1 số được tô màu. Ta
quan tâm đến số lượng các cặp số liên tiếp cùng được tô màu trong dãy.
Dễ thấy 220 ≡ 1(mod 41) nên nếu thêm số hạng đầu của mỗi dãy vào cuối thì dãy mới
gồm 21 số vẫn thỏa mãn tính chất trên nên ta có thể chuyển thành vòng tròn và phát biểu
lại bài toán như sau: Cho đa giác đều 20 đỉnh nội tiếp trong một đường tròn sao cho trong
2 điểm đối xứng qua tâm thì có đúng 1 đỉnh được tô màu. Tính số các cặp đỉnh liên tiếp
được tô màu có thể có.
Với n là số chẵn, gọi Sn là tập hợp số các cặp kề nhau cùng được tô màu có thể có của đa
giác có n đỉnh. Ta sẽ chứng minh bằng quy nạp rằng

S4n = {2k + 1|0 ¶ k ¶ n − 1} và S4n−2 = {2k|0 ¶ k ¶ n − 1}.

Điều này có thể chứng minh bằng quy nạp như sau:
Với n = 2 thì dễ thấy nhận xét đúng.
Giả sử nhận xét đúng đến n ¾ 2. Xét đa giác có 4n + 2 đỉnh. Đa giác này có thể tạo thành
bằng cách thêm đỉnh A vào giữa hai đỉnh thứ 2n, 2n + 1 và thêm đỉnh B vào giữa hai đỉnh
thứ 4n, 1. Ta xét các trường hợp:
• Nếu đỉnh 2n, 2n + 1 đều được tô và A không được tô thì tương ứng: đỉnh 4n, 1 không
được tô và B được tô. Số cặp kề nhau cùng được tô giảm đi 1.

• Nếu đỉnh 2n, 2n + 1 đều được tô và A cũng được tô thì tương ứng: đỉnh 4n, 1 không
được tô và B không được tô. Số cặp kề nhau tăng lên 1.

• Nếu trong hai đỉnh 2n, 2n + 1 có 1 đỉnh được tô và A cũng được tô thì tương ứng:
trong hai đỉnh 4n, 1có 1 đỉnh được tô và B không được tô. Số cặp kề nhau tăng lên 1.

• Nếu trong hai đỉnh 2n, 2n + 1, có 1 đỉnh được tô và A không được tô thì tương ứng:
trong hai đỉnh 4n, 1 có 1 đỉnh được tô và B được tô. Số cặp kề nhau tăng lên 1.
Việt Nam Team Selection Test 2014 8

Do đó, S4n+2 = {x ± 1|x ∈ S4n } hay S4n+2 = {2k|0 ¶ k ¶ n}.


Tương tự, ta cũng có S4n+4 = {x ± 1|x ∈ S4n+2 }. Tất nhiên không xảy ra trường hợp S4n+4
có chứa số −1 vì để có trường hợp 0 cặp số ở 4n +2, các đỉnh phải được tô xen kẽ và trường
hợp giảm đi số bộ không xảy ra, suy ra S4n+4 = {2k + 1|0 ¶ k ¶ n}.
Rõ ràng trong cách chứng minh trên, ta cũng đã chỉ ra được cách xây dựng các trường hợp
để có thể tô màu thỏa mãn được tất cả các giá trị trong tập hợp tương ứng. Nhận xét được
chứng minh.
Từ đó suy ra S20 = {1, 3, 5, 7, 9} .
Do đó, kết quả cho bài toán phụ sẽ là 2S20 với định nghĩa 2S = S + S = {a + b|a, b ∈ S}.
Quay trở lại bài toán ban đầu, để chuyển từ 1 thành phần x ở trên thành 2 thành phần
(x , y), ta có thể thực hiện như sau:
Ứng với mỗi vòng tròn chứa các số thuộc dãy A, ta lấy một vòng tròn mới cũng gồm các số
thuộc dãy A đặt lên đó sao cho mỗi số thuộc đường tròn cũ khớp với đúng một số thuộc
đường tròn mới. Viết các cặp số khớp nhau thành một dãy, dãy đó chính là dãy các tọa độ
điểm mà liền sau của (x 1 , y1 ) là (2x 1 , 2y1 ) theo mod 41.
Dễ thấy có tất cả 20 cách ghép như thế (cố định vòng tròn cũ và xoay vòng tròn mới).
Tương tự với việc ghép các dãy A − B, B − A, B − B nên có tổng cộng là 80 cách ghép tạo
thành 80 dãy. Tuy nhiên, ta cũng xét thêm 4 dãy đặc biệt, tương ứng với các điểm nằm trên
trục tung và trục hoành.
Cụ thể là xét thêm dãy C gồm 20 số 0 và xét 4 cách ghép: A − C , C − A, B − C , C − B. Do
đó, tổng cộng có 84 dãy các tọa độ.
Theo chứng minh ở trên thì ở mỗi dãy, số các cặp có thể có là S20 vậy nên đáp số của bài
toán là 84S20 , cũng chính là các số chẵn từ 1 · 84 = 84 đến 9 · 84 = 756. Bài toán được giải
quyết hoàn toàn.

Nhận xét.
Để giải được bài 2, ta phải “cảm nhận” được bài toán. Một cách tự nhiên khi lấy x , y thì
nghĩ đến việc tìm 2x , 2y (thay vì ngược lại là xét các điểm cùng được tô màu). Từ đó, khi
biết các số có thể xoay thành một vòng tròn thì sẽ nhận thấy được miền giá trị kia chính là
số cạnh và việc đưa thành một mô hình như đã nêu sẽ dễ trình bày nhất, mọi thứ đều sáng
sủa hơn nhiều. Sẽ có ích khi ta phát biểu bài toán tổng quát sau đó làm việc với các tham số
nhỏ để tìm ra quy luật. Bài này bản chất không khó, nhưng rất dễ sai nhất là khi kết luận
vội vàng thông qua một số nhận xét nhỏ.
Theo ý kiến chúng tôi, việc chấm bài này khá là khó khăn trong việc đánh giá các bài giải
theo hướng 7− hay 0 + . Điểm mấu chốt ở đây là việc tách riêng hoành độ, tung độ, mô
hình hóa dưới dạng đa giác đều và chú ý đến đồng dư thức 210 = 1024 ≡ −1(mod 41).
Bài toán này có phong cách của bài 4 VMO 2012, bài 3 VMO 2013, đều là các bài khá mới
mẻ, là dạng tổ hợp đếm có kết hợp số học, tuy phát biểu hơi có phần gượng ép trong việc
đặt vào trục tọa độ nhưng lại giúp học sinh dễ tưởng tượng hơn.
Việc giải quyết bài toán trong trường hợp 1 chiều mang tính quyết định để xử lý bài toán
với nhiều chiều hơn. Chẳng hạn ta phát biểu lại bài toán trên như sau:
Việt Nam Team Selection Test 2014 9

Trong không gian Ox y z, xét các điểm nguyên có tọa độ thuộc tập hợp

T = (x ; y; z) : −20 ¶ x , y, z ¶ 20, (x ; y; z) 6= (0; 0; 0) .




Tô màu các điểm thuộc T sao cho với mọi điểm có tọa độ (x ; y; z) ∈ T thì có đúng một
trong hai điểm (x ; y; z) và (−x ; −y; −z) được tô màu. Với mỗi cách tô như thế, gọi N là
số các bộ (x 1 ; y1 ; z1 ), (x 2 ; y2 ; z2 ) mà cả hai điểm này cùng được tô màu và các tọa độ của
chúng thỏa mãn điều kiện x 1 ≡ 2x 2 , y1 ≡ 2y2 , z1 ≡ 2z1 (mod 41). Tìm tất cả các giá trị có
thể có của N.
Lập luận tương tự như trên nhưng ở đây, ta lại đặt thêm một vòng tròn nữa trên hai vòng
tròn cũ. Ở trên ta có ord41 (2) = 20 nên nếu hợp thay 20 và 41 bởi các số khác. Ta thấy các
số 41, 20 quyết định độ dài chu kỳ của các dãy ở trên và trong trường hợp số lượng các số
chia hết cho chu kỳ này, nghĩa là ta có thể chia đều dãy số đã cho thành các vòng tròn thì
bài toán được giải quyết tương tự. Như thế, ta đi đến bài toán tổng quát:

Với k, n, p, r là số nguyên dương mà ord p (r ) (2k), xét tập hợp T các bộ số nguyên không
đồng thời bằng 0 là (x 1 , x 2 , · · · , x n ) sao cho |x i | ¶ k, i = 1, n. Tô màu các số thuộc T sao
cho có đúng một trong hai bộ (x 1 , x 2 , · · · , x n ) và (−x 1 , −x 2 , · · · , −x n ) được tô màu. Với
mỗi cách tô như thế, gọi N là số các bộ (x 1 , x 2 , · · · , x n ) và (x 0 1 , x 0 2 , · · · , x 0 n ) thỏa mãn:

• Cả hai bộ này cùng được tô màu.

• x i = r x 0 i (mod p). với i = 1, 2, 3, · · · , n.


‚‚ Œn Œ
1 2k 2
Khi đó, tất cả các giá trị có thể có của N là RSordp (r ) với R = +1 −1 và
k ord p (r )
số S2k được định nghĩa như trên.
Tất nhiên, có những trường hợp dãy không lặp lại thì phải tính toán theo cách khác vì rõ
ràng ta không thể xây dựng được thành một mô hình khép kín như đa giác đều ở trên. Bài
toán vẫn còn có thể phát triển theo nhiều hướng rất thú vị!

Bài 3.
Cho tam giác ABC nội tiếp trong đường tròn (O). Trên cung BC không chứa A của
(O) lấy điểm D. Giả sử C D cắt AB ở E và BD cắt AC ở F . Gọi (K ) đường tròn nằm
trong tam giác EBD, tiếp xúc với EB, ED và tiếp xúc với đường tròn (O). Gọi (L) là
tâm đường tròn nằm trong tam giác F C D, tiếp xúc với F C , F D và tiếp xúc với đường
tròn (O).

a) Gọi M là tiếp điểm của (K ) với BE và N là tiếp điểm của (L) với C F . Chứng
minh rằng đường tròn đường kính M N luôn đi qua một điểm cố định khi D di
chuyển trên (O).

b) Đường thẳng qua M và song song với C E cắt AC ở P, đường thẳng qua N và
song song với BF cắt AB ở Q. Chứng minh rằng đường tròn ngoại tiếp tam giác
AM P, ANQ cùng tiếp xúc với một đường tròn cố định khi D di chuyển.
Việt Nam Team Selection Test 2014 10

Lời giải. a) Trước hết, ta chứng minh bổ đề sau:


Bổ đề (định lý Sawayama và Thébault mở rộng). Cho tam giác ABC nội tiếp đường tròn
(O). D là một điểm thuộc tia đối tia BA. Đường tròn (K ) tiếp xúc DA, DC lần lượt tại M, N
và tiếp xúc ngoài (O). Chứng minh rằng M N đi qua tâm bàng tiếp ứng với đỉnh A của tam
giác ABC .

Chứng minh. Giả sử đường phân giác ngoài tại đỉnh C của tam giác ABC cắt đường tròn
(O) tại E khác C . Đường thẳng C E cắt M N tại J. Đường tròn (K ) tiếp xúc với đường tròn
(O) tại F . Dễ dàng thấy rằng E, N, F thẳng hàng. Thật vậy, vì E là trung điểm cung AB
chứa C của (O) nên OE ⊥ AB. Ngoài ra, ta cũng có M N ⊥ AB nên M N k OE. Mặt khác,
K , O, F thẳng hàng nên E, N, F cũng thẳng hàng.
Từ đây, ta có tam giác EAF và E N A đồng dạng, suy ra EA2 = EB2 = E N · E F .

B
C
F

Ta lại có
1 1
∠F M N = ∠F K N = ∠F OE = ∠FAE = ∠F C J
2 2
suy ra tứ giác C F M J nội tiếp. Do đó,

∠E F J = 180◦ −∠N F J = 180◦ −(∠N F M +∠M F J) = 180◦ −(∠J MC +∠MC J) = ∠M JC .

Từ đó, ta có 4E F J ∼ 4E J N, suy ra E J 2 = E N · E F = EA2 = EB2 .


Từ đây, bằng biến đổi góc, dễ dàng suy ra AJ là phân giác của ∠BAC nên J là tâm bàng
tiếp ứng với đỉnh A của tam giác ABC . Ta có điều phải chứng minh.
Việt Nam Team Selection Test 2014 11

Trở lại bài toán,


Giả sử đường tròn (K ) tiếp xúc ED tại G và đường tròn (L) tiếp xúc F D tại H. Theo bổ đề
ở trên thì MG, N H cùng đi qua tâm bàng tiếp J ứng với đỉnh A của tam giác ABC .
Ta sẽ chứng minh rằng ∠M J N = 90◦ . Thật vậy,
1 1
∠K E F +∠LF E = ∠DEB+ ∠DF C +∠DE F +∠DF E = 90◦ −∠BAC +180◦ −∠BDC = 90◦
2 2
do đó E K ⊥ LF . Ta cũng có MG ⊥ E K và LF ⊥ N H nên MG vuông góc N H tại J. Vậy
đường tròn đường kính M N luôn đi qua điểm J cố định. Ta có đpcm.
A

Q
O

B C

K N
M
D
L
G
E

H
Z

J
Y
F

b) Dễ thấy tam giác E MG cân và M P k C E nên ∠E MG = ∠EGM = ∠GM P. Do đó, M J


chính là phân giác ngoài tam giác AM P mà AJ là phân giác góc BAC nên suy ra J cũng là
tâm bàng tiếp góc A của tam giác AM P.
Gọi (R) là đường tròn tiếp xúc với C A, AB tại Y, Z và tiếp xúc ngoài với đường tròn (O).
Theo bổ đề ban đầu thì khi cho D hoặc E chạy ra xa vô tận thì các đường tròn (K ), (L)
trùng với đường tròn (R) và cũng có đoạn nối hai tiếp điểm đi qua J. Từ đây, dễ dàng suy ra
J là trung điểm Y Z. Do J là tâm bàng tiếp của AM P nên đường tròn (R) tiếp xúc AP, AM
tại Y, Z cũng tiếp xúc đường tròn ngoại tiếp tam giác AM P.
Việt Nam Team Selection Test 2014 12

Chứng minh tương tự, ta cũng có (R) cũng tiếp xúc với đường tròn ngoại tiếp tam giác
ANQ. Vậy đường tròn ngoại tiếp tam giác AM P, ANQ luôn tiếp xúc đường tròn (R) cố định.
Ta có đpcm.

Nhận xét. Với bài toán này, nếu ai đã quen thuộc với định lý Sawayama và Thébault và
các mở rộng của nó thì giải quyết không quá khó khăn. Nhưng nói chung thì với hầu hết
các thí sinh, một mặt phải xác định được mô hình nêu ra trong đề bài chính là dạng mở
rộng; mặt khác, các bạn phải chứng minh lại thành công định lý này trong thời gian tương
đối ngắn, bài toán này xứng đáng ở vị trí số 3. Đây là lần đầu tiên trong đề thi chọn đội
tuyển của Việt Nam có một bài hình ở vị trí này (một luật bất thành văn là bài 3 chính là
bài khó nhất đề thi).
Trước hết, ta nhắc lại định lý Sawayama và Thébault dạng gốc: Cho tam giác ABC nội tiếp
đường tròn (O) . D là một điểm thuộc đoạn BC . Đường tròn (K ) tiếp xúc DA, DC lần lượt
tại M, N và tiếp xúc trong với (O) . Chứng minh M Nđi qua tâm nội tiếp tam giác ABC .
Trước hết, ta thử tập trung vào khai thác ý a. Rõ ràng có thể thay đường tròn ngoại tiếp
thành đường tròn bất kỳ qua B, C , ta đã thu được một bài toán rất thú vị. Chú ý rằng câu
a) nói lên MG, N H đi qua điểm Jcố định. Ta có các bài toán sau:
Bài 3.1. Cho tam giác ABC và một đường tròn (O) bất kỳ cố định đi qua B, C . D là điểm
di chuyển trên (O) sao cho A, D khác phía với B, C . Giả sử C D cắt AB ở E và BD cắt AC ở
F. Gọi (K ) là đường tròn tiếp xúc với EB, ED lần lượt tại M, Nvà tiếp xúc trong với (O) .
Gọi (L) là đường tròn tiếp xúc với F C , F D lần lượt tại P, Q và tiếp xúc trong với (O) . Chứng
minh rằng giao điểm M N, PQ luôn nằm trên một đường tròn cố định khi D di chuyển.
Bài 3.2. Cho tam giác ABC nội tiếp đường tròn (O) . Gọi P là điểm di chuyển trên cung
BC chứa A của (O) . P B, PC cắt C A, AB lần lượt tại E, F. Đường tròn (K ) tiếp xúc với đoạn
E A, E B và tiếp xúc trong với (O) . Đường tròn (L) tiếp xúc với đoạn F B, F C và tiếp xúc với
cung BC không chứa A của (O) . Chứng minh rằng đường tròn đường kính M N luôn đi qua
một điểm cố định khi P di chuyển.
Bài 3.3. Cho tam giác ABC và (O) cố định đi qua B, C . Gọi D là điểm di chuyển trên cung
BC của (O) sao cho D, A cùng phía với BC . DB, DC cắt C A, AB lần lượt tại E, F. Đường
tròn (K ) tiếp xúc với đoạn EA, EB tại M, N và tiếp xúc trong với (O) . Đường tròn (L) tiếp
xúc với đoạn F B, F C tại P, Q và tiếp xúc (O) tại một điểm không cùng phía A so với BC .
Chứng minh rằng giao điểm của M N và PQ luôn thuộc một đường tròn cố định.
Lời giải chi tiết của các bài toán trên có thể tham khảo trong [2].
Ý thứ hai của bài toán là một xây dựng thú vị khi đòi hỏi phải xây dựng thêm đường tròn
Mixtilinear bàng tiếp góc A của tam giác ABC . Tính chất sử dụng trong lời giải trên đã khá
quen thuộc và do đã chứng minh được định lý Sawayama và Thébault nên không cần thực
hiện lại chứng minh tính chất này. Một bài toán có liên quan đã xuất hiện cách đây khá lâu
trong đề chọn đội tuyển năm 1999 của Việt Nam:
Bài 5 VN TST 1999. Cho tam giác A1 A2 A3 nội tiếp trong đường tròn (O). Một đường tròn
(K 1 ) nằm trong góc A2 A1 A3 của tam giác A1 A2 A3 , tiếp xúc với các cạnh A1 A2 , A1 A3 và
tiếp xúc trong với đường tròn (O) lần lượt tại các điểm M1 , N1 , P1 . Các điểm M2 , N2 , P2 và
M3 , N3 , P3 xác định một cách tương tự. Chứng minh rằng các đoạn thẳng M1 N1 , M2 N2 , M3 N3
cắt nhau tại trung điểm mỗi đoạn.
Việt Nam Team Selection Test 2014 13

Bài 4.
Cho tam giác ABC nhọn, không cân có đường cao AD và P thuộc AD. Các đường
thẳng P B, PC lần lượt cắt C A, AB tại E, F.
PA A
a) Giả sử tứ giác AEDF nội tiếp. Chứng minh rằng = (tan B + tan C ) cot .
PD 2
b) Gọi C P cắt đường thẳng qua B vuông góc AB tại M. BP cắt đường thẳng qua
C vuông góc AC tại N. K là hình chiếu của A lên M N. Chứng minh rằng
∠BKC + ∠M AN không đổi khi P di chuyển trên AD.

Lời giải. a) Gọi Q là giao điểm của E F và BC thì hàng điểm (Q, D, B, C ) điều hòa. Từ đây
suy ra chùm (AQ, AD, AB, AC ) và (DE, DF, DA, DQ) cũng điều hòa.
Hơn nữa, do DA ⊥ DQ nên DA là phân giác trong của ∠EDF . Do tứ giác AEDF nội tiếp
đường tròn nên dễ thấy AE = AF .

F P

Q
B D G C

Giả sử đường tròn ngoại tiếp tứ giác AEDF cắt BC tại G khác D. Ta có DG là phân giác
ngoài tại đỉnh D nên GE = GF do đó 4AGE = 4AGF nên AG là phân giác ∠BAC . Theo
định lý Menelaus cho cát tuyến BP E của tam giác ADC và cát tuyến C P F của tam giác
ABD thì
EA PA BD FA PA C D
= · , = ·
EC P D BC F B P D BC
Từ đây suy ra
FA EA PA BD C D PA
 
+ = + = .
F B EC P D BC BC PD
Do đó, ta được

PA FA GF EA GE A A A
= . + . = cot tan B + cot . tan C = (tan B + tan C ) cot .
PD GF F B GE EC 2 2 2
Ta có đpcm.
Việt Nam Team Selection Test 2014 14

b) Gọi L là giao điểm của BM và C N thì rõ ràng điểm L cố định. Tứ giác AC K N có


∠AC N = ∠AK N = 90◦ nên nội tiếp. Tương tự, ABK M cũng nội tiếp. Do đó, ta có
∠M AN + ∠BKC = ∠M AK + ∠N AK + ∠BKC = ∠M BK + ∠NC K + ∠BKC = ∠BLC
không đổi. Ta có đpcm.
A

D
C
B

L
M

Nhận xét. Đây là bài toán ở mức độ dễ và thí sinh có thể xử lý một cách nhẹ nhàng với các
kiến thức quen thuộc về hình phẳng. Bài toán mang màu sắc đại số khá nhiều và nói chung
đòi hỏi thí sinh tính toán, biến đổi (tương tự như bài 3 trong đề IMO 2013) hơn là tiếp cận
theo hướng hình học thuần túy. Hai câu a) và b) của bài toán không liên quan đến nhau.
Ta sẽ tách riêng thành hai bài toán và phân tích từng bài toán một. Câu a) phát biểu điều
kiện dưới dạng một biểu thức lượng giác như vậy chưa đẹp, ta hoàn toàn có thể có một hệ
thức lượng thuần túy hình học của câu a). Ta xét bài toán sau:
Bài toán 4.1. Giả thiết tương tự như bài toán. Chứng minh rằng nếu tứ giác AE F D nội tiếp
AD AB · AC + AD 2
thì = .
PD DB.DC
Đây chính là dạng tổng quát của câu a của bài toán gốc và có thể xử lý dễ dàng bằng định
lý Van-Obel: Cho tam giác ABC có D thuộc BC và P thuộc đoạn AD. Tia BP cắt AC tại N,
AD AM AN
tia C P cắt AB tại M. Khi đó, ta có = + . Bên cạnh đó, lời giải nêu ở trên gợi
AP AB AC
cho ta nhớ đến bài toán trong đề IMO Shortlist 1994 với nội dung như sau:
Việt Nam Team Selection Test 2014 15

Bài toán 4.2. Cho tam giác ABC có đường phân giác AD. Gọi E, F là hình chiếu của D lên
các cạnh C A, C B. Giả sử BE giao C F tại P. Gọi H là hình chiếu của P lên BC . Chứng minh
rằng H P là phân giác ∠EH F.
Cuối cùng, ta các nội dung này, ta có thể phát biểu bài toán tổng quát như sau:
Bài toán 4.3. Cho tam giác ABC với P là điểm bất kỳ trong tam giác ABC . Các đường
thẳng PA, P B, PC cắt AB, BC , C A lần lượt tại D, E, F. Gọi H, K là hình chiếu của F, E lên
AD và L, N là hình chiếu của F, E lên BC . Giả sử tứ giác AEDF nội tiếp. Chứng minh rằng
PA F H DA E K DA
= · + · .
PD F L DB E N DC
Nói về phần b, ta thấy đây chỉ là một bài toán đẳng giác quen thuộc. Ta có thể thêm một số
ý vào bài toán này để vấn đề phong phú hơn:
Bài toán 4.4. Cho tam giác ABC đường cao AD. Điểm P di chuyển trên AD. Giả sử P B, PC
lần lượt cắt các đường thẳng qua C vuông góc với C A và qua B vuông góc với AB tại N, M.
Gọi K là hình chiếu của A lên M N.

a) Chứng minh rằng ∠M AN + ∠BKC không đổi khi P di chuyển.

b) Chứng minh rằng ∠M AC = ∠N AB.

c) Chứng minh rằng K A là phân giác ∠BKC .

Dưới đây là hai bài mở rộng theo con đường đẳng giác cho bài toán trên:
Bài toán 4.5. Cho tam giác ABC nội tiếp (O) . Các điểm E, F cố định thuộc (O) sao cho
E F k BC . Điểm P di chuyển trên AE. Giả sử P B, PC lần lượt cắt F C , F B tại N, M. và trung
trực BC cắt M N tại K . Chứng minh rằng ∠M AN + ∠BKC không đổi khi P di chuyển.
Bài toán 4.6. Cho tam giác ABC nội tiếp đường tròn (O) . E, F cố định thuộc (O) sao cho
E F k BC . Các điểm P, Q lần lượt thuộc AE, AF. Giả sử P B, PC lần lượt cắt QB, QC tại N, M.
Chứng minh rằng ∠M AB = ∠N AC .
Lời giải chi tiết của các bài toán trên có thể tham khảo trong [3].

Bài 5.
Tìm tất cả đa thức P(x ), Q(x ) có hệ số nguyên và thỏa mãn điều kiện: Với dãy số
(x n ) xác định bởi

x 0 = 2014, x 2n+1 = P(x 2n ), x 2n = Q(x 2n−1 ) với n ¾ 1.

thì mỗi số nguyên dương m là ước của một số hạng khác 0 nào đó của dãy (x n ).

Lời giải. Ta sẽ chứng minh rằng nếu các đa thức P(x ), Q(x ) thỏa mãn đề bài thì chúng
phải đều có bậc là 1. Thật vậy,
Xét trường hợp một trong hai đa thức P(x ), Q(x ) là đa thức hằng. (1) Nếu P(x ) ≡ a, ∀x ∈
Z thì (x n ) có dạng:
x 0 = 2014, x 1 = a, x 2 = Q(a), x 3 = a, · · ·
Việt Nam Team Selection Test 2014 16

và dễ thấy mọi số hạng của dãy chỉ nhận 1 trong 3 giá trị (2014, a, Q(a)).
(2) Nếu Q(x ) ≡ a, ∀x ∈ Z thì (x n ) có dạng

x 0 = 2014, x 1 = P(2014), x 2 = a, x 3 = P(a), · · ·

và dễ thấy mọi số hạng của dãy chỉ nhận một trong ba giá trị (2014, P(2014), a, P(a)).
Cả hai điều này đều không thỏa mãn điều kiện đề bài do mỗi số nguyên dương m phải là
ước của một số hạng khác 0 nào đó của dãy số x n .
Tiếp theo, nếu một trong hai đa thức P(x ), Q(x ) có bậc lớn hơn 1, không mất tính tổng
quát, ta giả sử đó là Q(x ). Rõ ràng khi đó Q(P(x )) cũng có bậc lớn hơn 1.
Ta thấy nếu R(x ) là đa thức có bậc lớn hơn 1 thì với mọi k > 0 lớn tùy ý, tồn tại x có giá trị
tuyệt đối đủ lớn sau cho |R(x )| > k |x | . Điều này là dễ thấy do khi x → +∞ thì ta có giới
hạn
|R(x )|
lim = +∞.
x →+∞ |x |

Ta sẽ chứng minh rằng tồn tại N đủ lớn sao cho |Q(P(x ))| > |P(x )| + |x | với mọi x > N.
Ta chỉ cần xét 2 trường hợp: Nếu P(x ) là bậc 1 thì dễ thấy tồn tại k đủ lớn sao cho
k |P(x )| > |x | với mọi x ∈ Z.
Suy ra, tồn tại N đủ lớn sao cho

|Q(P(x ))| > (k + 1) |P(x )| = |P(x )| + k |P(x )| > |P(x )| + |x | với mọix > N.

Nhận xét được chứng minh.


Theo giả thiết thì trong dãy số đã cho, phải tồn tại số hạng |x i | lớn tùy ý và rõ ràng, ta cũng
phải có j > 0 sao cho x 2 j > N + 1 và x 2 j có giá trị tuyệt đối lớn nhất trong 2 j số hạng
đầu tiên của dãy (x n ).

Thật vậy, ta thấy rằng tồn tại vô số số hạng x 2 j thỏa mãn điều kiện x 2 j = max {|x k |},
0¶k¶2 j
gọi T là tập hợp
các
chỉ số thỏa mãn. Nếu như trong các số hạng như thế, không có số hạng
nào thỏa mãn x 2 j > N + 1 thì với mọi t ∈ T , ta có |x i | ¶ |x t | ¶ N + 1 với mọi i ¶ t ∈ T .

Tuy nhiên, do |T | vô hạn nên điều giả sử ở trên là vô lý và nhận xét được chứng minh. Với
x 2 j là số hạng thỏa mãn điều kiện trên, chọn m = x 2 j+2 − x 2 j thì ta thấy


m = Q(P(x 2 j )) − x 2 j ¾ Q(P(x 2 j )) − x 2 j > P(x 2 j ) > x 2 j và

x 2n+2 = Q(x 2n+1 ) > x 2n+1 .

Do đó, trong 2 j + 1 số hạng đầu tiên của dãy, không có số hạng nào chia hết cho m.
. .
Mặt khác, x 2k+2 −x 2k = Q(P(x 2k )) − Q(P(x 2k−2 )) .. P(x 2k ) − P(x 2k−2 ) .. x 2k − x 2k−2 =

.€
m và tương tự thì x 2k+3 − x 2k+1 = P(x 2k+2 ) − P(x 2k ) .. x 2 j − x 2 j−2 = m. Từ đây suy ra
Š

với k ¾ j thì x 2k+2 − x 2k và x 2k+3 − x 2k+1 đều chia hết cho m, tuy nhiên x 2 j+1 và x 2 j +2 đều
không chia hết cho m nên x k không chia hết cho m với k > 2 j + 2. Do đó, trong dãy đã
cho, không có số hạng nào chia hết cho m.
Việt Nam Team Selection Test 2014 17

Điều mâu thuẫn này cho ta thấy nhận xét ban đầu là đúng và deg P(x ) = deg Q(x ) = 1.
Đặt P(x ) = ax + b với |a| > 1 và Q(x ) = cx + d với a, b, c, d ∈ Z và ab 6= 0 thì ta có

x 2n+1 = ax 2n + b
¨
với mọi n ¾ 0.
x 2n+2 = cx 2n+1 + d

Suy ra x 2n+2 = cax 2n + bc + d và x 2n+3 = cax 2n+1 + ad + b với mọi n ¾ 0.


Cả hai dãy này đều có công thức truy hồi dạng yn+1 = k yn + h với k = ac, h ∈ Z. Giả sử
k n −1
k 6= 1 thì công thức tổng quát của dãy này là yn = k n y0 + h k−1 với mọi n. Rõ ràng nếu
k = −1 thì dãy số tương ứng không thỏa mãn, ta xét k 6= −1.

• Nếu h = 0 thì ta có yn = k n y0 , rõ ràng không thỏa mãn điều kiện.


‚ Œ
kn − 1
• Nếu h 6= 0 thì do gcd k, = 1 với mọi n nên giả sử giả sử t là số mũ lớn
k −1
nhất mà k t |h thì các số nguyên dương có dạng k s với s > t đều không là ước của
bất cứ số hạng nào của dãy, không thỏa mãn. Từ đây, suy ra k = 1 hay ac = 1.

Cuối cùng, ta chỉ cần xét 2 trường hợp:

• Nếu P(x ) = x + a và Q(x ) = x + b với a, b ∈ Z thì bằng quy nạp, ta chứng minh được
x 2k = 2014 + k(a + b) và x 2k+1 = 2014 + a + k(a + b). Dễ thấy rằng nếu a + b = 0
thì dãy này không thỏa mãn. Nếu như a + b 6= 0 thì gọi S, T, R lần lượt là tập hợp các
ước nguyên của a + b, 2014 và 2014 + a. Ta xét các trường hợp sau:

+ Với m ∈/ S thì giả sử a + b chia m dư t với t 6= 0; do đó, khi k chạy qua một hệ
thặng dư đầy đủ modulo m thì tồn tại một số hạng của dãy chia hết cho m và
số lượng các số hạng như thế là vô hạn. Rõ ràng số các số hạng bằng 0 của dãy
là hữu hạn (không quá 2) nên tồn tại số hạng khác 0 của dãy chia hết cho m.
/ T, R thì dãy số tương ứng không thỏa mãn.
+ Với m ∈ S mà m ∈
+ Với m ∈ S và m ∈ T hoặc m ∈ R thì tương ứng, mọi số hạng có chỉ số chẵn
hoặc mọi số hạng có chỉ số lẻ của dãy đều chia hết cho m và dễ thấy, tồn
tại số hạng khác 0 của dãy chia hết cho m. Do đó, các số a, b phải thỏa mãn
(S\T ) ∩ (S\R) = ∅ hay mỗi ước của a + b phải là ước của 2014 hoặc là ước của
2014 + a.

• Nếu P(x ) = −x + a và Q(x ) = −x + b thì cũng có lập luận tương tự vì các số hạng
của dãy tương ứng khi đó là x 2k = 2014 − k(a − b) và x 2k+1 = −2014 + a + k(a − b).
Điều kiện của a, b là a − b 6= 0 và mỗi ước của a − b phải là ước của 2014 hoặc là
ước của a − 2014.

Vậy tất cả các đa thức P(x ), Q(x ) cần tìm là

• P(x ) = x + a, Q(x ) = x + b trong đó a, b ∈ Z thỏa mãn a + b 6= 0 và mỗi ước của


a + b phải là ước của 2014 hoặc là ước của 2014 + a.

• P(x ) = −x + a, Q(x ) = −x + b trong đó a, b ∈ Z thỏa mãn a − b 6= 0 và mỗi ước


của a − b phải là ước của 2014 hoặc là ước của −2014 + a.
Việt Nam Team Selection Test 2014 18

Nhận xét.
Đây là bài toán liên quan đến các tính chất số học của đa thức. Trong các bài toán này,
có tính chất cơ bản sau đây khá hữu dụng: Nếu P(x ) là đa thức có hệ số nguyên thì
(a − b)| (P(a) − P(b)) với a 6= b. Học sinh cũng phải vận dụng tốt tính đơn điệu của đa
thức khi biến đủ lớn và một chút khéo léo trong việc xử lí các kĩ thuật ở những vấn đề nhỏ.
Bài toán yêu cầu sự cẩn thận, chi tiết; suy luận một cách logic, khoa học, có thứ tự hợp lý.
Có thể nói đây là một bài toán hay, mới lạ và có nhiều “bẫy” trong đó.
Thoạt nhìn vào bài toán này, ta có thể thấy không dễ hình dung ra mô hình và cách tiếp
cận vì có tới 2 đa thức không liên quan đến nhau xuất hiện. Một suy nghĩ tự nhiên là nên
giải thử với trường hợp P, Q trùng nhau.
Khi đó ta có bài toán chỉ với một đa thức và cách tiếp cận cũng dễ nhìn ra hơn, thậm chí
các học sinh có thể xử lý tốt hơn nếu đã có biết về bài toán sau:
(Arthur Angel, Problem Solving Strategies) Cho đa thức P(x ) với hệ số nguyên thỏa mãn
điều kiện P(n) > n với mọi số nguyên dương n. Xét

x 1 = 1, x 2 = P(x 1 ), x 3 = P(x 2 ), · · ·

Biết rằng với mọi số nguyên dương N, tồn tại một số hạng của dãy số chia hết cho N. Chứng
minh rằng P(n) = n + 1.
Ta có được x n+2 − x n+1 chia hết cho x n+1 − x n với mọi n, ta suy ra các phần tử của dãy số
x n+1 , x n+2 , x n+3 ... khi chia cho x n+1 − x n có cùng số dư.
Cùng với tính tăng của đa thức, ta sẽ thấy nếu x n+1 “quá lớn” so với x n thì đa thức tương
ứng sẽ không thể thỏa mãn điều kiện bài toán. Cách tiếp cận này chắc hẳn sẽ mang lại một
cái nhìn vừa bao quát, vừa đơn giản hơn cho bài số 5.
Lời giải trên đã cố gắng làm sáng tỏ tất cả các vấn đề đặt ra trong bài toán nên tương đối
dài. Ta thấy rằng tư tưởng chính là loại trường hợp bậc 0 của các đa thức để đưa về xét 2
trường hợp quan trọng:

• Ít nhất một trong hai đa thức có bậc lớn hơn 1.

• Bậc của cả hai đa thức đều bằng 1.

Tuy nhiên, ta cũng cần hiểu nguyên nhân tại sao cần phải chia ra như thế, tức là tại sao lập
luận ở trường hợp (1) lại không áp dụng được cho trường hợp (2)? Câu trả lời như sau:
cách lập luận ở (1) đòi hỏi cần có cả hai bất đẳng thức sau

|Q(P(x )) − x | > |x | và |Q(P(x )) − x | > |P(x )|

đều phải đúng (*). Khi đó, dùng tính chất đơn điệu của đa thức để chỉ ra một số nguyên
dương m không là ước của số hạng khác 0 nào của dãy và dẫn đến mâu thuẫn.
Từ việc xác định rõ bản chất của vấn đề ở đây, ta có thể xử lý bài toán theo một cách ngắn
gọn hơn (đưa về trường hợp hệ số của cả hai đa thức đều có giá trị tuyệt đối là (1) như sau:
Việt Nam Team Selection Test 2014 19

Áp dụng lập luận tương tự trường hợp (1), ta có thể giải quyết trường hợp (2) như sau:
Nếu hệ số cao nhất của một trong hai đa thức P(x ), Q(x ) có giá trị tuyệt đối lớn hơn hoặc
bằng 3 thì tính chất (*) vẫn đúng.
Nếu hệ số cao nhất của cả hai đa thức P(x ), Q(x ) đều không vượt quá 2 thì ta xét hiệu
x n+4 − x n và lập luận tương tự.
Các bạn thử từ ý tưởng trên để tìm ra một cách lập luận ngắn gọn hơn cho bài toán trên!
Về một hướng mở rộng, chúng ta thử xem xét bài toán trong trường hợp thay 2 đa thức bởi
3 đa thức như sau:
Tìm tất cả đa thức P(x ), Q(x ), R(x ) có hệ số nguyên và thỏa mãn điều kiện: Với dãy số
(x n ) xác định bởi x 0 = 2014, x 3n+1 = P(x 3n ), x 3n+2 = Q(x 3n+1 ), x 3n+3 = R(x 3n+2 ), n ¾ 0,
thì mỗi số nguyên dương m là ước của một số hạng khác 0 nào đó của dãy (x n ).
Các bạn thử xem xét đặc điểm chung cơ bản cũng như cái khó của bài toán mới này hoặc
bài toán tổng quát khi thay 2, 3 bởi k ¾ 4 đa thức so với bài toán gốc là gì nhé!

Bài 6.
Cho m, n, p là các số tự nhiên không đồng thời bằng 0. Không gian tọa độ được
chia thành các mặt phẳng song song cách đều nhau. Một cách điền vào mỗi khối lập
phương đơn vị một trong các số từ 1 đến 60 được gọi là cách điền Điện Biên nếu
thỏa mãn: trong mỗi hình hộp chữ nhật với các mặt trên hệ mặt đã cho và tập hợp
kích thước ba cạnh (số hình lập phương trên trên cạnh) xuất phát từ một đỉnh là
2m + 1, 2n + 1, 2p + 1, khối lập phương đơn vị có tâm trùng với tâm của hình hộp
chữ nhật được điền số bằng trung bình cộng của các số điền ở tâm của 8 hình lập
phương ở các góc của hình hộp đó. Hỏi có tất cả bao nhiêu cách điền Điện Biên?
Những cách điền là giống nhau nếu trong mỗi cách, ta chọn một hình lập phương
làm tâm thì các số được điền vào các khối lập phương đơn vị có cùng vị trí tương đối
với tâm trong hai cách điền là bằng nhau

Lời giải. Không mất tính tổng quát, giả sử các mặt phẳng song song trong hệ mặt cách đều
nhau một khoảng là 1 đơn vị. Ta chọn một hình lập phương nào đó rồi lấy tâm O của nó
làm tâm của không gian vuông góc, các trục Ox , O y, Oz song song với các đường thẳng
của hệ đã cho và chiều dương các trục tọa độ chọn tùy ý. Khi đó, ta quy ước tọa độ của
một hình lập phương trong hệ mặt đã cho là khoảng cách từ hình chiếu của nó lên các trục
O x , O y, Oz đến O (chọn dấu tùy vào chiều âm hay chiều dương của các trục tọa độ).
Xét một cách điền có tính chất Điện Biên nào đó và ta gọi hai hình lập phương phải được
điền cùng một số trong cách điền này là có quan hệ với nhau. Để đếm số cách điền Điện
Biên, ta đếm số lớn nhất các hình lập phương đơn vị có thể được chọn sao cho các hình đôi
một không có quan hệ với nhau, đặt S là số lượng hình này.
Trước hết, ta sẽ chứng minh rằng với mỗi hình hộp chữ nhật có kích thước các cạnh là
2m + 1, 2n + 1, 2p + 1 thì 8 số điền ở góc và số điền ở tâm là bằng nhau.
Thật vậy, Do các số được dùng để điền là nguyên dương nên phải tồn tại một số nhỏ nhất,
giả sử là a và hình lập phương A nào đó được điền số a này. Khi đó, hình hộp chữ nhật có
Việt Nam Team Selection Test 2014 20

kích thước 2m + 1, 2n + 1, 2p + 1 và các hình có A là tâm sẽ có 8 số điền ở 8 ô ở đỉnh đều


không nhỏ hơn a mà lại có trung bình cộng bằng a. Suy ra 8 số điền ở các đỉnh và số điền
ở tâm của hình hộp chữ nhật này là giống nhau. Lập luận tương tự, ta có thể mở rộng ra
trong không gian và được một tập hợp T các hình lập phương điền số a.
Với các hình lập phương không thuộc T , giả sử b > a là số nhỏ nhất được điền cho các hình
lập phương còn lại và tương tự trên, ta được một tập hợp T 0 gồm các hình lập phương điền
số b. Rõ ràng cứ như thế, ta thấy nhận xét trên được chứng minh và đây cũng chính là điều
kiện cần để một cách điền là có tính chất Điện Biên.
Như thế, hình nằm ở tọa độ (x , y, z) sẽ có quan hệ với hình nằm ở tọa độ
(x + (−1) r m, y + (−1) s n, z + (−1) t p).
Các số r, s , t ∈ {0; 1} được chọn một cách độc lập với nhau (là 8 đỉnh của hình hộp chữ
nhật có tâm là (x , y, z) và các cạnh là 2m + 1, 2n + 1, 2p + 1). Hơn nữa, ở đây ta xét
hình hộp chữ nhật có cạnh 2m + 1, 2n + 1, 2p + 1 tương ứng song song với các mặt phẳng
O y z, O zx , Ox y và ta hoàn toàn có thể hoán đổi thứ tự này.
Từ đây suy ra, hình lập phương ở tọa độ (x , y, z) sẽ được điền số giống với hình  lập phương
ở tọa độ x + a1 m + a2 n + a3 p, y + b1 m + b2 n + b3 p, z + c1 m + c2 n + c3 p . Tuy nhiên,
với hai hình có quan hệ với nhau, khi xét sự thay đổi ở các thành phần x , y, z tương ứng
thì các đại lượng m, n, p sẽ hoặc tăng, hoặc giảm chứ không giữ nguyên và mỗi lần như
thế thì tính chẵn lẻ của a1 + a2 + a3 , b1 + b2 + b3 , c1 + c2 + c3 đồng thời thay đổi. Ban đầu,
xét ở vị trí (x , y, z) thì ba tổng này cùng tính chẵn lẻ (cùng bằng 0) nên tính chẵn lẻ của
a1 + a2 + a3 , b1 + b2 + b3 , c1 + c2 + c3 là luôn giống nhau.
Một hệ quả của nhận xét trên là: hình lập phương ở (x , y, z) sẽ có quan hệ với hình lập
phương ở tọa độ (x + x 1 , y + y1 , z + z 1 ) với (x 1 , y1 , z1 ) là một hoán vị của bộ các số
(2k m, 2kn, 2k p) và k ∈ Z, d ∈ m, n, p . (∗)
Hơn nữa, theo định lý Bezout thì trong các tổ hợp tuyến tính của m, n, p, ta có d =
gcd(m, n, p) > 0 chính là tổ hợp tuyến tính có giá trị tuyệt đối nhỏ nhất. Từ đây suy
ra các hình lập phương đơn vị trong một hình lập phương lớn hơn có kích thước d × d × d
sẽ đôi một không có quan hệ với nhau (vì hai hình lập phương đơn vị bất kỳ trong hình này
sẽ có chênh lệch tọa độ nhỏ hơn d). Từ đó suy ra S là bội của d 3 .
Khi đó, ta thấy với mỗi hình lập phương d × d × d tùy ý, đặt tên là A, thì có 6 hình lập
phương có chung mặt với nó như hình vẽ dưới đây:
Việt Nam Team Selection Test 2014 21

m n p
Đặt m 1 = , n1 = , p1 = , ta xét các trường hợp sau:
d d d
• Nếu cả ba số m 1 , n1 , p1 đều lẻ, đặt m 1 = 2m 2 + 1, n1 = 2n2 + 1, p1 = 2p2 + 1 thì rõ
ràng m = 2dm 2 + d, n = 2dn2 + d, p = 2d p2 + d.

Do đó, theo (∗), hình lập phương ở tọa độ (m, n, p) sẽ có quan hệ với hình ở các tọa
độ (m ± d, n ± d, p ± d) (các hình kề đỉnh với A). Trong mô hình trên, nếu ta chọn
hình A và 3 hình kề mặt với nó, mỗi chiều một hình thì 4 hình này đôi một không
có quan hệ với nhau và không thể chọn số lượng nào lớn hơn 4 được. Từ đó, ta có
S = 4d 3 .

• Nếu trong ba số m 1 , n1 , p1 có 2 số lẻ và 1 số chẵn, ta có thể giả sử m 1 , n1 lẻ và p1


chẵn. Ta lại đặt m 1 = 2m 2 +1, n1 = 2n2 +1, p1 = 2p2 thì rõ ràng m = 2dm 2 +d, n =
2dn2 + d, p = 2d p2 .

Do đó, theo (∗), hình lập phương ở tọa độ (m, n, p) sẽ có quan hệ với hình ở các tọa
độ (m ± d, n ± d, p) (các hình kề cạnh với A). Ta lại hoán đổi vị trí của m, n, p, tức
là thay đổi chiều của (x , y, z) thì suy ra thêm (m, n ± d, p ± d) và (m ± d, n, p ± d)
có quan hệ với A. Trong mô hình trên, nếu ta chọn hình A và 1 hình kề mặt với nó thì
hai hình này không có quan hệ với nhau và không thể chọn số lượng nào lớn hơn 2
được. Từ đó, ta có S = 2d 3 .

• Nếu trong ba số m 1 , n1 , p1 có 1 số lẻ và 2 số chẵn, ta có thể giả sử m 1 lẻ và n1 , p1


chẵn. Ta lại đặt m 1 = 2m 2 + 1, n1 = 2n2 , p1 = 2p2 thì rõ ràng m = 2dm 2 + d, n =
2dn2 , p = 2d p2 .

Do đó, theo (∗), hình lập phương ở tọa độ (m, n, p) sẽ có quan hệ với hình ở các tọa
độ (m ± d, n, p) (các hình kề mặt với A). Ta lại hoán đổi vị trí của m, n, p, tức là thay
đổi chiều của (x , y, z) thì suy ra thêm (m, n ± d, p) và (m, n, p ± d) có quan hệ với
A. Trong mô hình trên, ta chỉ chọn được hình A chứ không chọn được thêm hình nào
không có quan hệ với nó. Từ đó, ta có S = d 3 .

Cuối cùng, ta thấy mỗi ô độc lập với nhau sẽ có 60 cách điền số (từ 1 đến 60) cho nó
nên số cách điền Điện Biên tổng cộng là 60S với S được xác định như sau: Trong ba số
m n p
, , , nếu có
gcd(m, n, p) gcd(m, n, p) gcd(m, n, p)

• 3 số lẻ thì S = 4d 3 .

• 2 số lẻ thì S = 2d 3 .

• 1 số lẻ thì S = d 3 .

Nhận xét.
Con số 60 và thuật ngữ Điện Biên trong bài toán có lẽ nhắc chúng ta về năm 2014 là kỷ
niệm 60 năm chiến thắng Điện Biên Phủ (07/05/1954 – 07/05/2014). Tuy nhiên, các bạn
Việt Nam Team Selection Test 2014 22

thí sinh chắc cũng nhanh chóng nhận ra rằng số 60 không có nhiều ảnh hưởng trong việc
đi tìm lời giải.
Bài toán này khá lạ và khó. Có thể nói khi xử lý các bài toán đếm tổ hợp thì công việc đầu
tiên chính là đoán đáp số và nếu đoán thành công thì nhiều khi mọi thứ sẽ dễ dàng hơn.
Điểm bất ngờ nhất ở đây lại chính là đáp số khi mà trên thực tế, có nhiều đáp số chưa
hoàn toàn chính xác đã được đưa ra. Điều này có lẽ xuất phát từ việc xử lí hai điều kiện khi
chuyển về dạng số học cuối cùng không cẩn thận. Nếu thực sự bình tĩnh, và có thời gian, ta
có thể xét thử vài ví dụ nhỏ với m, n, p để thấy có các kết quả “tương đối khác nhau” chứ
không cùng một dạng.
Tuy nhiên, ở trong phòng thi, cùng với việc bài 5 làm mất không ít thời gian, đây thực sự là
một thử thách không dễ.
Đây là một bài toán hay, có nhiều đất cho các bạn học sinh thể hiện khả năng. Bài toán đòi
hỏi những tư duy đơn giản, nhưng tinh tế ở phần suy luận tổ hợp, và sự cẩn thận, khoa học
trong phần xử lí điều kiện số học cuối cùng. Nếu chưa có một hình dung tốt về không gian
3 chiều, ta hoàn toàn có thể chọn cách tiếp cận từ việc, đưa bài toán về 2 ( xét mặt phẳng
và hình chữ nhật 2 cạnh lẻ) hay thậm chí 1 chiều ( đường thẳng và đoạn thẳng chứa lẻ
điểm). Cụ thể như dưới đây:
Cho k, a là các số nguyên dương. Trên trục số nguyên, người ta đánh số các điểm nguyên
bằng các số từ 1 đến k sao cho: Mỗi đoạn thẳng có hai đầu mút nguyên và có chứa đúng
2a + 1 điểm nguyên thì trung điểm của nó được đánh số bằng trung bình cộng số đánh ở
hai đầu mút. Hỏi có tất cả bao nhiêu cách đánh số khác nhau?
Bằng lập luận theo nguyên lý cực hạn tương tự như lời giải bài toán gốc, ta thấy rằng:

• Với các đoạn thẳng có chứa đúng 2a + 1 điểm nguyên thì số được đánh cho 2 đầu
mút bằng với số đánh cho trung điểm.

• Các điểm cách nhau ít hơn a đơn vị độ dài thì không có ràng buộc với nhau. Như thế
a điểm có tọa độ từ 0, 1, 2, ..., a − 1 sẽ đôi một đánh số độc lập với nhau và kết quả
của bài toán sẽ là k a .

Tiếp tục phát triển lên, ta xét bài toán sau:


Cho k, a, b là các số nguyên dương. Trên trục số nguyên, người ta đánh số các điểm nguyên
bằng các số từ 1 đến k sao cho: Mỗi đoạn thẳng có hai đầu mút nguyên và có chứa đúng
2a + 1 hoặc 2b + 1 điểm nguyên thì trung điểm của nó được đánh số bằng trung bình cộng
số đánh ở hai đầu mút. Hỏi có tất cả bao nhiêu cách đánh số khác nhau?
Tiếp tục lập luận như bài trên, ta thấy các vị trí 0, a, 2a, b, 2b được đánh cùng một số hay
nói cách khác, các vị trí cách nhau a hoặc b đơn vị thì được đánh cùng một số. Từ đó suy ra
tất cả các điểm ở vị trí x a + y b với x , y ∈ Z và x , y không đồng thời bằng 0 sẽ được đánh
cùng một số với nhau.
Theo định lý Bézout thì trong các số có dạng x a + y b thì c = gcd(a, b) > 0 có giá trị tuyệt
đối nhỏ nhất. Vì thế nên các số cách nhau một khoảng c đơn vị thì được điền cùng một số
và các số cách nhau một khoảng nhỏ hơn c đơn vị thì không có ràng buộc gì với nhau. Sau
bước này, rõ ràng ta đã chỉ được số cách tô màu là k gcd(a,b) .
Việt Nam Team Selection Test 2014 23

Bằng cách tương tự, việc thay 2 số a, b thành nhiều số vẫn được giải quyết tương tự và bài
toán luôn có một kết quả theo dạng lũy thừa trên. Tuy nhiên, nếu thay bởi bài toán 2 chiều,
ta lại có đến 2 kết quả: Cho k, a, b là các số nguyên dương. Trong mặt phẳng tọa độ, người
ta đánh số các điểm nguyên bằng các số từ 1 đến k sao cho: Mỗi hình chữ nhật có các cạnh
song song với 2 trục tọa độ và trên mỗi cạnh có 2a + 1, 2b + 1 điểm nguyên thì trung bình
cộng 4 số điền ở đỉnh bằng số điền ở tâm. Hỏi có tất cả bao nhiêu cách đánh số khác nhau?

a b
Kết quả của bài toán là: Đặt c = gcd(a, b), nếu trong hai số , có:
c c
2
• 2 số lẻ thì kết quả là k 2c .
2
• 1 số lẻ thì kết quả là k c .

Ở trên, ta đã lập luận dựa trên định lý Bézout về ước chung lớn nhất – là một định lý cơ
bản trong số học. Ta cũng có thể lập luận dựa theo định lý Sylvester về biểu diễn các số
cũng cho kết quả tương tự. Điều này tương ứng với kết quả trong trường hợp 3 chiều của
bài toán gốc. Như thế ta thấy rằng số lượng số tăng lên không ảnh hưởng đến mức độ khó
của bài toán mà chính số chiều mới là vấn đề, nếu tiếp tục xét với số chiều tăng lên là n,
liệu có n đáp số hay không? Câu trả lời xin dành cho các bạn.
Việt Nam Team Selection Test 2014 24

Tài liệu tham khảo

1. Topic đề thi và bình luận đề VN TST 2014 của diễn đàn mathscope.org

http://forum.mathscope.org/showthread.php?t=46988

2. Trần Quang Hùng, Xung quanh bài hình học trong kỳ thi chọn đội tuyển VN ngày 1

http://analgeomatica.blogspot.com/2014/03/xung-quanh-mot-bai-toan-hinh-hoc-trong.html

3. Trần Quang Hùng, Xung quanh bài hình học trong kỳ thi chọn đội tuyển VN ngày 2

http://analgeomatica.blogspot.com/2014/03/tom-tat.html

4. Nguyễn Thị Hường, Lương Ánh Nguyệt, Lương Thị Thanh Mai, Đào Thị Quỳnh
Nga, Định lý Sawayama và Thébault

http://analgeomatica.blogspot.com/2014/02/inh-ly-sawayama-va-thebault.html

5. Functional equation APMO 1989

http://www.artofproblemsolving.com/Forum/viewtopic.php?p=450331

6. Bình luận của GS Nguyễn Tiến Dũng trên trang Sputnik Education

https://www.facebook.com/sputnikedu/posts/720502957971422

7. Bài toán IMO Shortlist 1994, G4

http://www.artofproblemsolving.com/Forum/viewtopic.php?p=352892

8. Mẫu Latex của thầy Châu Ngọc Hùng

https://www.writelatex.com/read/htndbgqrjqzp

LATEX by Lê Phúc Lữ. Email: lephuclu@gmail.com


LỜI GIẢI VÀ BÌNH LUẬN ĐỀ THI
CHỌN ĐỘI TUYỂN IMO 2017
Trần Nam Dũng – Võ Quốc Bá Cẩn – Trần Quang Hùng
Lê Phúc Lữ - Nguyễn Tất Thu

1. Lời nói đầu

f
af
Dù Epsilon đã nói lời tạm biệt với bạn đọc từ ngày 13/2/2017 nhưng tinh thần Epsilon và đội ngũ
Epsilon thì vẫn còn. Và có nghĩa là những sản phẩm mang tinh thần Epsilon vẫn sẽ còn được ra
đời. Tinh thần đó ngắn gọn là: Chuyên nghiệp – Từ cộng đồng – Vì cộng đồng.
Minh chứng cho tinh thần đó là tài liệu mà các bạn đang đọc “Giải và bình luận đề thi chọn
st
đội tuyển Việt Nam dự thi Toán Quốc tế 2017”, một đóng góp của đội ngũ Epsilon dành cho
cộng đồng. Khi viết đội ngũ Epsilon, chúng tôi không chỉ muốn nhắc đến các người lính ngự
lâm thuộc Ban biên tập (Epsilon staff) mà còn là những người đã luôn sát cánh cùng chúng
tôi trong suốt hơn 2 năm qua trong quá trình xây dựng Epsilon thành một niềm yêu mến và
n
sự chờ đợi của cộng đồng.
Giải và bình luận đề thi, chúng tôi không chỉ muốn đem lại cho độc giả lời giải, đáp án để so
ilo

khớp đúng sai mà hơn thế là những phân tích về hướng tiếp cận, về nguồn gốc, về lớp các bài
toán tương tự. Chúng tôi cũng mạn phép đưa ra những bình luận chủ quan của mình về cái hay,
cái dở, độ khó dễ, tính phù hợp, độ mới cũ của bài toán ngõ hầu giúp cho các thầy cô trong ban
ra đề có thêm những ý kiến phản biện, để công tác đề thi ngày càng tốt hơn, chất lượng hơn.
s

Hy vọng tập tài liệu này sẽ nhận được sự đón nhận của cộng đồng. Chúng tôi luôn lắng nghe
những ý kiến đóng góp, trao đổi thẳng thắn của bạn đọc về nội dung tài liệu cũng như các vấn đề
Ep

liên quan. Chúng ta là một cộng đồng.


“If you want to go far, go together.”

2. Thông tin bản quyền


Bản quyền thuộc về tất cả các thành viên trong nhóm biên soạn (Trần Nam Dũng, Võ Quốc Bá
Cẩn, Trần Quang Hùng, Lê Phúc Lữ, Nguyễn Tất Thu).
Đây là thành quả của quá trình lao động miệt mài của nhóm để chia sẻ đến cộng đồng. Mọi người
đều có thể xem tài liệu MIỄN PHÍ. Tuy nhiên, vui lòng ghi rõ nguồn khi chia sẻ.
Tất cả các hoạt động mua bán, kinh doanh liên quan đến tài liệu này mà không được sự chấp
thuận của nhóm là trái pháp luật. Chúng ta hãy lên án những hành vi vi phạm bản quyền để bảo
vệ quyền lợi của các tác giả, của những sản phẩm trí tuệ. Xin cảm ơn.

1
2 Lời giải và bình luận đề thi chọn đội tuyển IMO 2017

3. Đề thi
3.1. Ngày thi thứ nhất (25/03/2017)
Bài 1 (7.0 điểm). Cho 44 cái lỗ phân biệt trên một cái rãnh là đường thẳng và 2017 con kiến.
Mỗi con kiến sẽ chui lên từ một cái lỗ và bò đến một cái lỗ khác với vận tốc không đổi rồi chui
xuống đó. Gọi T là tập các thời điểm mà con kiến chui lên hoặc chui xuống các cái lỗ. Biết rằng
vận tốc của các con kiến đôi một khác nhau và jT j  45: Chứng minh rằng tồn tại ít nhất hai
con kiến nào đó không gặp nhau. (Quy ước hai con kiến gặp nhau khi và chỉ khi tồn tại một thời
điểm mà cả hai ở cùng một vi trí trên rãnh kể cả lúc chui lên và xuống.)
n
Bài 2 (7.0 điểm). Với mỗi số nguyên dương n, đặt xn D C2n :

f
2017k
a) Chứng minh rằng nếu 2
< n < 2017 k với k là số nguyên dương nào đó thì x n

af
là bội của 2017:

b) Tìm tất cả số nguyên dương h > 1 để tồn tại các số nguyên dương N ; T sao cho với mọi
n > N thì x n là dãy số tuần hoàn theo modulo h với chu kỳ T :

st
Bài 3 (7.0 điểm). Cho tam giác AB C ngoại tiếp đường tròn .I / và .I / tiếp xúc với các cạnh
B C ; C A ; AB lần lượt tại D ; E ; F : Gọi I b ; I c lần lượt là các tâm đường tròn bàng tiếp góc
B ; C của tam giác AB C : Gọi P ; Q lần lượt là trung điểm của I b E ; I c F : Giả sử .PAC /
cắt A B tại điểm thứ hai R và .QAB / cắt AC tại điểm thứ hai S :
n
a) Chứng minh rằng P R ; QS ; AI đồng quy.
ilo

b) Giả sử DE ; DF lần lượt cắt I b I c tại K ; J : E J cắt F K tại M và P E ; QF cắt


. P A C /; .QAB / lần lượt tại các điểm thứ hai X ; Y : Chứng minh rằng ba đường thẳng
B Y ; C X ; AM đồng quy.
s

3.2. Ngày thi thứ hai (26/03/2017)


Ep

Bài 4 (7.0 điểm). Cho tam giác AB C nội tiếp đường tròn .O /: Điểm A di động trên .O /
sao cho A B > B C và M là trung điểm của AC : Đường tròn đường kính B M cắt .O / tại
R : Giả sử R M cắt .O / tại điểm thứ hai Q ; cắt B C tại P : Đường tròn đường kính BP cắt
A B ; B O lần lượt tại các điểm thứ hai K ; S :

a) Chứng minh rằng SR đi qua trung điểm của KP :

b) Gọi N là trung điểm của B C : Trục đẳng phương của hai đường tròn đường kính
A N ; B M cắt SR tại E : Chứng minh rằng M E luôn đi qua một điểm cố định.

Bài 5 (7.0 điểm). Cho 2017 số thực dương a 1 ; a 2 ; : : : ; a 2 0 1 7 : Với mỗi n > 2017; ta đặt
˚
a n D max a i 1 a i 2 a i 3 j i 1 C i 2 C i 3 D n ; 1  i 1  i 2  i 3  n 1 :

Chứng minh rằng tồn tại số nguyên dương m không vượt quá 2017 và số nguyên dương
N > 4 m sao cho a n a n 4 m D a n2 2 m với mọi n > N :
Lời giải và bình luận đề thi chọn đội tuyển IMO 2017 3

Bài 6 (7.0 điểm). Với mỗi số nguyên dương n, xét a 1 ; a 2 ; : : : ; a 2 n là hoán vị của 2 n số
nguyên dương đầu tiên. Một hoán vị như thế được gọi là “đẹp” nếu với mọi 1  i < j  2 n
thì a i C a nC i D 2 n C 1 và a i a i C 1 không đồng dư với a j a j C 1 theo modulo 2 n C 1.
(Quy ước a 2 nC 1 D a 1 :)

a) Với n D 6, hãy chỉ ra một hoán vị đẹp.

b) Chứng minh rằng với mỗi n nguyên dương thì luôn tồn tại một hoán vị đẹp.

4. Bình luận chung


Trong hai ngày 25, 26/3/2017 đã diễn ra kỳ thi chọn đội tuyển Việt Nam dự thi Toán Quốc tế năm
2017 với sự tham dự của 49 học sinh xuất sắc nhất đến từ các tỉnh thành và các trường chuyên

f
thuộc các trường đại học lớn trong cả nước. Mỗi ngày thi, các thí sinh phải giải quyết 3 bài toán

af
trong thời gian 240 phút.
Đề thi năm nay được đánh giá là khá khó và có phong cách gần với đề thi IMO hơn các năm
trước. Cấu trúc đề thi bao gồm 2 bài hình học, 1 bài đại số, 1 bài tổ hợp, 1 bài số học và 1 bài tổ
hợp số học. Như vậy đề chọn đội tuyển năm nay cũng theo xu hướng của IMO là thiên về tổ hợp
st
nhiều. Ngay cả bài 5 (bài đại số) cũng có cách phát biểu (và cả cách giải) có nhiều nét tổ hợp.
Các bài toán 2 và 4 theo đánh giá chung được coi là bài dễ nhất của mỗi ngày. Bài hình câu 4
phần a) cũng chỉ cần các kiến thức chương trình cấp 2, phần b) vận dụng các kiến thức trục
đẳng phương, ở ý này điểm cố định có đặc biệt hơn so với các bài thông thường trước đây.
n
Bài số 2 là một bài toán số học thuần tuý, khai thác một chủ đề khá quen thuộc đối với học
sinh Việt Nam là số dư trong phép chia hệ số C nk cho số nguyên tố p với các định lý Lucas,
ilo

Wolstenhome, Babbage hay công thức Legendre, vì vậy, dù ở các mức độ khác nhau, các thí
sinh đều có thể tiếp cận được bài này.
Đáng chú ý là bài số 1, bài được ban đề thi đánh giá là dễ nhất của ngày thứ nhất lại gây nhiều
khó khăn cho các thí sinh. Các em đã không thể phát biểu lại bài toán một cách rành mạch bằng
ngôn ngữ Toán học nên đã không thể xử lý được, không những thế còn để tốn nhiều thời gian
s

vào bài toán này. Nếu biết cách chuyển đổi bài toán (mô hình hoá) thành đồ thị (theo nghĩa đồ
thị) hoặc đồ thị (theo nghĩa các đường biểu diễn đường đi con kiến) thì bài toán có thể giải quyết
Ep

một cách khá đơn giản, chẳng hạn bằng quy nạp.
Các bài toán còn lại gồm bài số 3, 5 và 6 được đánh giá là khó. Bài hình số 3 phần a) sử dụng
kiến thức tam giác đồng dạng và tứ giác nội tiếp của chương trình cấp 2, 3 ý khó tập trung ở câu
b) đòi hỏi vận dụng nhiều kiến thức tổng hợp có liên quan nhiều đến bài hình G7 trong IMO
shortlist 2002 trong đó có sử dụng cả kiến thức về hàng điều hòa. Cấu trúc là ghép nối các bài
toán riêng lẻ thành một bài tổng hợp. Bài số 5 là một bài toán có cách phát biểu rất tổ hợp, dạng
dãy số truy hồi với các số hạng đầu tiên bất kỳ. Để tiếp cận được bài này, học sinh phải tỉnh táo
nhận ra rằng hằng số 2017 trong đề bài có thể thay bằng một số nguyên dương bất kỳ và bắt đầu
làm thử với các trường hợp tham số nhỏ để dự đoán quy luật của dãy số. Cũng có thể thấy rằng
bằng cách logarith hoá, các phép nhân trong bài toán này sẽ biến thành phép cộng và bài toán có
thể đưa về một biến thể của bài toán số 6 trong đề thi IMO 2010. Điều này một lần nữa nhấn
mạnh độ khó của bài số 5, đồng thời cũng là một điểm chưa hay của đề thi lần này.
Bài toán số 6 là một bài toán số học-tổ hợp với yêu cầu xây dựng một hoán vị của f1; 2 ; : : : ; 2 ng
thoả mãn một điều kiện số học. Với bài toán này, ngoài trường hợp n D 6 có thể xây dựng ví dụ
4 Lời giải và bình luận đề thi chọn đội tuyển IMO 2017

bằng phép thử sai, có thể xây dựng ví dụ tổng quát cho một số trường hợp. Chẳng hạn nếu 2 n C 1
có căn nguyên thuỷ thì đặt a i D ˛ i .mod 2 n C 1/ thì rõ ràng các hiệu a i C 1 a i  ˛ i .˛ 1/
phân biệt modulo 2 n C 1 và a i C a nC i  ˛ i .1 C ˛ n /  0 .mod 2 n C 1/: Trong trường
hợp tổng quát, ý tưởng chính là xây dựng n số hạng đầu tiên của dãy số, còn đoạn sau lấy phần
bù. Tuy nhiên, cần có điều kiện bổ sung đối với a n và a 1 ; vì a nC 1 D 2 n C 1 a 1 : Và đây
cũng chính là điểm mà các thí sinh có thể phạm sai lầm do ngộ nhận.
Với những nhận định trên đây cũng như tham khảo một số thông tin về bài làm của các thí sinh,
chúng tôi cho rằng bên cạnh 2 bài toán 2 và 4 thuộc dạng bài “cần phải làm được nếu muốn đậu
đội tuyển” thì chiếc vé dự IMO sẽ được quyết định ở 4 bài còn lại ở các bài 1, 3, 5, 6 với tình
trạng rất khó đoán. Khác với một số năm trước khi có những bài toán quá khó không ai làm
được, đề thi năm nay độ khó được chia đều, khó dễ thuỳ theo sở trường của từng em. Thực tế là
hầu như bài nào cũng có học sinh giải được tạo thành các thế “cài răng lược”, ví dụ có học sinh
làm được bài 1, 6 lại không làm được 3, 5, có học sinh làm được bài 5 lại không làm được bài 1.

f
Ngoài ra các bài 1, 5, 6 đều là các bài có thể có lỗi trong trình bày, thậm chí có những sai sót

af
lớn do ngộ nhận. Với đề thi như thế này, nếu làm chắc chắn 4 bài thì gần như sẽ có vé, còn điểm
chuẩn vùng tranh chấp sẽ là 3.5++.

5. Lời giải và bình luận các bài toán


st
Bài 1 (7.0 điểm). Cho 44 cái lỗ phân biệt trên một cái rãnh là đường thẳng và 2 01 7 con
kiến. Mỗi con kiến sẽ chui lên từ một cái lỗ và bò đến một cái lỗ khác với vận tốc không đổi
n
rồi chui xuống đó. Gọi T là tập các thời điểm mà con kiến chui lên hoặc chui xuống các cái
lỗ. Biết rằng vận tốc của các con kiến đôi một khác nhau và j T j  4 5: Chứng minh rằng
ilo

tồn tại ít nhất hai con kiến nào đó không gặp nhau. (Quy ước hai con kiến gặp nhau khi và chỉ
khi tồn tại một thời điểm mà cả hai ở cùng một vi trí trên rãnh kể cả lúc chui lên và xuống.)

Lời giải. Ta xét trục toạ độ O x y với O x là trục đánh dấu vị trí của các lỗ trên rãnh thẳng còn
O y là trục thời gian. Các lỗ có hoành độ là x 1 ; x 2 ; : : : ; x 4 4 và các thời điểm mà các con
s

kiến chui lên hoặc chui xuống là y 1 ; y 2 ; : : : ; y 4 5 : Một con kiến sẽ đi từ điểm .x a ; y b / đến
Ep

điểm . x c ; y d / tương ứng với tình huống nó chui lên từ lỗ xa vào thời điểm y b và đến lỗ x c vào
thời điểm y d : Do kiến bò với vận tốc không đổi nên đồ thị biểu diễn đường đi theo thời gian
chính là đoạn thẳng nối hai điểm này. Có tất cả 2017 đoạn thẳng như vậy. Vì vận tốc các con
kiến đôi một khác nhau nên các đoạn thẳng đều có phương khác nhau. Ta cần chứng minh có ít
nhất hai đoạn thẳng trong số này không cắt nhau.
Chú ý là số điểm là đầu mút của các đoạn thẳng là 44  45 D 1980 < 2017 nên bài toán sẽ
được giải quyết hoàn toàn nếu ta chứng minh được mệnh đề sau: Nếu có n điểm trên mặt phẳng
thì không tạo được quá n đoạn thẳng với các đỉnh là các điểm đó sao cho đôi một có điểm chung
và không có hai đoạn nào song song hay đè lên nhau.
Ta chứng minh bằng quy nạp theo n : Với n D 2 ; 3 mệnh đề là hiển nhiên. Giả sử mệnh đề đã
đúng với n 1 điểm. Xét n điểm:

 Nếu trong n điểm này có một điểm với không quá một đoạn thẳng xuất phát từ điểm đó thì
ta xoá điểm đó (và đoạn thẳng xuất phát từ đó nếu có), ta đưa về trường hợp n 1 điểm.
Lời giải và bình luận đề thi chọn đội tuyển IMO 2017 5

 Giả sử bây giờ đỉnh nào cũng có ít nhất hai đoạn xuất phát. Nếu có đỉnh A có ba đoạn xuất
phát thì sẽ “có vấn đề”: chẳng hạn là AB ; AC ; AD: Nếu B C D chứa A bên trong thì
đoạn thứ hai từ B không thể cùng cắt AC và AD: Nếu B C D không chứa A bên trong
thì ta có chẳng hạn tia AC nằm giữa hai tia AB và AD: Khi đó đoạn thứ hai từ đỉnh C
không thể cùng cắt AB và AD: Như vậy từ mỗi đỉnh chỉ có đúng hai đoạn, suy ra số
đoạn bằng đúng số đỉnh trong trường hợp này.

Bài toán được giải quyết hoàn toàn.

Bình luận.

 Đây là một bài toán có cách phát biểu rất thú vị có lời giải không phức tạp nhưng thực ra
rất rối nếu không nhìn thấy được đúng bản chất và chuyển đổi được về mô hình điểm, đoạn
thẳng như ở trên, với hai chú ý quan trọng: các con kiến bò với vận tốc không đổi (các

f
đường biểu diễn quỹ đạo con kiến là các đoạn thẳng) và các vận tốc đôi một khác nhau
(các đoạn thẳng không cùng phương). Trên thực tế, rất nhiều bạn đã tốn nhiều thời gian

af
cho bài toán này nhưng vẫn không tiếp cận được lời giải. Và nếu trong lời giải, hai điều
kiện quan trọng này không được khai thác thì chắc chắn lời giải là không hoàn chỉnh.

 Lời giải trình bày trên đây là của GS Nguyễn Tiến Dũng, HCV IMO 1985 được chúng tôi

khác của GS Nguyễn Tiến Dũng:


st
trình bày lại cho giống một lời giải. Còn đây là phần dẫn nhập và một cách diễn đạt lời giải

Có gì liên quan giữa các số 44 ; 45 và 2017? Dễ thấy 44  45 D 1980 < 2017; nên
n
có thể đoán đây là mấu chốt bài toán?

Vậy tại sao lại dùng tích? Bởi vì có nhiều nhất là từng đó điểm .vị trí, thời gian chạm
ilo

lỗ khi lên hoặc xuống lỗ/ trên mặt phẳng toạ độ. Ta vẽ trên mặt phẳng toạ độ các đồ thị
đường đi của các con kiến, mỗi đồ thị là một đoạn thẳng nối hai trong số các điểm trên.
Tất cả các đoạn thẳng đó đều có hướng khác nhau .vì vận tốc các con kiến khác nhau/:
Hướng ở đây hiểu là góc so với đường nằm ngang modulo  :
s

Câu hỏi đặt ra bây giờ là nếu hai đoạn một đều có điểm chung (hai con kiến nào cũng có
gặp nhau) thì có nhiều nhất là bao nhiêu đoạn thẳng?
Ep

Ta có thể sắp xếp thứ tự các đoạn thẳng theo thứ tự vòng tròn theo góc của chúng.

Cố định một đoạn đầu tiên, từ điểm A 1 đến điểm A 2 : Đoạn thứ hai có hướng quay về
“bên phải” so với đoạn thứ nhất, nên phải có thêm ít nhất một điểm mới A 3 .tức là hoặc
là A 1 A 3 với A 3 nằm bên phải A 1 A 2 ; hoặc A 3 A 2 với A 3 nằm bên trái A 1 A 2 ; hoặc
A 3 A 4 .hai điểm mới/ nằm ở hai bên của A 1 A 2 /: Thêm đoạn thứ ba phải thêm ít nhất
một điểm mới, trừ trường hợp tạo thành ba đoạn A 1 A 2 ; A 1 A 3 ; A 2 A 3 : Cứ như thế:
thêm một đoạn thì cần thêm ít nhất một điểm mới, trừ khi thêm đoạn cuối cùng thì dùng
được hai điểm cũ. Như vậy để có n đoạn đôi một có điểm chung thì cần ít nhất n điểm. Với
n D 44 lần 45 thì có nhiều nhất là từng đó con kiến đôi một có gặp nhau. Vì 2017 lớn
hơn 44 lần 45 nên có hai con kiến không gặp nhau.

 Mô hình hoá, phát biểu lại bài toán bằng ngôn ngữ Toán học là một kỹ năng quan trọng để
có thể giải được bài toán, đặc biệt là các bài toán thực tiễn hoặc sử dụng ngôn ngữ thực
tiễn. Đây lại là điểm yếu của học sinh Việt Nam, vì chúng ta thường tập trung giải quyết
6 Lời giải và bình luận đề thi chọn đội tuyển IMO 2017

các bài toán đầu cuối, tức là các bài toán đã được phát biểu rành mạch bằng ngôn ngữ Toán
học mà lại ít chú trọng đến các bài toán đòi hỏi khá năng đọc hiểu, phát hiện bản chất vấn
đề và phát biểu lại bằng mô hình Toán học.

 Chúng tôi cung cấp cho bạn đọc một số bài toán mà lời giải đòi hỏi bước đọc hiểu, phát
biểu lại bằng mô hình Toán học để tham khảo:

1. (IMO Shortlist, 2010) Trong một buổi biểu diễn ca nhạc, có 20 ca sĩ sẽ trình diễn.
Với mỗi ca sĩ có một danh sách (có thể là rỗng) các ca sĩ khác mà anh ấy hoặc cô ấy
muốn diễn sau tất cả các ca sĩ thuộc danh sách đó. Có thể xảy ra là có đúng 20 10
cách sắp xếp để tất cả các mong muốn của các ca sĩ đều được thoả mãn?
2. (Tournament of the towns, 1992-1993) Trong từ điển thực vật mỗi một loài cây được
đặc trưng bởi 100 dấu hiệu (mỗi một dấu hiệu có thể có, có thể không có). Hai loài

f
cây được coi là không giống nhau nếu chúng khác nhau ở ít nhất 51 dấu hiệu.
a) Chứng minh rằng trong từ điển có không quá 50 loài cây đôi một không giống nhau.

af
b) Có thể có đúng 50 loài cây đôi một không giống nhau không?
3. (Moscow MO, 2010) Trên mặt phẳng ta đánh dấu 4 n điểm, sau đó ta nối tất cả các
cặp điểm có khoảng cách đúng bằng 1 cm. Người ta thấy răng trong n C 1 điểm bất
st
kỳ luôn có hai điểm được nối. Chứng minh rằng có ít nhất 7n được kẻ.
4. Ban đầu có một số lẻ viên sỏi. Ở bước tiếp theo, ta chia số sỏi làm hai, một nửa để
xuống phía dưới, bên trái, một nửa để xuống phía dưới, bên phải, còn dư một viên để
ngay xuống dưới vị trí đống sỏi ban đầu. Ở mỗi lượt tiếp theo, ta lại làm như vậy với
n
các đống sỏi. Như vậy ở một số vị trí ở bước tiếp theo sẽ có thể được nhận sỏi từ ba
đống sỏi trên nó: ở ngay phía trên, phía trên bên trái và phía trên bên phải.
ilo

Ví dụ, nếu bắt đầu bằng đống sỏi 21 viên thì ở lượt chia đầu tiên ta sẽ có ba đống sỏi

10 1 10:

Ở lượt chia thứ hai ta sẽ có


s

5 0 11 0 5:
Ep

Và ở lượt chia thứ ba sẽ là

2 1 7 1 7 1 2:

Chứng minh rằng nếu cứ tiếp tục chia như thế thì bắt đầu từ một đống sỏi có số lẻ
viên, sẽ đến một lúc ta đi đến trạng thái có các đống sỏi một viên liền nhau.

Bài 2 (7.0 điểm). Với mỗi số nguyên dương n, đặt x n D C 2nn :


2017k
a) Chứng minh rằng nếu 2
< n < 2 01 7 k với k là số nguyên dương nào đó thì
x n là bội của 2017:

b) Tìm tất cả số nguyên dương h > 1 để tồn tại các số nguyên dương N ; T sao cho với
mọi n > N thì x n là dãy số tuần hoàn theo modulo h với chu kỳ T :
Lời giải và bình luận đề thi chọn đội tuyển IMO 2017 7

Lời giải. a) Ta chứng minh kết luận bài toán đúng cho mọi số nguyên tố p lẻ thay cho số 2017:
k
Giả sử tồn tại số nguyên dương k sao cho p2 < n < p k : Ta có

v p .x n / D v p C 2nn D v p .2 n/Š
 
2v p .nŠ / :

pk
Do 2
< n < p k nên p k < 2 n < 2p k < p k C 1 ; từ đó suy ra
     
 2n 2n 2n
v p .2 nŠ/ D C CC :
p p2 pk
1
Mặt khác, với mọi x 2 R ; ta có b 2x c  2b x c; đẳng thức xảy ra khi fx g < 2
: Kết hợp với
k
giả thiết p2 < n < p k ; ta có

f
     
 n n n
v p .2 n/Š > 2 C CC D 2v p .nŠ / ;
p2 pk

af
p
:
hay v p . x n / > 0 nên ta có x n :: p :
b) Ở ý này, ta có hai cách tiếp cận sau.
st
Cách 1. (Dựa theo lời giải của Hà Duy Hưng) Giả sử h > 1 là số thoả mãn yêu cầu bài toán.
Với mỗi p nguyên tố lẻ mà p j h, ta có dãy số dư x n theo modulo p cũng tuần hoàn. Sử dụng
k
kết quả câu a) thì với p2 < n < p k thì
n
xn  0 .mod p /:
ilo

k
Chọn k đủ lớn để p2 > T C 1, ta suy ra tất cả các số dư của x n cho p đều bằng 0 với mọi
n  n 0 , trong đó n 0 2 Z C đủ lớn. Tuy nhiên chọn t 2 Z C đủ lớn để p t 1 > 2 n 0 và đặt
t
n D p 2 1 ta có ngay v p .x n / D 0, do đó x n không chia hết cho p ; vô lý.
Vậy h chỉ có ước nguyên tố là 2 hay h D 2 k với k nguyên dương. Nếu k > 1; ta chọn
r D k 1 và xét số n có dạng n D 2 a 1 C    C 2 a r ; trong đó a 1 > maxfT ; N g trong đó
s

T ; N là các hằng số trong giả thiết của h: Khi đó


Ep

v 2 .x n / D 2S 2 .n/ S 2 .2 n/ D r;

trong đó S 2 .x / là tổng các chữ số trong biểu diễn nhị phân. Thành thử x n  2 k 1 .mod h/.
Tuy nhiên, với mọi i 2 Z C mà i < 2 a 1 thì số chữ số 1 trong biểu diễn nhị phân của n C i
tăng thêm ít nhất 1 đơn vị, do đó mà x nC i  0 .mod h/. Do a 1 > maxfT ; N g nên
x n  x nC T  0 .mod h/, mâu thuẫn.
Vậy k D 1 do đó h D 2. Đây là đáp số bài toán, vì dễ thấy rằng x n là số chẵn với mọi số
nguyên dương n ; như sau: nếu n D 2 a 1 C    C 2 a r với 0  a 1 <    < a r thì

2 n D 2 a1 C1 C    C 2 ar C1

nên
v 2 .x n / D 2S n .n/ S 2 .2 n/ D r  1:
Do đó x n chẵn.
8 Lời giải và bình luận đề thi chọn đội tuyển IMO 2017

Cách 2. (Theo lời giải của Nguyễn Song Minh) Do h > 1; nên giả sử p là ước nguyên tố
lớn nhất của h: Giả sử T D p m T 0 là chu kỳ nhỏ nhất ứng với n đủ lớn, với . T 0 ; p / D 1:
Theo định lý Lucas, ta có
m m . nC T /p0 m 0
C 2nn  C 2pp mnn  C 2pp mnC T nC T
nC 2 T  C . 2 nC 2 T 0 / p m  C 2 n C 2 T 0 .mod p /:

Suy ra m D 0: Nếu p lẻ và giả sử T D . a 1 a 2    a r / p thì với R > r đủ lớn, ta có


R R
C 21  C 2pp R  C 2pp RCCT2 T  C 21 :C 2TT .mod p /:

Suy ra C 2TT  1 .mod p /, từ đây ta có với K đủ lớn thì

C 2KKTT  1 .mod p /:

f
af
Mặt khác do .T ; p / D 1 nên tồn tại số nguyên dương K đủ lớn sao cho p j .K T C 1/. Kết
hợp với . K T C 1/ j C 2KKTT ; ta thu được 1  0 . mod p / điều này vô lý, do đó h D 2 k :
Vì T lẻ và theo lý luận trên, nếu ta chọn K sao cho h j .n C K T C 1/ thì

C 2nn  C 2nC KT
nC2K T  0 st .mod h/;

với n đủ lớn. Ta chọn n D 2 N , ta có số các số lẻ trong dãy C nk .0  k  n/ bằng 2 nên


n
C 2nn  2 .mod 4/;

do đó h D 2 : Hơn nữa C 2nn luôn là số chẵn với mỗi số nguyên dương n : Vậy h D 2 :
ilo

Bình luận.

 Một lần nữa chủ đề về tính chất số học của hệ số nhị thức C nk lại được khai thác. Đó là
điều có thể dự đoán được trong các năm nguyên tố (như 2003 hay trước đó nhiều năm là
s

1979) nhưng không ngờ năm nay điều này lại được ưu ái đến hai lần (VMO bài số 6 và
bài số 2 của kỳ TST này). Vì vậy, với các công cụ được trang bị đầy đủ như công thức
Ep

Legendre, định lý Lucas, Lagrange, Babbage hay Wolstenhome thì bài toán này, đặc biệt là
câu a) sẽ không gây khó khăn cho các bạn học sinh.
ap
Nhắc lại ở đây định lý Babbage là đồng nhất thức C bp  C ba .mod p 2 /; đúng cho
mọi số nguyên tố p ; còn định lý Wolstenhome ở dạng hệ số nhị thức là đồng nhất thức
ap
C bp  C ba .mod p 3 / chỉ đúng cho số nguyên tố p > 3:

 Sự kiện C 2nn luôn chẵn là một tính chất quen thuộc và có rất nhiều cách chứng minh. Hơn
thế ta có số mũ của n trong C 2nn bằng số số 1 trong biểu diễn nhị phân của n ; cho nên với
h D 2 s thì chỉ có h D 2 là thoả mãn điều kiện.

 Chúng tôi dẫn một số bài toán thi Olympic những năm gần đây có sử dụng nhóm định lý này.

1. (Vietnam TST, 2010) Gọi S n là tổng bình phương các hệ số sau khai triển của
.1 C x / n : Chứng minh rằng 1 C S 2 n không chia hết cho 3:
Lời giải và bình luận đề thi chọn đội tuyển IMO 2017 9

2. (Saudi Arabia TST, 2015) Với mỗi số nguyên dương n ; đặt


n
X
S .n/ D rk ;
k D0

trong đó r k là số dư trong phép chia C nk cho 3: Tìm tất cả các số nguyên dương
n sao cho S .n/  n :
3. (Trường Xuân Toán học miền Nam, 2017) Cho dãy số .x n / xác định bởi công thức:
 
x0 C x1 C    C xn 1
x 0 D 1; x n D 2017 với 1  n  2017:
n

Hỏi có bao nhiêu chỉ số i mà 0  i  2017 sao cho jx i j chia hết cho 3?

f
af
Bài 3 (7.0 điểm). Cho tam giác AB C ngoại tiếp đường tròn .I / và . I / tiếp xúc với các
cạnh B C ; C A; AB lần lượt tại D ; E ; F : Gọi I b ; I c lần lượt là các tâm đường tròn bàng
tiếp góc B ; C của tam giác AB C : Gọi P ; Q lần lượt là trung điểm của I b E ; I c F : Giả

a) Chứng minh rằng P R ; QS ; AI đồng quy.


st
sử . P A C / cắt AB tại điểm thứ hai R và . QAB / cắt AC tại điểm thứ hai S :

b) Giả sử DE ; DF lần lượt cắt I b I c tại K ; J : E J cắt F K tại M và P E ; QF cắt


. P AC /; .QAB / lần lượt tại các điểm thứ hai X ; Y : Chứng minh rằng ba đường
n
thẳng B Y ; C X ; AM đồng quy.
ilo

Lời giải. Ta sẽ dùng các bổ đề sau để giải bài toán này.


Bổ đề 1. Cho tam giác AB C có đường tròn nội tiếp .I / tiếp xúc với các cạnh B C ; C A; AB
lần lượt tại D ; E ; F : Gọi M ; N lần lượt là trung điểm của DE ; DF : Gọi P là giao điểm
của các đường trung trực của M C và N B :
s

a) Chứng minh rằng đường tròn .P B C / tiếp xúc với đường tròn .I /.
Ep

b) Gọi J là điểm đối xứng với D qua P : Chứng minh rằng J là tâm đường tròn bàng tiếp
góc A của tam giác AB C .

c) Chứng minh rằng giao điểm của P M và AC nằm trên đường tròn .P B C /.

Chứng minh. a) Gọi G là giao điểm thứ hai của P D và .I /: Dễ thấy trung trực C M là trục
đẳng phương của đường tròn điểm C và đường tròn .I /; và trung trực B N là trục đẳng phương
của đường tròn điểm B và đường tròn .I /: Do đó

P C 2 D PD  P G D PB 2:

Từ đây suy ra ∠ P G C D ∠ P C B và ∠ P GB D ∠ P B C nên tứ giác P B G C nội tiếp đường


ID IG
tròn . K /. Lại có I D k KP và KP D K G
nên ba điểm G ; I ; K thẳng hàng. Do đó, .K /
tiếp xúc với .I / tại G .
10 Lời giải và bình luận đề thi chọn đội tuyển IMO 2017

Q
G
E
R

F I
N M
K
D
B C

f
af
P

st
n
J
b) Do GB là đường đối trung của tam giác GF D nên
lo

∠ F GN D ∠ B GD D ∠P B C

si

∠ GF N D ∠ GD C D ∠ P DB :
Từ đó suy ra hai tam giác GF N và BDP đồng dạng g-g. Lại có N ; P là trung điểm của
DF ; D J nên hai tam giác GF D và BD J đồng dạng c-g-c. Từ đó suy ra
Ep

∠ DB J D ∠ D GF D ∠ BDF
nên B J k DF : Vậy B J là phân giác ngoài góc B :
Tương tự, ta cũng có C J là phân giác ngoài góc C nên J là tâm đường tròn bàng tiếp góc A.
c) Dễ thấy tứ giác B C M N nội tiếp và tâm nội tiếp chính là giao điểm P của trung trực C M ;
B N : Từ đó suy ra
P M 2 D P C 2 D P D  P G:
Suy ra hai tam giác P DM và P M G đồng dạng. Từ đó
∠ DM P D ∠ D GM D ∠ E G C :
Lại có
∠ GE C D 180 ı ∠ GE A D 180 ı ∠GDE D ∠ P DM D ∠ P M G:
Từ đó hai tam giác GM P và GE C đồng dạng nên P M và AC cắt nhau trên .K /.
Lời giải và bình luận đề thi chọn đội tuyển IMO 2017 11

Bổ đề 2. Cho tam giác AB C : Đường tròn nội tiếp .I ; r / tiếp xúc với các cạnh B C ; C A; AB
lần lượt tại D ; E ; F : Một đường tròn qua B ; C và tiếp xúc .I / tại X : Khi đó, ta có

XE  XF r2
a) D :
XD 2 IB  I C
XE I B  DE 2
b) D :
XF I C  DF 2

Chứng minh. Gọi giao điểm của tiếp tuyến chung của hai đường tròn tiếp xúc tại X và B C là
M . Gọi G là điểm đối xứng của D qua M thì M G 2 D M D 2 D M X 2 D M B  M C nên
. B C ; D G / D 1; do đó EF đi qua G: Gọi AL là đường cao của tam giác AB C thì theo
bài toán quen thuộc thì I G ? AD nên hai tam giác I GD và DAL đồng dạng.

f
A

af
Z

F
st X
H
K E

I
n
ilo

G M B L D C
s
Ep

Lại có DX ? I M và M là trung điểm của GD nên DX đi qua trung điểm AL : Gọi DK là


đường kính của .I / thì chùm D .AL ; XK / D 1, chiếu lên .I / dẫn tới tứ giác DK ZX
điều hòa với Z là giao điểm của AD và .I /. Từ đó suy ra tiếp tuyến tại Z ; D và XK đồng
quy tại G . Nói cách khác, XK đi qua G . Gọi H là giao điểm của AD và EF : Ta có

XE K E GE HE ŒAED  DE DB I B
 D D D D   :
XF K F GF HF ŒAF D  DF I C D C
Lại có K E  I C D 2 r 2 D K F  I B nên

XE I B  DE 2
D :
XF I C  DF 2
Mặt khác, ta cũng có
F X EX GX GD 2 XD 2
 D D D
F K EK GK GK 2 4r 2
12 Lời giải và bình luận đề thi chọn đội tuyển IMO 2017

và K E  I C D 2 r 2 D K F  I B nên
XE  XF r2
D :
XD 2 IB  I C
Bổ đề được chứng minh.
Từ bổ đề trên, ta suy ra hai hệ quả là

XE 2 r 2  DE 2
D
XD 2 I B 2  DF 2

XF 2 r 2  DF 2
D :
XD 2 I C 2  DE 2

f
Bổ đề 3. Cho tam giác AB C ; đường tròn nội tiếp .I / tiếp xúc vớiB C tại D. Dựng một

af
đường tròn qua A; B tiếp xúc với .I / tại M và một đường tròn qua A; C tiếp xúc với .I / tại
N : B M cắt C N tại P : Chứng minh rằng ∠PAB D ∠ DAC :

Chứng minh. Gọi các tiếp điểm của .I / với C A; AB là E ; F và X Y Z là tam giác Ceva

là đường đối trung của tam giác M DF ; ta có


YC
D
ŒY B C 
D
st
của P . Gọi K là giao điểm của B Y và DF : Sử dụng bổ đề và hệ quả trên với chú ý rằng M K

ŒY B C  ŒK BD  ŒK BF 
 
YA ŒY BA ŒK BD  ŒK BF  ŒY BA
n
B C  B Y K D BF  BK
D  
BD  BK K F B Y  BA
ilo

B C MD2 BC r 2  DF 2
D  D  :
BA M F 2 BA EF 2  I A 2
Tương tự, ta cũng có
ZB BC r 2  DE 2
D  :
ZA C A EF 2  I A 2
s
Ep

F
I
Y
Z P M
N
K
B D X C
Lời giải và bình luận đề thi chọn đội tuyển IMO 2017 13

Sử dụng định lý Ceva, ta có

XB Y A ZB AB :DE 2
D  D :
XC Y C ZA AC :DF 2
Do đó
DB XB p b AB  DE 2 DF  I B AB  DE 2 AB 2
 D  D  D :
DC XC p c AC  DF 2 DE  I C AC  DF 2 AC 2

Từ đó suy ra AD và AX đẳng giác trong ∠BAC :


Trở lại bài toán:
a) Theo bổ đề 1 phần a), b), ta thấy các đường tròn .PAC / và .QBA/ tiếp xúc với .I /:
Từ đó, tiếp tục theo bổ đề 1 phần c) thì P R ; QS cùng đi qua trung điểm N của EF và hiển

f
nhiên M nằm trên AI :

af
Ib

K
A
st P
n
Ic
M
ilo

Q E
N

S
F
R I
s

X Z
Ep

Y
D
B C

b) Cũng theo bổ đề 1 phần a), ta thấy X ; Y lần lượt là tiếp điểm của các đường tròn .P C A/;
. QA B / với .I /. Gọi Z là giao điểm của B Y và C X : Theo bổ đề 3, ta thấy

∠ DAB D ∠ Z AC :

Ta sẽ chứng minh ∠ DAB D ∠M AC và từ đó suy ra A; M ; Z thẳng hàng. Thật vậy, ta dễ


thấy ∠ E A J D 90 ı 12 ∠ BAC D ∠ EDF nên tứ giác K AED nội tiếp.
Chứng minh tương tự, tứ giác JAF D nội tiếp. Từ đó dễ thấy ∠ E K A D ∠JDA D ∠ J F A
nên A J E M nội tiếp. Tương tự, tứ giác AK F M nội tiếp nên theo định lý Miquel, M nằm trên
. I /. Từ đó suy ra ∠ M AJ D ∠ M ED D ∠DAK : Như vậy, hai tia AM và AD đẳng giác
trong ∠ BAC . Ta hoàn thành chứng minh.
14 Lời giải và bình luận đề thi chọn đội tuyển IMO 2017

Bình luận.

 Hai ý a), b) của bài toán này cũng không liên quan tới nhau. Ý a) của bài toán giúp ta tìm
được trung điểm N của EF . Ý a) là hệ quả trực tiếp của bổ đề 1. Nội dung chính của bổ
đề 1 đã có trong [1] và thực chất bổ đề này có liên quan chặt chẽ tới bài toán IMO ShortList
2002, G7 xem [2], [3]. Phần khó nhất của ý b) là cần chứng minh ∠ Z AC D ∠ DAB ,
thực chất ý chứng minh này đã có trong [4] từ khá lâu và được chứng minh chặt chẽ như
trong trong bổ đề 3. Để chứng minh ý này có thể dùng phép nghịch đảo như trong [4]. Bạn
đọc có thể tham khảo một mở rộng của bài toán ý b) như sau, tham khảo [4].

 Bài toán mở rộng. Cho tam giác AB C nội tiếp trong đường tròn .O /. K là một điểm
nằm trên phân giác ∠ BAC và .K / là một đường tròn tiếp xúc trong với .O / tại L.
E ; F là các điểm thuộc .K / sao cho C E ; DF là các tiếp tuyến của .K /; trong đó

f
E ; F và A khác phía với B C . Đường tròn ngoại tiếp các tam giác AC E và ABF cắt
nhau tại P khác A. Chứng minh rằng ∠ PAC D ∠ LAB :

af
 Tài liệu tham khảo:

[1] Two circles are tangent,

[2] IMO Shortlist 2002, G7,


st
https://artofproblemsolving.com/community/c6h1299041.

https://artofproblemsolving.com/community/c6h17323p118682.
n
[3] http://analgeomatica.blogspot.com/2016/01/ve-bai-toan-g7-trong-
imo-shortlist-2002.html.
ilo

[4] Passes through tangent point,


https://artofproblemsolving.com/community/q1h1180223.

Bài 4 (7.0 điểm). Cho tam giác ABC nội tiếp đường tròn .O/: Điểm A di động trên .O/
sao cho AB > BC và M là trung điểm của AC: Đường tròn đường kính BM cắt .O/ tại
s

R: Giả sử RM cắt .O/ tại điểm thứ hai Q; cắt BC tại P: Đường tròn đường kính BP cắt
Ep

AB; BO lần lượt tại các điểm thứ hai K; S:

a) Chứng minh rằng SR đi qua trung điểm của KP:

b) Gọi N là trung điểm của BC: Trục đẳng phương của hai đường tròn đường kính
AN; BM cắt SR tại E: Chứng minh rằng ME luôn đi qua một điểm cố định.

Lời giải. a) Dễ thấy BQ là đường kính của .O/. Gọi I là giao điểm của SR và PK: Ta thấy

∠SPI D ∠SBK D ∠QCA


∠PSI D ∠PBR D ∠CQR:
Từ đó suy ra hai tam giác PSI và CQM đồng dạng g-g. Chứng minh tương tự, ta cũng có hai
tam giác KSI và AQM đồng dạng g-g, mà M là trung điểm AC nên I là trung điểm PK:
Lời giải và bình luận đề thi chọn đội tuyển IMO 2017 15

T M
E O X

K
S
L

f
I H

af
P
B N D C

R
st
b) Định nghĩa lại điểm E là chân đường cao hạ từ C của tam giác ABC: Gọi H là giao
n
điểm của các đường cao AD; BL của tam giác ABC: Dễ thấy tứ giác LMND nội tiếp nên
CM  CL D CN  CD và HA  HD D HB  HL; suy ra CH chính là trục đẳng phương của
đường tròn đường kính AN và BM .
ilo

Ta có E nằm trên CH nên thuộc trục đẳng phương của đường tròn đường kính AN và BM . Do
∠BEH D ∠BRP D 90ı nên tứ giác BHER nội tiếp, ta suy ra

∠HRE D ∠EBH D ∠OBC D ∠PBS D ∠PRS


s

nên ba điểm S; R; E thẳng hàng. Từ đó suy ra E là giao điểm của SR và trục đẳng phương của
các đường tròn đường kính AN; BM:
Ep

Gọi X là giao điểm của EM và BQ: Gọi T là trung điểm của BC thì dễ thấy tứ giác ETML
nội tiếp. Từ đó suy ra

∠MEC D 90ı ∠MET D 90ı ∠ALT D 90ı ∠BAC D ∠BAL D ∠BCQ:

Kết quả này chứng tỏ tứ giác BCXE nội tiếp, suy ra ∠BXC D BEC D 90ı : Vậy X là hình
chiếu của C lên BQ cố định. Tóm lại, ta có EM đi qua X cố định.

Bình luận.

 Hai ý của bài toán không liên quan tới nhau. Ý b) việc phát biểu để có điểm E chưa thực
sự đẹp vì bản chất E chỉ là chân đường cao hạ từ C; tuy nhiên điểm cố định cho lần này có
mới hơn các điểm cố định thông thường. Bài toán sử dụng các kiến thức về tam giác đồng
dạng và tứ giác nội tiếp thông thường. Ý b) cần vận dụng một số kiến thức về phương tích
và trục đẳng phương.
16 Lời giải và bình luận đề thi chọn đội tuyển IMO 2017

 Bạn đọc có thể tham khảo một phát triển của bài toán ý b) như sau: Cho tam giác ABC
cố định và một đường tròn .K/ thay đổi đi qua B; C cắt CA; AB lần lượt tại E; F . Gọi
J; L lần lượt là tâm ngoại tiếp các tam giác ABE và ACF: EJ cắt FL tại P: Q là tâm
ngoại tiếp tam giác KEF: Chứng minh rằng đường thẳng PQ luôn đi qua một điểm cố
định khi .K/ thay đổi.

Bài 5 (7.0 điểm). Cho 2017 số dương a 1 ; a 2 ; : : : ; a 2 0 1 7 : Với mỗi n > 20 17; ta đặt
˚
a n D max a i 1 a i 2 a i 3 j i 1 C i 2 C i 3 D n ; 1  i 1  i 2  i 3  n 1 :

Chứng minh rằng tồn tại số nguyên dương m không vượt quá 2 01 7 và số nguyên dương
N > 4 m sao cho a n a n 4 m D a n2 2 m với mọi n > N :

f
Lời giải. Với mỗi n nguyên dương, ta đặt b n D ln a n : Khi đó bài toán có thể được phát biểu

af
lại như sau: Cho 2017 số thực b 1 ; b 2 ; : : : ; b 2 0 1 7 : Với mỗi n > 2017; ta đặt
˚
b n D max b i 1 C b i 2 C b i 3 j i 1 C i 2 C i 3 D n ; 1  i 1  i 2  i 3  n 1 :

Chứng minh rằng tồn tại số nguyên dương m không vượt quá 2017 và số nguyên dương
N > 4 m sao cho b n C b n 4 m D 2 b n 2 m với mọi n > N :

Nhận xét 1. Với mọi n nguyên dương thì


n ˇ
ˇ
o st
Gọi ` . 1  `  2017/ là số sao cho b`` D max bii ˇ 1  i  2017 : Ta có nhận xét sau:
n
bn b`
 :
n `
ilo

Chứng minh. Ta chứng minh bằng quy nạp theo n : Rõ ràng khẳng định đúng với n  2017
do cách định nghĩa ` : Ta chỉ cần xét trường hợp n > 2017: Giả sử khẳng định đúng với mọi
k < n : Từ cách xác định b n ; ta thấy tồn tại j 1 ; j 2 ; j 3 2 N  thỏa j 1 C j 2 C j 3 D n sao cho
s

b n D b j1 C b j2 C b j3 :
Ep

Sử dụng giả thiết quy nạp, ta có


b` b` b` b`
bn  j1  C j2  C j3  Dn ;
` ` ` `
bn b`
từ đó suy ra n
 `
: Do đó khẳng định cũng đúng với n : Nhận xét được chứng minh.
Bây giờ, với mỗi n nguyên dương, đặt c n D n b ` `b n thì từ nhận xét trên ta có c n  0 với
mọi n : Đồng thời, với mọi n  2017 thì
c nC 2 ` D .n C 2 `/b ` `b nC 2 `
 .n C 2 `/b ` `.b n C b ` C b ` /
D n b ` `b n D c n :
Từ đó suy ra

c nC 2 k `  c nC 2 . k 1/`      cn ; 8 n  2017; k  1:
Lời giải và bình luận đề thi chọn đội tuyển IMO 2017 17

Gọi x số nguyên dương nhỏ nhất sao cho 2x ` > 2017 và đặt

M D maxf c i j 1  i  4 x ` 1g:

Khi đó, với mọi n > 2x ` ; đặt n D 2 k x ` C r (với 0  r < 2x `), ta có

c n  c r C2 . k 1/x `  cr C2.k 2/x `      cr C2x `  M :

Ta có các nhận xét sau:


Nhận xét 2. Với mọi n nguyên dương, tồn tại các số tự nhiên s 1 ; s 2 ; : : : ; s 2 0 1 7 sao cho

cn D s1 c1 C s2 c2 C    C s2017 c2017 :

Chứng minh. Ta chứng minh bằng quy nạp theo n : Rõ ràng khẳng định đúng với mọi

f
n  2 0 1 7 (chỉ cần chọn s n D 1 và s i D 0 với i ¤ n). Ta chỉ cần xét trường hợp n > 20 17

af
là đủ. Giả sử khẳng định đúng với mọi k < n : Tư công thức xác định của b n ; ta suy ra

c n D minfc i 1 C c i 2 C c i 3 j 1  i 1  i 2  i 3  n 1; i 1 C i 2 C i 3 D n g; 8 n > 2017:

Do đó tồn tại j 1 ; j 2 ; j 3 2 N  thỏa j 1 C j 2 C j 3 D n sao cho


st
c n D c j1 C c j2 C c j3 :

Theo giả thiết quy nạp, tồn tại các số tự nhiên u 1 ; u 2 ; : : : ; u 2 0 1 7 ; v 1 ; v 2 ; : : : ; v 2 0 1 7 ; w 1 ;


n
w 2 ; : : : ; w 2 0 1 7 sao cho

c j1 D u 1 c 1 C u 2 c 2 C    C u 2017 c 2017 ;
ilo

c j2 D v 1 c 1 C v 2 c 2 C    C v 2017 c 2017 ;
c j2 D w 1 c 1 C w 2 c 2 C    C w 2017 c 2017 :

Do đó, ta có
cn D s1 c1 C s2 c2 C    C s2017 c2017 ;
s

trong đó s i D u i C v i C w i với 1  i  2017: Như vậy khẳng định cũng đúng với n : Nhận
Ep

xét được chứng minh.


Nhận xét 3. Dãy .c n / chỉ nhận hữu hạn giá trị.

Chứng minh. Nhận xét này được suy ra trực tiếp từ tính bị chặn của c n và kết quả của nhận
xét ở trên.
Bây giờ, do c n chỉ nhận hữu hạn giá trị và

c nC 2 k x `  c nC 2 . k 1/x `      cn

với mọi n  2017 nên tồn tại N 1 đủ lớn để c n D c n 2x ` với mọi n > N 1 : Khi đó, ta có

n b` `b n D .n 2x `/b ` `b n 2x ` ; 8n > N1 ;

hay
bn D 2 b` C bn 2x ` ; 8n > N1 :
18 Lời giải và bình luận đề thi chọn đội tuyển IMO 2017

Từ đây, ta cũng suy ra

bn 2x ` D 2 b` C bn 4x` ; 8 n > N 1 C 2x ` :

Cộng chéo vế hai kết quả trên, ta được

bn C bn 4x` D 2 bn 2x ` ; 8 n > N 1 C 2x ` :

Chọn N D N 1 C 2x ` và m D x ` ; ta có điều phải chứng minh.

Bình luận.

 Bước logarith hoá để đưa về dãy b n định nghĩa bởi b n D maxfb i C b j C b k j i C j C


k D ng là khá hiển nhiên. Nó không tạo ra bước ngoặt đáng kể nhưng ít nhất sẽ tạo sự

f
tiện lợi trong trình bày và tính toán. Trong các phần sau ta coi như đang làm việc với bài

af
toán đã chuyển đổi sang phép cộng.

 Đây là một bài toán dãy số được định nghĩa truy hồi thông qua max; tạo một tình huống tổ
hợp khá thú vị. Để hình dung “hành vi” của dãy số này, ta cần xây dựng thử dãy số với tham
st
số nhỏ. Chẳng hạn, nếu thay 2017 bằng 5 và với 5 số hạng đầu tiên là 1; 2 ; 4 ; 5; 10
thì các số hạng tiếp theo sẽ là 7; 12 ; 13; 15; 16; 21; : : : và chúng ta sẽ nhìn thầy vai
trò của số hạng thứ 5 (số 10) trong dãy số này, từ đó dẫn đến ý tưởng xét ` sao cho b`` lớn
nhất và các bước tiếp theo.
n
 Bài toán này có thể coi là anh em song sinh với bài số 6 trong đề thi Toán Quốc tế năm
2010, cụ thể như sau:
ilo

Cho a 1 ; a 2 ; a 3 ; : : : là dãy các số thực dương và s là số nguyên dương sao cho

a n D maxfa k C a n k j1k n 1g
s

với mọi n > s : Chứng minh rằng tồn tại các số nguyên dương l  s và N sao cho
a n D a l C a n l với mọi n  N :
Ep

Sự khác biệt chỉ nằm ở phép cộng và phép nhân, vốn có thể chuyển sang dễ dàng nhờ
logarith, cũng như độ sâu của phép truy hồi. Tuy nhiên bước chuyển từ 2 sang 3 này
gần như không
˚ a i ảnh hưởng đến các bước lý luận của lời giải. Ý tưởng chính vẫn là xét
t D max i ; i D 1; : : : ; s và xét dãy số b n D n t a n :

Bài 6 (7.0 điểm). Với mỗi số nguyên dương n, xét a 1 ; a 2 ; : : : ; a 2 n là hoán vị của 2 n số
nguyên dương đầu tiên. Một hoán vị như thế được gọi là “đẹp” nếu với mọi 1  i < j  2 n
thì a i C a nC i D 2 n C 1 và a i a i C1 không đồng dư với a j a j C 1 theo modulo 2 n C 1.
(Quy ước a 2 nC 1 D a 1 :)

a) Với n D 6, hãy chỉ ra một hoán vị đẹp.

b) Chứng minh rằng với mỗi n nguyên dương thì luôn tồn tại một hoán vị đẹp.
Lời giải và bình luận đề thi chọn đội tuyển IMO 2017 19

Lời giải. a) Ta xét hoán vị sau:

1; 2; 4; 8; 3; 6; 12 ; 11; 9; 5; 10; 7:

Dễ thấy rằng 1 C 12 D 2 C 11 D 4 C 9 D 8 C 5 D 3 C 10 D 6 C 7. Đồng thời, hiệu của


các số liên tiếp lấy theo modulo 13 lần lượt là 1; 2 ; 4 ; 8; 3; 6; 12 ; 11; 9; 5; 10; 7 đều
phân biệt. Do đó, hoán vị này thỏa mãn điều kiện đề bài.
b) Gọi A là tập hợp các hoán vị đẹp và B là tập hợp các cách điền các số 1; 2 ; : : : ; 2 n
lên các đỉnh của đa giác đều X 1 X 2 X 3 : : : X 2 n , sao cho hai số thuộc các đầu mút của một
đường kính có tổng là 2 n C 1I đồng thời không có bất cứ một dãy liên tiếp các số nào có
tổng là bội của 2 n C 1:
Ta sẽ chứng minh rằng tồn tại song ánh từ A ! B và khi đó, để chứng minh tồn tại hoán vị, ta
chỉ cần chỉ ra rằng jA j D j B j > 0:

f
(1) Tồn tại song ánh A ! B :

af
Xét một hoán vị thuộc A là .a 1 ; a 2 ; : : : ; a 2 n / thì với i D 1; 2 n, ta chọn b i 2 f 1; : : : ; 2 ng
sao cho b i  a i C 1 a i .mod 2 n C 1/. Theo giả thiết thì tất cả các số b i này phải đôi một
phân biệt. Ta quy ước rằng a 2 nC 1 D a 1 và b 2 nC 1 D b 1 .

ai C ai C1Cn
st
Ta điền các số này lên vòng tròn theo thứ tự, b i điền cho đỉnh X i . Dễ thấy

bi C bi Cn  ai C1 a i Cn D 2 .2 n C 1/  0

Do 0 < b i C b i C n < 2 .2 n C 1/ nên b i C b i C n D 2 n C 1: Ngoài ra, ta cũng có


.mod 2 n C 1/:
n
i 1
X
a i D .a i ai 1/ C .a i 1 ai 2/ C    C .a 2 a1 / C a1 D a1 C bk
ilo

k D1

và các số a i đôi một phân biệt theo modulo 2 n C 1 nên không tồn tại i ; j sao cho
j
X1
aj ai D bk  0 .mod 2 n C 1/:
s

k Di
Ep

Do đó, cách điền các số b i như trên thỏa mãn điều kiện cách điền trong B : Ngược lại, xét một
cách điền trong B và b i được điền cho đỉnh X i ; i D 1; 2 n thì xét dãy số
i
X
a1 D b1 ; a2 D b1 C b2 ; a3 D b1 C b2 C b3 ; :::; ai D bk
k D1

và tổng lấy theo modulo 2 n C 1: Khi đó, ta thấy rằng các số a i đều dương và đồng thời cũng
phân biệt, vì nếu không, giả sử có a i D a j thì
j
X1
0 D aj ai D bk .mod 2 n C 1/;
k Di

mâu thuẫn. Hơn nữa b i C b i C n  a i ai 1 C ai Cn ai Cn 1  0 .mod 2 n C 1/ nên

ai C ai Cn  ai 1 C a i Cn 1 .mod 2 n C 1/ với mọi i  2 :


20 Lời giải và bình luận đề thi chọn đội tuyển IMO 2017

Suy ra, tổng a i C a i C n với mọi i D 1; 2 n đều có cùng số dư là a 2 f 0; 1; 2 ; : : : ; 2 ng khi


chia cho 2 n C 1: Chú ý rằng có 2 n tổng như thế và chúng đôi một rời nhau nên
2n
X 2n
X
k D .a i C a i Cn /  2 n a .mod 2 n C 1/;
k D1 i D1

mà . 2 n ; 2 n C 1/ D 1 nên cần có a D 0 hay a i C a i C n D 2 n C 1 với mọi i D 1; 2 n.


Do đó, dãy số a i là hoán vị đẹp nên nó thuộc A : Đến đây ta có song ánh đi từ A vào B và
. 1 / được chứng minh.
(2) Chứng minh j B j > 0:
Trước hết, ta thấy rằng số cách điền sao cho b i C b i C n D 2 n C 1 với mọi i D 1; 2 n là nŠ  2 n
vì các số 1 ! 2 n thuộc các cặp rời nhau có tổng là 2 n C 1 và có thể đổi chỗ số điền cho X i

f
và X i C n : Giả sử trong một cách điền số lên đa giác, có một dãy liên tiếp có k số mà tổng chia
hết cho 2 n C 1 thì k  3 và nếu gọi S k là số tập con có k phần tử như thế thì ở đây, ta có

af
n  2 n k  .n k /Š  S k cách điền. Cụ thể là có n cách chọn vị trí bắt đầu của dãy (vì tính đối
xứng qua tâm nên nếu dãy k số này thỏa thì k số đối xứng của nó cũng thỏa), còn lại 2 n 2k
số xếp tùy ý thỏa mãn tính đối xứng là 2 n k  .n k /Š.

liên tiếp có tổng chia hết cho 2 n C 1 sẽ không vượt quá

S Dn
n
X
2n k
.n
st
Từ đó suy ra số cách điền thỏa điều kiện tổng b i C b i C n D 2 n C 1 nhưng có một dãy các số

k /ŠS k :
n
k D3

Để đánh giá đại lượng này, ta tiếp tục chứng minh các nhận xét sau:
ilo

1 k 1
a) S k  C .
k 2n
b) C 2kn 1  4 k 1 C nk 11 với mọi n  5; k  3; n  k : Khi đó với n D 1; 2 ; 3; 4 ta có
thể chỉ ra hoán vị cụ thể thỏa mãn đề bài.
s

x0 x1 x2 xn
< e x với mọi n.
Ep

c) C C CC
0Š 1Š 2Š nŠ
Thật vậy:
a) Ta thấy rằng có đúng C 2kn 1 tập con tùy ý có k 1 phần tử của 2 n. Với mỗi tập con như
thế thì có không quá một cách để thêm vào một số để được k số có tổng là bội của 2 n C 1.
Ngược lại, mỗi tập con k phần tử có tổng chia hết cho 2 n C 1 thì tương ứng với đúng k
tập con có k 1 phần tử của 2 n : Dùng đếm bằng hai cách, ta có k  S k  1  C 2kn 1 hay
(a) được chứng minh.

b) Có thể chứng minh dễ dàng bằng quy nạp theo n với chú ý rằng
2n kC1 k n k 2n kC1 k
C 2kn  C nk 1 D C2n 1
 C nk 1
1  4 1
< 4k :
k k n k

xi
Pn
c) Có thể chứng minh f .x / D e x i D0 i Š luôn đồng biến bằng đạo hàm n lần.
Lời giải và bình luận đề thi chọn đội tuyển IMO 2017 21

Cuối cùng, ta có
n n
X
n k 1 X 1
S n 2 .n k /Š C nk 1 n 2n k
.n k /Š C nk 1 k
1 4
1
k k
k D3 k D3
n
n 1 X 2k e2 .1 C 2 C 2/
D 2 nŠ < 2 n nŠ < 2 n nŠ:
4 kŠ 4
k D3

Do đó, j B j  2 n nŠ S > 0. Bài toán được giải quyết hoàn toàn.

Bình luận.

 Ở bài 6 này, chúng tôi có tham khảo ý tưởng của Nguyễn Huy Tùng, IMO 2014 (ý
tưởng cho lời giải chi tiết ở trên) và Lê Hồng Quý, IMO 2006 (ý tưởng lời giải trong

f
phần bình luận bên dưới).

af
 Lời giải trên tiếp cận theo hướng xác suất, cụ thể là để chứng minh tồn tại một mô hình
thỏa mãn ràng buộc của đề bài, thay vì xây dựng trực tiếp, chúng ta có thể đếm hoặc ước
lượng tương đối số mô hình thỏa mãn và chứng minh nó là số dương.

 Một số bài toán tương tự: st


1. (Bulgaria, 2016) Hỏi có tồn tại hay không số nguyên dương n < 10 9 sao cho n có thể
biểu diễn dưới dạng tổng của ba số chính phương dương phân biệt bởi hơn 1000 cách?
n
Tương tự ý tưởng đã nêu trong bài 6, ta có thể đếm số lượng tổng a 2 C b 2 C c 2 phân
biệt với 1  a < b < c  N và N là số nguyên dương nào đó. Có tất cả C N3 bộ
.a ; b ; c / nhưng nhận không quá 3N 2 giá trị nên phải tồn tại một giá trị tương ứng
ilo

C3
với ít nhất 3 NN2 bộ. Chọn N D 18003 thì câu trả lời cho bài toán trên là khẳng định.
2. Chứng minh rằng trong 2 1 0 0 người, không nhất thiết phải có 200 người đôi một
quen nhau hoặc đôi một không quen nhau.
s

3. Trong một bảng 100  100; người ta viết vào mỗi ô một trong các số nguyên dương
không vượt quá 5000 và mỗi số xuất hiện đúng hai lần. Chứng minh rằng có thể
Ep

chọn ra 100 số phân biệt thuộc các hàng và cột khác nhau từ trong bảng trên.

 Để giải quyết bài 6, ta cũng có thể tiến hành trực tiếp với ý tưởng là xây dựng sao cho với
mọi i thì j a i C 1 a i j  2 i C 1 .mod 2 n C 1/, nếu có được điều này thì yêu cầu của
bài toán được thỏa mãn. Cụ thể là:

Xếp các số lên các đỉnh của đa giác X 1 X 2 : : : X 2 n sao cho X i D i với i D 1; n và
X i D i với i D n C 1; 2 n. Ta tiến hành chọn các số của hoán vị theo quy tắc sau:

 Xuất phát từ đỉnh X 1 , ta chọn a 1 D 1.


 Đi đến đỉnh liền sau của X n C1 là X nC 2 , và chọn a 2 D 2.
 Đi đến đỉnh liền sau của X 2 là X 3 và chọn a 3 D 3:
 Cứ như thế, ta chọn được n số đầu tiên của hoán vị là: 1; 2 ; 3; 4 ; 5; 6; : : : (ở
đây ta hiểu số âm theo nghĩa là lấy modulo với 2 n C 1). Sau đó các số còn lại chọn
đối xứng qua tâm là xong.
22 Lời giải và bình luận đề thi chọn đội tuyển IMO 2017

f
af
Tuy nhiên, vấn đề ở đây là a nC 1 D 1 nên hiệu j a nC 1 a n j 2 fn 1; n C 1g,

st
do đó ta cần tránh bước thứ i nào đó mà ja i C 1 a i j 2 f n 1; n C 1g (nếu hai
số này chẵn thì giá trị tương ứng là n ; n C 2).
Để làm được điều này, ở bước thứ i , thay vì ta đi vào điểm bên cạnh của điểm đối xứng
qua đường kính thì ta sẽ đi đến điểm bên cạnh của số đó để cho j a i C 1 a i j D 1,
n
đoạn còn lại vẫn thực hiện như cũ. Do việc chọn các giá trị theo cùng một chiều nên
không xảy ra trường hợp lặp lại số cũ.
ilo

Cuối cùng, ta chỉ cần xác định bước thứ i như thế. Để ý rằng nếu thực hiện như ban đầu,
ta sẽ có các số a 1 ; a 2 ; : : : ; a n chỉ chứa số nguyên dương lẻ và số nguyên âm chẵn.
Khi đó, nếu n chẵn thì a n D n và ja n a nC1 j D n 1, cần tránh giá trị n C 2
ra vì .n 1/  .n C 2/ .mod 2 n C 1/. Thay vì đi đến điểm nC 2
2
, ta đi sang
s

điểm bên cạnh của nó.


Nếu n lẻ thì tương tự, thay vì đi đến n C1 , ta cũng đi sang điểm bên cạnh. Theo quy
Ep

2
tắc này thì các giá trị ja i a i C1 j sẽ nhận tất cả các giá trị lẻ từ 1 ! 2 n 1 và
đương nhiên thỏa mãn điều kiện đề bài.
LỜI GIẢI VÀ BÌNH LUẬN ĐỀ THI
CHỌN ĐỘI TUYỂN IMO 2018
Trần Nam Dũng – Võ Quốc Bá Cẩn – Trần Quang Hùng
Lê Phúc Lữ – Nguyễn Lê Phước

1. Lời nói đầu

f
Kỳ thi chọn đội tuyển Việt Nam dự thi toán quốc tế năm 2018 đã diễn ra trong hai ngày 30 và

af
31/3 vừa qua. Trong mỗi ngày, thí sinh phải làm ba bài toán trong vòng 270 phút.
Tiếp nối truyền thống nhiều năm nay, Ban biên tập chúng tôi vẫn muốn theo sát các thí sinh
và giáo viên chuyên Toán, cùng nhau giải và phân tích, khai thác các bài có trong đề thi.

st
Thông qua đó, mọi người có thể thấy được bản chất, vẻ đẹp của các vấn đề, quan trọng hơn là
có nguồn tư liệu, định hướng rèn luyện cho các mùa thi sau. Ban biên tập hiểu rằng các đề thi
TST luôn mang tính thử thách, tạo cảm hứng và bên cạnh VMO, IMO, đây cũng là một trong
nội dung luôn được chờ đón.
n
Năm 2018, cùng với sự ra đời và đồng hành của chương trình BM2E – Bring Math to Everyone,
chúng tôi đã có nhiều động lực và sự hỗ trợ hơn. Với mong muốn đẩy mạnh phong trào chuyên
Toán nhiều nơi, hy vọng rằng B2ME sẽ luôn nhận được sự đón nhận của mọi người và chung tay
ilo

phát triển. Ngoài ra với tinh thần hội nhập quốc tế, năm nay chúng tôi cũng sẽ xuất bản Epsilon
14, phiên bản tiếng Anh.
Cuối cùng, để hoàn thành tài liệu này, chúng tôi xin gửi lời cám ơn đến thầy Nguyễn Chu Gia
Vượng (viện Toán học) và các bạn Hoàng Đỗ Kiên (HCB IMO 2013), Nguyễn Nguyễn (PTNK
s

TPHCM) vì những tài liệu cũng như những góp ý đáng giá. Mọi thắc mắc, góp ý, xin hãy gửi tin
nhắn cho trang chủ của tạp chí Epsilon.
Ep

1
2 Lời giải và bình luận đề thi chọn đội tuyển IMO 2018

2. Đề thi
2.1. Ngày thi thứ nhất (30/03/2018)
Bài 1 (7.0 điểm). Cho tam giác ABC nhọn không cân có D; E; F lần lượt là trung điểm của
các cạnh BC; CA và AB: Gọi .O/; .O 0 / lần lượt là tâm ngoại tiếp và tâm Euler của tam giác.
Xét điểm P bên trong tam giác DEF và DP; EP; FP cắt lại .O 0 / lần lượt tại D 0 ; E 0 ; F 0 : Gọi
A0 là điểm đối xứng với A qua D 0 : Xác định tương tự với B 0 và C 0 :
a) Chứng minh rằng nếu PO D PO 0 thì đường tròn .A0 B 0 C 0 / đi qua O:
b) Lấy X đối xứng với A0 qua đường thẳng OD: Xác định tương tự với Y và Z: Gọi H là
trực tâm tam giác ABC và XH; Y H; ZH cắt BC; CA; AB theo thứ tự tại M; N; K:
Chứng minh rằng M; N; K thẳng hàng.

f
Bài 2 (7.0 điểm). Với m là số nguyên dương, xét bảng ô vuông m  2018 gồm m hàng, 2018

af
cột mà trong đó có một vài ô trống, còn một vài ô được đánh số 0 hoặc 1: Bảng được gọi là “đầy
đủ” nếu với bất kỳ chuỗi nhị phân S có 2018 ký tự nào, ta đều có thể chọn ra một hàng nào đó
của bảng rồi điền thêm 0; 1 vào để 2018 ký tự của hàng tạo thành chuỗi S (nếu chuỗi S đã có
sẵn trên hàng nào đó rồi thì coi như thỏa mãn). Bảng được gọi là “tối giản” nếu nó đầy đủ và nếu
ta bỏ đi bất kỳ hàng nào thì nó không còn đầy đủ nữa.
st
a) Với 0  k  2018; chứng minh rằng tồn tại bảng tối giản 2 k  2018 sao cho có
đúng k cột có đủ cả 0 lẫn 1:
b) Cho bảng tối giản m  2018 có đúng k cột chứa cả 0 lẫn 1. Chứng minh rằng m  2 k :
n
(Một dãy nhị phân độ dài 2018 là dãy có dạng x 1 x 2 : : : x 2 0 1 8 trong đó x i 2 f 0; 1 g với
mọi i 2 f1; 2 ; : : : ; 2018g:)
ilo

Bài 3 (7.0 điểm). Cho số nguyên n  3 và A n là tập hợp tất cả các số nguyên dương nhỏ hơn
n ; nguyên tố cùng nhau với n : Xét đa thức
X
P n .x / D xk 1:
k 2A n
s

a) Chứng minh rằng P .x / chia hết cho đa thức x r C 1 với r là số nguyên dương nào đó.
Ep

b) Tìm tất cả các số nguyên dương n để P n .x / bất khả quy trên ZŒx :

2.2. Ngày thi thứ hai (31/03/2018)


Bài 4 (7.0 điểm). Cho a là số thực thuộc đoạn 21 ; 23 : Xét các dãy số .u n / và .v n /
 

. n D 0 ; 1 ; : : :/ được xác định như sau:


3 nC 1 3 nC 1
u n D nC 1  . 1/ b 2 a c ; v n D nC 1  . 1/ nC b 2 a c :
2 2
a) Chứng minh rằng
2X018
!2 2X018
!2
2
ui C vi  72a 2 48 a C 10 C 2 0 1 9 :
i D0 i D0
4

b) Tìm tất cả các giá trị của a để đẳng thức xảy ra.
Lời giải và bình luận đề thi chọn đội tuyển IMO 2018 3

Bài 5 (7.0 điểm). Một bảng ô vuông m  n AB C D có các đỉnh là các giao lộ (có tất cả
. m C 1 /  .n C 1/ giao lộ). Người ta muốn thiết lập một tuyến đường bắt đầu từ A; đi theo
các cạnh song song với các cạnh của hình chữ nhật và đi qua tất cả các giao lộ đúng một lần,
sau đó quay về A :

a) Chứng minh rằng có thể xây dựng được đường đi khi và chỉ khi m lẻ hoặc n lẻ.

b) Với m ; n thỏa mãn điều kiện câu a), hỏi có ít nhất bao nhiêu giao lộ mà tại đó có ngã rẽ?

Bài 6 (7.0 điểm). Cho tam giác AB C nhọn nội tiếp .O / và .J / là đường tròn bàng tiếp góc
A của tam giác. Gọi D ; E ; F lần lượt là tiếp điểm của .J / với B C ; C A và AB :

a) Gọi L là trung điểm của B C : Đường tròn đường kính LJ cắt các đường thẳng
DE ; DF lần lượt tại K và H : Chứng minh rằng các đường tròn .BDK / và .C DH /
cắt nhau trên đường tròn .J /:

f
af
b) Giả sử đường thẳng EF cắt đường thẳng B C tại G và GJ cắt các đường thẳng
A B ; AC lần lượt tại M ; N : Gọi P và Q là các điểm trên các đường thẳng JB ; J C
sao cho ∠ PAB D ∠ QAC D 90 ı : Gọi T là giao điểm của hai đường thẳng
P M ; QN và S là điểm chính giữa cung lớn B C của đường tròn .O /: Gọi I là
st
tâm đường tròn nội tiếp tam giác AB C : Chứng minh rằng đường thẳng S I cắt đường
thẳng AT tại một điểm thuộc đường tròn .O /:
n
s ilo
Ep
4 Lời giải và bình luận đề thi chọn đội tuyển IMO 2018

3. Bình luận chung


Các bài toán năm nay được phân theo phân môn như sau:

 Bài 1 và bài 6: Hình học.

 Bài 2 và bài 5: Tổ hợp.

 Bài 3: Số học.

 Bài 4: Đại số.

Hai bài hình có cấu hình khá đẹp, đặc biệt là ở bài 1, nơi ta có thể gặp lại những ý tưởng
kinh điển của hình học qua đường tròn Euler và đường tròn Hagge: chứng minh đồng viên
thông qua phép vị tự. Tuy nhiên có vẻ như hai bài 1 và 6 quá giống nhau về dạng bài, công

f
cụ: định lý Menelaus, đẳng giác, phương tích, phép biến hình. Nếu sử dụng hai bài hình, nên

af
là hai bài khác hướng.
Hai bài tổ hợp khác nhẹ nhàng dù đều liên quan đến những lĩnh vực của toán hiện đại: lý thuyết
mã hóa thông tin và lý thuyết đồ thị. Bài 5 có vẻ là bài quen thuộc nhưng nếu chưa gặp thì cũng
không khó xử lý.
st
Bài 3 là một bài phát biểu dưới dạng đa thức (thậm chí có cả từ bất khả quy) nhưng bản chất là
một bài số học. Ý tưởng cơ bản là xây dựng các song ánh (dạng k ! n k ; k ! k C n2 hoặc
tương tự) giữa các đơn thức, còn trường hợp n phi chính phương (square-free) thì dùng quy nạp
n
theo số ước nguyên tố. Bài này có ít nhiều liên quan đến tổng Ramanujan trong số học.
Bài 4 là một bài bất đẳng thức dãy số có hình thức khá lạ và khá xấu. Tuy nhiên, nếu nhìn kỹ có
thể thấy được tư tưởng dùng hệ đếm nhị phân. Nếu “bắt” được ý này thì phần sau chỉ là phần xử
ilo

lý kỹ thuật. Có lẽ vì vậy mà ban đề thi đưa bài này vào vị trí bài dễ nhất trong ngày thứ hai, dù
theo chúng tôi, bài này khó hơn bài 5 nhiều và sẽ là bài “sát thủ”.
Như vậy, chúng ta có thể thấy rằng, hai bài 3, 4 là hai bài Số học và Đại số song kiếm hợp bích.
Nhìn tổng thể, đề thi năm nay khá hay và tốt, có nhiều đất diễn cho thí sinh, không có bài siêu
s

sát thủ. Năm nay ban chấm thi sẽ vất vả. Dự kiến là bài nào cũng có thể có thí sinh làm trọn vẹn.
Có một điểm chưa được hay là tất cả các bài TST năm nay đều có hai ý a) và b). Trong một số
Ep

trường hợp có hai ý là hay nhưng trong nhiều trường hợp nó làm bài toán trở nên lắt nhắt, hay
như ý b) của bài 3 trở nên tầm thường khi đã có ý a). Việc có hai bài hình “đồng dạng” cũng là
điều cần khắc phục và chúng ta nên khai thác thêm mảng Hình tổ hợp và bất đẳng thức hình học
để đa dạng hóa. Hơn nữa hai bài hình đều có hai ý a), b) gần như độc lập với nhau thành ra gần
như thí sinh phải giải quyết bốn bài hình khác nhau trong cùng một kỳ thi.
Dự kiến ngưỡng điểm lọt vào đội tuyển vẫn là 3++.
Lời giải và bình luận đề thi chọn đội tuyển IMO 2018 5

4. Lời giải và bình luận các bài toán

Bài 1 (7.0 điểm). Cho tam giác AB C nhọn không cân có D ; E ; F lần lượt là trung điểm
của các cạnh B C ; C A và AB : Gọi .O / ; . O 0 / lần lượt là tâm ngoại tiếp và tâm Euler của
tam giác. Xét điểm P bên trong tam giác DEF và DP ; EP ; F P cắt lại .O 0 / lần lượt tại
D 0 ; E 0 ; F 0 : Gọi A 0 là điểm đối xứng với A qua D 0 : Xác định tương tự với B 0 và C 0 :

a) Chứng minh rằng nếu P O D P O 0 thì đường tròn . A 0 B 0 C 0 / đi qua O :

b) Lấy X đối xứng với A 0 qua đường thẳng OD: Xác định tương tự với Y và Z : Gọi H
là trực tâm tam giác AB C và XH ; Y H ; ZH cắt B C ; C A; AB theo thứ tự tại
M ; N ; K : Chứng minh rằng M ; N ; K thẳng hàng.

f
Lời giải. a) Gọi I là điểm đối xứng với O qua P : Ta có O 0 là trung điểm của OH nên

af
O 0 P k I H : Lại có P O D P O 0 nên I O D I H :
A

G
S D0
st
n
I

P
ilo

A0
O
H O0
s

B D C
Ep

Gọi S và G lần lượt là trung điểm của các đoạn thẳng AI; AH: Ta có
1 1
SP D AO D R D O 0 D
2 2
và SP k AO k O 0 D nên tứ giác O 0 SPD là hình bình hành. Từ đó suy ra DP k O 0 S:
Lại có SP D 12 R D O 0 D 0 nên tứ giác SD 0 PO 0 là hình thang cân. Suy ra O 0 P D SD 0 và
IH D 2O 0 P D 2SD 0 D IA0 :
Từ đó, ta có IA0 D IH D IO nên A0 nằm trên đường tròn .I; IO/: Chứng minh tương tự, ta
cũng có B 0 ; C 0 nằm trên đường tròn .I; IO/: Từ đây, ta có điều phải chứng minh.
6 Lời giải và bình luận đề thi chọn đội tuyển IMO 2018

b) Gọi R là bán kính của đường tròn .O/: Dễ thấy GD D R: Xét phép vị tự tâm A tỉ số 12 biến
B; C; A0 ; X; H; M và trung trực BC lần lượt thành thành F; E; D 0 ; U; G; M 0 và trung trực
MB 0F
EF: Khi đó, ta có M C
DM M 0E
và U đối xứng với D 0 qua trung trực EF: Suy ra
p
MB M 0F GF UF R2 DF 2 D 0 E
D 0 D  Dp  :
MC ME GE UE R2 DE 2 D 0 F

G
D0

f
U
M0 E

af
F P

O
H O0

M B
X

D
st A0

C
n
ilo

NC KA
Tương tự, ta cũng tính được NA
và KB
: Do DD 0 ; EE 0 ; FF 0 đồng quy nên

D0F F 0E E 0D
  D 1:
s

D0E F 0D E 0F
Ep

MB NC KA
Từ đó MC
 NA
 KB
D 1; hay M; N; K thẳng hàng.

Bình luận. Đây là một bài toán hình học hay và có nhiều ý mới. Nếu như câu b) chỉ là ứng
dụng của định lý Menelaus khá cơ bản thì câu a) là một bài toán rất thú vị. Điểm đáng chú ý của
câu a) là ta có thể viết lại thành bài toán thú vị như sau:
Bài toán 1. Cho tam giác ABC với D; E; F lần lượt là trung điểm của các cạnh BC; CA và
AB: P là một điểm bất kỳ trên mặt phẳng. Các đường thẳng PD; PE; PF cắt lại đường tròn
.DEF / lần lượt tại X; Y và Z: Gọi U; V và W lần lượt là đối xứng của A; B; C qua X; Y; Z:
a) Chứng minh rằng đường tròn .U V W / đi qua trực tâm tam giác ABC:

b) Gọi K là tâm của đường tròn .U V W /: Chứng minh rằng đường thẳng PK luôn đi qua
một điểm cố định khi P thay đổi.
Bài toán trên là một bài toán đồng viên với trực tâm mới và có nhiều phát triển hay, ta có thể áp
dụng cách giải như trong lời giải của bài TST để giải bài toán này.
Lời giải và bình luận đề thi chọn đội tuyển IMO 2018 7

Sau đây là một tổng quát hơn nữa cho bài toán đồng viên trên:
Bài toán 2. Cho tam giác ABC nội tiếp trong đường tròn .O/ và hai điểm P; Q bất kỳ trên
mặt phẳng. Phép vị tự tâm P tỉ số k biến các điểm A; B; C thành các điểm A0 ; B0 và C0 : Gọi
A0 A1 ; B0 B1 và C0 C1 lần lượt là các đường kính của đường tròn .A0 B0 C0 /: Các đường thẳng
QA1 ; QB1 và QC1 cắt lại đường tròn .A1 B1 C1 / lần lượt tại A2 ; B2 ; C2 :

a) Phép vị tự tâm A; B; C tỉ số 1 1 k lần lượt biến các điểm A2 ; B2 ; C2 thành A3 ; B3 và C3 :


Chứng minh rằng đường tròn .A3 B3 C3 / đi qua P:
1
b) Gọi K là tâm của đường tròn .A3 B3 C3 /: Chứng minh rằng phép vị tự tâm K tỉ số k
biến
P thành tâm ngoại tiếp tam giác ABC:

Mặt khác cũng từ cấu hình bài toán TST chúng ta cũng có thể tìm và khai thác được nhiều điểm
thú vị khác, sau đây là một khai thác thú vị từ cấu hình này:

f
Bài toán 3. Cho tam giác AB C có trực tâm H : Gọi A 1 ; B 1 ; C 1 lần lượt là trung điểm

af
của các cạnh B C ; C A và AB : P là điểm bất kỳ trên đường thẳng Euler của tam giác
A B C : . N / là đường tròn Euler của tam giác AB C : Gọi A 1 A 0 ; B 1 B 0 và C 1 C 0 lần lượt
là các đường kính của đường tròn .N /:

st
a) Chứng minh rằng các đường thẳng AA 0 ; BB 0 và C C 0 đồng quy tại X :

b) Các đường thẳng AA 0 ; BB 0 và C C 0 lần lượt cắt lại đường tròn .N / tại A 2 ; B 2 ; C 2 :
Q là điểm bất kỳ trên đường thẳng P X : Các đường thẳng QA 2 ; QB 2 và Q C 2 lần lượt
cắt lại đường tròn .N / tại A 3 ; B 3 ; C 3 : Gọi A 4 ; B 4 và C 4 lần lượt là đối xứng của A;
n
B ; C lần lượt qua A 3 ; B 3 ; C 3 : Chứng minh rằng đường tròn .K / ngoại tiếp tam giác
A 4 B 4 C 4 đi qua H :
ilo

c) Chứng minh rằng K luôn nằm trên một đường thẳng cố định song song với đường
thẳng P X khi Q thay đổi.

Ngoài ra ta cũng có thể sử dụng kết quả bài toán sau làm bổ đề để chứng minh ý b) của bài TST
bằng phương pháp phương tích và trụ đẳng phương:
s

Bài toán 4. Cho tam giác AB C và điểm P bất kỳ trên mặt phẳng. Các điểm X ; Y và Z nằm
Ep

trên các đường tròn .P B C /; .P C A/; .PAB / sao cho đường tròn .X Y Z / đi qua P : Gọi
U ; V và W lần lượt là đối xứng của X ; Y ; Z qua trung trực của B C ; C A; AB : Chứng minh
rằng đường tròn .U V W / cũng đi qua P :
Có thể tham khảo thêm các mở rộng tại liên kết: http://analgeomatica.blogspot.
com/2018/04/cac-bai-toan-hinh-hoc-hang-tuan-tuan-1.html.
8 Lời giải và bình luận đề thi chọn đội tuyển IMO 2018

Bài 2 (7.0 điểm). Với m là số nguyên dương, xét bảng ô vuông m  20 18 gồm m hàng,
2 0 1 8 cột mà trong đó có một vài ô trống, còn một vài ô được đánh số 0 hoặc 1: Bảng được
gọi là “đầy đủ” nếu với bất kỳ chuỗi nhị phân S có 2 01 8 ký tự nào, ta đều có thể chọn ra
một hàng nào đó của bảng rồi điền thêm 0 ; 1 vào để 20 18 ký tự của hàng tạo thành chuỗi
S (nếu chuỗi S đã có sẵn trên hàng nào đó rồi thì coi như thỏa mãn). Bảng được gọi là “tối
giản” nếu nó đầy đủ và nếu ta bỏ đi bất kỳ hàng nào thì nó không còn đầy đủ nữa.

a) Với 0  k  2018; chứng minh rằng tồn tại bảng tối giản 2 k  20 18 sao cho có
đúng k cột có đủ cả 0 lẫn 1:

b) Cho bảng tối giản m  2018 có đúng k cột chứa cả 0 lẫn 1. Chứng minh rằng m  2 k :

(Một dãy nhị phân độ dài 2018 là dãy có dạng x 1 x 2 : : : x 2 0 1 8 trong đó x i 2 f0; 1 g
với mọi i 2 f1; 2 ; : : : ; 2018g:)

f
af
Lời giải. a) Đầu tiên, xét một bảng ô vuông trống kích thước 2 k  2018 và 2 k xâu nhị phân
có độ dài k ; ta sẽ tiến hành điền các số 0; 1 của các xâu này lần lượt vào phần bên trái của từng
hàng, mỗi xâu dùng cho một hàng. Khi đó, phần còn lại phía bên phải là 2018 k cột được

::: 0
st
để trống. Dễ thấy rằng ở mỗi cột trong k cột bên trái của bảng đều có chứa đúng 2 k 1 số 0 và
2 k 1 số 1 (tức là có đủ cả 0 lẫn 1 như ràng buộc). Ta sẽ chứng minh bảng này tối giản.

0 0 ::: ::: :::


0 0 ::: 1 ::: ::: :::
n
::: ::: ::: ::: 0 ::: :::
ilo

1 1 ::: 0 ::: ::: :::


1 1 ::: 1 ::: ::: :::
Với một xâu nhị phân s D a 1 a 2 : : : a 2 0 1 8 tùy ý, ta xét xâu con s 0 D a 1 a 2 : : : a k của
nó. Rõ ràng s 0 đã xuất hiện ở phần đầu của hàng nào đó trên bảng nên nếu ta điền tiếp
s

a k C1 ; a k C 2 ; : : : ; a 2 0 1 8 vào phần trống thì sẽ thu được s : Ngoài ra, ta thấy rằng cũng chỉ có
đúng một hàng chứa các số của s 0 nên nếu bỏ hàng đó đi thì không có hàng nào có thể dùng để
Ep

“khôi phục” s được. Do đó, bảng đã cho thỏa mãn.


b) Giả sử rằng k cột đầu tiên bên trái của bảng có chứa đủ 0 và 1: Dưới đây, ta sẽ chứng minh
nhận xét quan trọng mấu chốt để giải bài toán này.
Nhận xét. Tất cả các ô trong 2018 k cột bên phải đều trống.
Chứng minh. Xét xâu nhị phân s có độ dài k bất kỳ và giả sử A s là tập hợp các hàng mà k ô
đầu tiên bên trái của nó có thể sinh ra s : Ta sẽ chứng minh rằng trong A s sẽ có một hàng mà
toàn bộ các ô từ vị trí k C 1 đến vị trí 2018 đều trống.
Xét các ô ở vị trí thứ k C 1 ở mỗi hàng trong A s : Dễ thấy tất cả các ô đó thuộc về cột thứ k C 1
của bảng, là cột không chứa đồng thời 0 và 1:
Nếu toàn bộ cột này có chứa số thì giả sử đều là số 0 (tương tự nếu là 1). Khi đó, chuỗi độ dài
k C 1 có dạng 00 : : : 01 gồm k số 0 bên trái sẽ không thể biểu diễn được bởi bất cứ hàng nào,
mâu thuẫn với tính đầy đủ của bảng. Do đó, phải có một tập con của A s có các hàng mà tại vị trí
thứ k C 1 là ô trống, đặt là A 0s :
Lời giải và bình luận đề thi chọn đội tuyển IMO 2018 9

Tiếp tục, lại xét vị trí thứ k C 2 thì tương tự trên, nếu toàn bộ cột k C 2 có chứa số thì không
thỏa nên phải có cột trống, như vậy ta lại tiếp tục chọn được tập con A 0s0 của A 0s mà vị trí thứ
k C 1 ; k C 2 đều trống. Cứ làm như thế cho đến khi gặp cột cuối cùng và ta được một hàng có
tất cả các vị trí từ k C 1 đến 2018 đều trống.
Như thế, với mỗi xâu nhị phân s có độ dài k , ta luôn tìm được một hàng mà tất cả các ô từ vị trí
k C 1 đến 2018 đều trống. Chú ý rằng các hàng này không nhất thiết phân biệt vì có thể có một
hàng được sử dụng cho nhiều xâu. Gọi tập hợp các hàng này là A :
Ta thấy A có thể biểu diễn được cho tất cả các xâu độ dài 2018: Rõ ràng mỗi hàng trên bảng
đều phải thuộc vào A vì nếu không, ta có thể loại bỏ hàng đó đi mà bảng vẫn còn đầy đủ, mâu
thuẫn với tính chất tối giản. Như vậy, A cũng chính là tập hợp tất cả các hàng của bảng. Do đó,
toàn bộ các cột từ k C 1 đến 2018 của bảng đều trống. Nhận xét được chứng minh. 
Tiếp theo, vì toàn bộ bảng con bên phải trống nên có thể biểu diễn được mọi xâu nhị phân độ dài

f
2 0 1 8 k. Vì bảng ban đầu là tối giản nên buộc bảng con bên trái cũng tối giản.

af
Xóa đi 2 0 18 k cột trống để còn bảng con kích thước m  k : Đánh số các hàng từ 1 đến m
và đặt A i . i D 1; 2 ; : : : ; m/ là tập hợp các xâu nhị phân độ dài k có thể sinh ra từ hàng thứ
i (chú ý rằng trên các hàng đó vẫn có thể còn các ô trống xen kẽ, không nhất thiết phải được điền
toàn bộ bởi các số 0 và 1).

st
Vì bảng ban đầu tối giản đối với các xâu nhị phân độ dài 2018 nên bảng mới sinh ra cũng tối
giản với các xâu nhị phân độ dài k : Suy ra nếu đặt B D A 1 [ A 2 [    [ A m thì j B j D 2 k
(nghĩa là bảng này có thể sinh ra được tất cả 2 k xâu nhị phân độ dài k ).
Rõ ràng với mỗi i .i D 1; 2 ; : : : ; m/; phải có một xâu nhị phân độ dài k nào đó chỉ được
n
sinh ra bởi hàng thứ i bởi nếu ngược lại, tất cả các xâu nhị phân sinh ra từ hàng i cũng được sinh
ra từ hàng khác thì ta bỏ hàng đó đi dẫn đến bảng không còn tối giản nữa. Điều này có nghĩa là
ilo

A i phải đóng góp ít nhất một phần tử vào A và phần tử này không thuộc các tập khác. Do đó m
tập hợp sẽ đóng góp ít nhất m phần tử vào B và kéo theo j B j  m :
So sánh các đánh giá trên, ta có ngay m  2 k . Đây chính là điều phải chứng minh.
Bình luận. Ở câu a), với số hàng của bảng là 2 k và với điều kiện đặt ra là đúng k cột có chứa
cả 0 lẫn 1 ; ta có thể dễ dàng nghĩ đến việc xét tất cả các xâu nhị phân có độ dài k (hiển nhiên có
s

đúng 2 k xâu như thế).


Ep

Cái khó nhất của câu b) chính là tính tùy ý của các cột trong 2018 k cột còn lại, ngoài trường
hợp đặc biệt vừa xét, vẫn có thể còn các cột chỉ được điền 0; chỉ được điền 1: Ta phải dùng tính
tối giản của bảng để loại các trường hợp đó đi. Chú ý rằng câu b) trong đề cho số k tùy ý nên có
thể xuất phát từ trường hợp k D 0 để thấy rằng tất cả các ô đều phải trống. Đây cũng chính là
định hướng của lời giải trên.
Với quan hệ giữa các hàng của bảng và các chuỗi nhị phân, bài toán này hứa hẹn sẽ có cách tiếp
cận sử dụng lý thuyết graph, đặc biệt là bipartie graph. Mong được trao đổi thêm với bạn đọc.
Dạng toán về tập hợp mô hình hóa trên bảng này đã từng xuất hiện một lần trong đề VMO 2015.
Dưới đây, ta xét một số bài toán tương tự:
Bài toán 1 (IMO Shortlist, 1998). Cho số nguyên n  2 và tập hợp A D f1; 2 ; : : : ; ng:
Một họ F gồm t tập hợp con của A; đặt là A 1 ; A 2 ; : : : ;ˇ A t được gọi ˇlà “rời nhau” nếu
như với mỗi cặp số fx ; y g lấy từ A thì tồn tại A i 2 F mà ˇ A i \ f x ; y g ˇ D 1: Họ F được
gọi là “bao phủ” nếu như mỗi phần tử của A đều thuộc vào ít nhất một tập nào đó trong F :
Chứng minh rằng n  2 t 1:
10 Lời giải và bình luận đề thi chọn đội tuyển IMO 2018

Bài toán 2 (Bổ đề Kleitman). Một họ F gồm các tập con của tập hợp n số nguyên dương đầu
tiên được gọi là “down closed” nếu mỗi X 2 F thì tất cả tập con của X cũng thuộc F : Một họ
F gồm các tập hợp được gọi là “up closed” nếu mỗi X 2 F thì tất cả các tập hợp nhận X là
tập con cũng thuộc F : Chứng minh rằng

a) Nếu F 1 ; F 2 là hai họ “down closed” thì

jF1 j  jF2 j
jF1 \ F2 j  :
2n
b) Nếu F 1 ; F 2 lần lượt là họ “down closed” và “up closed” thì

jF1 j  jF2 j
jF1 \ F2 j  :
2n

f
Bài toán 3 (Iran, 2001). Với số nguyên dương n  2 ; xét bảng ô vuông n  n được điền

af
các số 0 và 1; một “đường chéo suy rộng” là tập hợp n phần tử không cùng hàng và cùng
cột. Giả sử trên bảng này có đúng một đường chéo suy rộng chứa toàn số 1: Chứng minh
rằng có thể sắp xếp lại các hàng và cột của bảng để được bảng mới mà mọi số tại vị trí
. i ; j / với 1  j < i  n đều là 0:

st
Bài toán 4. Xét bảng ô vuông m  n có chứa các số 0 hoặc 1: Giả sử toàn bộ bảng có ít nhất
˛ m n số 1 với 0 < ˛ < 1: Chứng minh rằng có ít nhất một trong hai ràng buộc sau phải đúng:
p
i) Có ít nhất một hàng có chứa ít nhất n ˛ số 1:
n
p
ii) Có ít nhất m ˛ hàng mà mỗi hàng có chứa ít nhất ˛ n số 1:
s ilo
Ep
Lời giải và bình luận đề thi chọn đội tuyển IMO 2018 11

Bài 3 (7.0 điểm). Cho số nguyên n  3 và A n là tập hợp tất cả các số nguyên dương nhỏ
hơn n ; nguyên tố cùng nhau với n : Xét đa thức
X
Pn .x / D xk 1:
k 2A n

a) Chứng minh rằng P .x / chia hết cho đa thức x r C 1 với r là số nguyên dương nào đó.

b) Tìm tất cả các số nguyên dương n để P n .x / bất khả quy trên ZŒ x  :

km
Lời giải. a) Trước hết, ta thấy rằng với m ; k 2 Z C và k lẻ thì xP C 1 chia hết cho x m C 1:
k
Để thuận lợi trong lập luận, xét đa thức Q n .x / D x P n .x / D k 2A n x và dễ thấy chỉ cần
r C
chứng minh Q n .x / chia hết cho x C 1 với số r 2 Z nào đó là được. Xét các trường hợp sau:

f
 Nếu n là số lẻ: Vì gcd.n ; k / D gcd.n ; n k / với mọi k D 1; 2 ; : : : ; n 1

af
nên với k 2 A n thì n k 2 A n : Suy ra, ta có thể nhóm các đơn thức trong P n .x /
thành các cặp rời nhau có dạng .x k ; x n k /: Vì k ; n k khác tính chẵn lẻ nên
. n k / k D n 2k lẻ và x k C x n k D x k .x n 2 k C 1/ chia hết cho x C 1: Từ
đó suy ra Q n .x / chia hết cho x C 1; thỏa mãn.
st
 Nếu n chia hết cho 4: Tương tự như trường hợp trên, ta cũng ghép cặp .x k ; x n k / với
chú ý hiệu n 2k là số chia 4 dư 2: Suy ra x k C x n k D x k .x n 2k C 1/ chia hết cho
x 2 C 1: Do đó, Qn .x/ chia hết cho x 2 C 1; cũng thỏa mãn.
n
 Nếu n chia 4 dư 2: Ta sẽ chứng minh các nhận xét sau:
Nhận xét 1. Nếu Qn .x/ chia hết cho x r C 1 .r 2 ZC / thì với số nguyên tố lẻ p mà p j n;
ilo

đa thức Qpn .x/ cũng thế.


Chứng minh. Giả sử a 2 A n thì các số kp C a 2 A p n với k D 0; 1; : : : ; p 1:
với mũ là kp C a với a 2 A n tương ứng sẽ có
Ứng với mỗi số k ; tổng các đơn thức P
cùng nhân tử là kp và sẽ chia hết cho a 2 A x a , tức là cũng chia hết cho x r C 1: Do
s

đó, Q p n .x / chia hết cho x r C 1: 


Nhận xét 2. Với n D 2p 1 p 2    p m ; trong đó m 2 Z C và p 1 ; p 2 ; : : : ; p m là các
Ep

số nguyên tố lẻ phân biệt tùy ý thì Q n .x / sẽ phân tích được.


Chứng minh. Ta sẽ chứng minh bằng quy nạp theo m là số ước nguyên tố lẻ của n :
Nếu m D 1 thì khi n D 2p với p nguyên tố, dễ thấy A n D f1; 2 ; : : : ; 2p g nf p ; 2p g
nên Q n .x / sẽ chia hết cho x p C 1:
Giả sử khẳng định đúng với m  1: Xét n D 2p 1 p 2    p m và p là số nguyên tố lẻ,
gcd. p ; n/ D 1; trong đó Q n .x / chia hết cho x r C 1: Ta sẽ chứng minh rằng với
N D p n thì Q N .x / cũng chia hết cho x r C 1:
Trước hết, giả sử trong A N có tính luôn cả các số nguyên tố cùng nhau với n nhưng lại
chia hết cho p thì thay vì có Q N .x /; ta sẽ có một đa thức mới là R N .x /: Đa thức này
sẽ là sự lặp lại p lần của của đa thức Q n .x / (nghĩa là R N .x / có thể chia thành các
nhóm mà mỗi nhóm so với Q n .x / chỉ sai khác lũy thừa x kp với 1  k  p 1). Do
đó R N .x / chia hết cho x r C 1:
12 Lời giải và bình luận đề thi chọn đội tuyển IMO 2018

Tiếp theo, ta cần bỏ đi các lũy thừa chia hết cho p, đó là các đơn thức có dạng x ap với
a 2 A n : Khi đó, dễ thấy tổng của chúng sẽ là đa thức Q n .x p / mà Q n .x p / chia hết cho
x p r C 1, tức là cũng chia hết cho x r C 1: Tóm lại Q N .x / D R N .x / Q n .x p / chia
hết cho x r C 1: Do đó, khẳng định cũng đúng với số n có m C 1 ước nguyên tố lẻ. Theo
nguyên lý quy nạp thì nhận xét 2 được chứng minh. 
Kết hợp hai nhận xét lại, ta thấy trường hợp này vẫn thỏa mãn điều kiện. Bài toán được
giải quyết hoàn toàn.
b) Theo câu a), ta đưa về tìm tất cả các số nguyên dương n  3 sao cho jA n j D 2 ; điều này
tương đương với việc tìm n  3 để ' .n/ D 2 : Ta xét các trường hợp sau:
 Nếu n D p nguyên tố thì 2 D ' .n/ D n 1 nên n D 3 thỏa mãn.
 Nếu n D p ˛ với ˛ > 1 và p nguyên tố thì 2 D ' .n/ D p ˛ 1
.p 1/ nên dễ thấy

f
rằng p D 2 ; ˛ D 2 và n D 4 thỏa mãn.

af
 Nếu n có ít nhất hai ước nguyên tố khác nhau là p ; q thì theo tính chất nhân tính của
hàm Euler, 2 D ' .n/ sẽ chia hết cho ' .p / D p 1 và ' .q / D q 1 nên chỉ có thể
p D 2 và q D 3; tương ứng với n D 6 thỏa mãn.
Vậy tất cả các số n cần tìm là n 2 f3; 4 ; 6g :
st
Bình luận. Câu b) của bài toán thực sự rất nhẹ nhàng và được hướng dẫn trực tiếp từ câu a).
Nếu đề bài chỉ cho ý a) hoặc ý b) thì sẽ trọn vẹn hơn nhiều. Câu a) thực sự là một kết quả đẹp
và thú vị, nhất là trong trường hợp n là số square-free chẵn (n không chia hết cho số chính
phương lớn hơn 1 nào). Có một ý tưởng để giải quyết trường hợp này là chứng minh số lượng
n
các số có dạng 4 k C 1 và 4 k C 3 trong A n là bằng nhau; tuy nhiên, điều này chỉ đúng khi
n có ước nguyên tố dạng 4 k C 1. Điều này có thể chứng minh nhờ hai nhận xét:
ilo

 Trong A n luôn chứa số có dạng b D 4 k C 3 (nếu n D 4 m C 2 thì đó là một trong hai


số b D 2 m C 3 và b D 2 m C 5).
 Tập hợp A 0 n D f b a j a 2 A n g có cùng số lượng số chia 4 dư 1 và dư 3 như A n . Chú ý
rằng nếu a  1 . mod 4/ thì b a  3 . mod 4/ và ngược lại.
s

k
Một kết quả thú vị khác là đa thức P n . x / chia hết cho x 2 C 1 nếu như có một số nguyên tố p
thỏa mãn p j n và v 2 .p C 1/ D k : Bạn đọc hãy tự chứng minh kết quả này.
Ep

Bài toán cũng có thể giải bằng tổng Ramanujan với chú ý rằng nếu đặt C n .r / D Q n . r / mà
2i 
 D e n thì C m n D C m C n với mọi m ; n nguyên dương thỏa gcd.m ; n/ D 1:
Dưới đây là một số bài toán tương tự:
a 1
P
Bài toán 1. Với mỗi số nguyên dương n ; xét đa thức P n .x / D a 2An x với A n là tập
hợp các ước dương của n : Tìm tất cả các số nguyên dương n sao cho P n .x / bất khả quy.
Pn i
Bài toán 2 (China MO, 2013). Với mỗi số nguyên dương n ; xét đa thức P n .x / D i D0 ci x
với c i 2 f0; 1 g và c i  C ni . mod 2/:
a) Với m ; n ; x 0 là các số nguyên dương và x 0 C 1 không phải là lũy thừa của 2 ; chứng
minh rằng nếu P m .x 0 / chia hết cho P n .x 0 / thì P m .x / chia hết cho P n .x / với mọi
số nguyên dương x :
b) Hỏi có bao nhiêu số nguyên dương m  2018 sao cho P n .x / có thể phân tích thành
tích của đúng 7 đa thức bất khả quy‹
Lời giải và bình luận đề thi chọn đội tuyển IMO 2018 13

1 2

Bài 4 (7.0 điểm). Cho a là số thực thuộc đoạn 2
; 3
: Xét các dãy số . u n / và .v n /
. n D 0 ; 1; : : :/ được xác định như sau:
3 nC 1 a c 3 nC 1 a c
un D  . 1/ b 2 ; vn D  . 1 / nC b2 :
2 nC 1 2 n C1
a) Chứng minh rằng

2X
018
!2 2X
018
!2
2
ui C vi  72 a 2 48 a C 1 0 C :
i D0 i D0
42019

b) Tìm tất cả các giá trị của a để đẳng thức xảy ra.

f
Lời giải. a) Từ giả thiết, ta có v i D u i với i chẵn và v i D u i với i lẻ nên bất đẳng thức cần

af
chứng minh có thể được viết lại thành
1X009 1X008
!2 1X009 1X008
!2
2
u2i C u2 i C1 C u2i u2 i C1  72a 2 48 a C 10 C 2 0 1 9 ;

hay
i D0 i D0

1X
009
u2i
!2
i D0

C
1X
008
i D0

u2 i C1
!2
st
 36a 2
4

24 a C 5 C
1
: .1/
42019
n
i D0 i D0

Bây giờ, gọi biểu diễn nhị phân của a là


ilo

C 1
X xi
aD
i D1
2i

với x i 2 f 0; 1 g: Do 12  a  23 nên dễ thấy x 1 D 1: Với mỗi số tự nhiên i ; ta thấy tính


chẵn lẻ của b 2 i C1 a c phụ thuộc vào x i C1 ; cụ thể: Nếu x i C 1 D 0 thì b 2 i C 1 a c chẵn và nếu
s

i C1 i C1
x i C 1 D 1 thì b 2 i C 1 a c lẻ. Suy ra . 1/ b 2 a c D 1 nếu x i C 1 D 0 và . 1/ b 2 a c D 1
nếu x i C1 D 1: Trong mọi trường hợp, ta đều có
Ep

i C1 a c
. 1/ b 2 D1 2x i C 1 :
P1009 x2 i C1 P1008 x2 i C2
Đặt A D i D0 22 i C1
và B D i D0 2 2 i C2
; ta có
1X
009 1X
009
3.1 2x 2 i C 1 / 1
u2i D D2 6A;
i D0 i D0
2 i C1
2 2  41009
1X
008 1X
008
3.1 2x 2 i C 2 / 1
u2 i C1 D D1 6B :
i D0 i D0
22 i C2 41009
1
Ngoài ra, ta cũng có a  A C B  2
nên
1 1
36a 2 24 a C 5 C D 4 .3a 1/ 2 C 1 C
42019 42019
1
 4 .3A C 3B 1/ 2 C 1 C : .2/
42019
14 Lời giải và bình luận đề thi chọn đội tuyển IMO 2018

Ta sẽ chứng minh
 2  2
1 1 1
2 6A C 1 6B  4 .3A C 3B 1/ 2 C 1 C : .3/
2  41009 41009 42019

Bằng biến đổi tương đương, ta viết được bất đẳng thức trên dưới dạng
 
6 1 1 1
1 0 0 9
A C 12B 1 C 1 0 0 9 6A  1 0 0 8 :
4 4 4 4 018
2

1 1
Do A  2
nên 6A > 1 C 4 1009
: Suy ra
 
6 1 6
1009
A C 12B 1C 6A  A
4 41009 41009

f
1X
009
6 1


af
41009 i D0
22 i C1
 
6 2 1
D  1
41009 3 41010

st
4
1
D 1008
1
42018
Bất đẳng thức (3) được chứng minh. Bằng cách sử dụng các bất đẳng thức (2) và (3), ta thu được
bất đẳng thức (1). Ta có điều phải chứng minh.
:
n
1 2 1
 xảy ra khi và chỉ khi a D A C B ; B D 0 và
b) Từ các đánh giá ở trên, ta thấy dấu đẳng thức
2
A D 3 1 4 1010 ; hay a D 3 1 4 1010 :
ilo

Bình luận. Biểu thức b 2 nC 1 a c chính là gợi ý quan trọng trong việc xét biểu diễn nhị phân của
a : Thực tế thì không có cách nào xác định được tính chẵn lẻ của đại lượng này nếu không dùng
đến biểu diễn nhị phân của a : Ngoài ra, trong bài toán trên, chúng tôi đã sử dụng đẳng thức
. 1 / a D 1 2 a với a 2 f0; 1 g để lời giải được gọn gàng hơn.
s

Biểu diễn nhị phân (và biểu diễn p-phân nói chung) là một công cụ quan trọng trong toán học,
có nhiều ứng dụng trong tổ hợp, số học, đại số và giải tích. Theo dõi các Shortlist của IMO, ta
Ep

thấy ý tưởng này xuất hiện khá nhiều. Dưới đây chúng tôi xin đưa ra một số bài toán như vậy:
Bài toán 1 (Putnam, 1981). Gọi f .n/ là số chữ số 1 trong biểu diễn nhị phân của số nguyên
dương n : Tính giá trị của tổng:
1
X f .n/
:
nD1
n.n C 1/

Bài toán 2 (IMO Shortlist, 1983). Cho f W Œ0; 1 ! R là hàm số liên tục thỏa mãn đồng
thời các điều kiện:
1
8
< f .x / D bf .2 x /;
ˆ 0x  ;
2
: f .x / D b C .1 b /f .2x 1/; 1
 x  1;
ˆ
2
1Cc
trong đó b D 2Cc
và c > 0: Chứng minh rằng 0 < f .x / x < c với mọi 0 < x < 1:
Lời giải và bình luận đề thi chọn đội tuyển IMO 2018 15

Bài toán 3 (IMO, 1988). Hàm số f xác định trên tập hợp các số nguyên dương được cho
bởi: f . 1 / D 1; f .3/ D 3; f .2 n/ D f .n/; f .4 n C 1/ D 2f .2 n C 1/ f .n/ và
f . 4 n C 3 / D 3f .2 n C 1/ 2f .n/ với mọi số nguyên dương n : Tìm số tất cả các số nguyên
dương n ; nhỏ hơn hay bằng 1988 thỏa mãn điều kiện f .n/ D n :
Bài toán 4 (IMO Shortlist, 1996). Cho dãy số a .n/ .n D 1; 2 ; 3; : : :/ được xác định bởi
a . 1 / D 0 và với mọi n > 1 thì
j n k n.nC 1/
a .n/ D a C . 1/ 2 :
2
a) Tìm giá trị lớn nhất của a .n/ với n là các số nguyên dương  1996 và tìm tất cả các
giá trị n  1996 để giá trị lớn nhất đó đạt được.

b) Tìm số các số n  1996 mà a .n/ D 0:

f
af
st
n
s ilo
Ep
16 Lời giải và bình luận đề thi chọn đội tuyển IMO 2018

Bài 5 (7.0 điểm). Một bảng ô vuông m  n AB C D có các đỉnh là các giao lộ (có tất
cả . m C 1/  .n C 1/ giao lộ). Người ta muốn thiết lập một tuyến đường bắt đầu từ A;
đi theo các cạnh song song với các cạnh của hình chữ nhật và đi qua tất cả các giao lộ
đúng một lần, sau đó quay về A :

a) Chứng minh rằng có thể xây dựng được đường đi khi và chỉ khi m lẻ hoặc n lẻ.

b) Với m ; n thỏa mãn điều kiện câu a), hỏi có ít nhất bao nhiêu giao lộ mà tại đó có ngã rẽ?

Lời giải. Đánh số các hàng và cột theo thứ tự từ trái sang phải 1 ! m C 1; trên xuống dưới
1 ! n C 1: Khi đó, điểm A sẽ ở góc trên bên trái với vị trí .1; 1/:
a) Điều kiện cần: Ta thấy rằng đường đi có thể được biểu diễn bởi dãy các ký tự L ; R ; U
và D ; trong đó các ký tự này lần lượt chỉ hướng đi sang trái, sang phải, lên trên và xuống

f
dưới tại mỗi giao lộ.

af
Do có tổng cộng .m C 1/.n C 1/ giao lộ nên cũng có bấy nhiêu đó số ký tự L ; R ; U ; D:
Vì đường đi xuất phát từ A và quay trở về A nên số lần rẻ trái bằng số lần rẻ phải, số lần đi
xuống bằng số lần đi lên. Điều này chứng tỏ số ký tự L ; R bằng nhau, số ký tự U ; D bằng

st
nhau. Suy ra tổng số ký tự là .m C 1/.n C 1/ phải chẵn. Vì thế nên phải có m lẻ hoặc n lẻ.
n
ilo

Điều kiện đủ: Giả sử m là số lẻ, trường hợp n lẻ hoàn toàn tương tự. Ta sẽ đi theo quy tắc sau
s

(có mô tả như hình trên):


Ep

 Lần đi ngang đầu tiên: xuất phát tại A; đi từ cột 1 ! n.


 Mỗi lần đi dọc thì chỉ xuống một ô; đi ngang thì di chuyển giữa cột 2 $ n :
 Lần đi ngang cuối cùng: đi từ cột n ! 1 sau đó đi ngược từ hàng n ! 1 đến điểm A :
Điều này thực hiện được do m lẻ nên số tuyến đường nằm ngang là chẵn. Vậy điều kiện cần và
đủ để đi được như đề bài là m hoặc n phải lẻ.
b) Xét hai giao lộ có ngã rẽ gần nhau nhất (điểm A tạm tính là một trong các giao lộ đó) thì giữa
hai giao lộ sẽ có một đoạn đường, nằm ngang hoặc nằm dọc. Ta thấy đường đi sẽ là một dãy các
đoạn dọc và ngang luân phiên (mỗi đoạn đi qua hai hoặc nhiều giao lộ), nếu xuất phát theo chiều
ngang thì về theo chiều dọc và ngược lại.
Gọi số đoạn ngang là r , đoạn dọc là c và k là số ngã rẽ (không tính A). Ta sẽ chứng minh
các nhận xét sau:
Nhận xét 1. k C 1 D 2 r D 2 c :
Lời giải và bình luận đề thi chọn đội tuyển IMO 2018 17

Chứng minh. Tại mỗi giao lộ chuyển có ngã rẽ sẽ đúng một đoạn ngang và dọc. Số ngã rẽ,
tính thêm cả A; chính bằng số cặp không thứ tự có dạng fđoạn ngang, đoạn dọcg mà đoạn
ngang và dọc có chung đầu mút tại một giao lộ nào đó. Hơn nữa, mỗi đoạn ngang có chung
đầu mút với đúng 2 đoạn dọc, và mỗi đoạn dọc có chung đầu mút với đúng 2 đoạn ngang nên

k C 1 D 2r D 2c: 

Để tìm giá trị nhỏ nhất của k , ta đưa về tìm giá trị nhỏ nhất của r và c :
Nhận xét 2. r  m C 1 hoặc c  n C 1:
Chứng minh. Giả sử rằng r  m thì số đoạn ngang không xuất hiện đủ trên m C 1 hàng,
do đó có một hàng mà cả n C 1 giao lộ trên đó đều ứng với đoạn dọc, tức là c  n C 1:
Tương tự nếu c  n thì r  m C 1: 
Từ đó, ta đưa về xét các trường hợp sau đây:

f
af
 Nếu m lẻ, n chẵn: Giả sử r  m thì theo nhận xét trên, có một hàng mà cả n C 1 giao
lộ đều ứng với đoạn dọc, nhưng vì có lẻ điểm trên hàng này nên với các đoạn dọc đó, ta
không thể quay về được A, không thỏa. Do đó r  m C 1 và k D 2 r 1  2 m C 1.
Ta dễ dàng xây dựng được đường đi với đúng m C 1 đoạn dọc tương tự câu a). Vì thế
nên min k D 2 m C 1:
st
 Nếu m chẵn, n lẻ: Lập luận tương tự, ta có min k D 2n C 1:

 Nếu m; n đều lẻ: Dễ thấy rằng cả hai đẳng thức trong các đánh giá r  m C 1; c  n C 1
n
đều có thể xảy ra nên min k D 2 min.m; n/ C 1:
Nói tóm lại:
ilo

 Nếu m; n đều lẻ thì min k D 2 min.m; n/ C 1:

 Nếu m chẵn, n lẻ thì min k D 2n C 1:

 Nếu m lẻ, n chẵn thì min k D 2m C 1:


s

Bình luận. Đây là một bài toán khá kinh điển về đường đi Hamilton qua tất cả các đỉnh trên
Ep

grid graph, có thể tham khảo thêm tại liên kết: http://mathworld.wolfram.com/
GridGraph.html.
Kết quả câu a) đã cũ và thậm chí, ta còn có thể đếm được số đường đi như thế theo m ; n nhờ
công thức truy hồi. Ngoài cách giải như trên, ta cũng có thể tiếp cận theo hướng tô màu như sau:
tô màu đen cho các giao lộ tại vị trí .i ; j / mà i C j chẵn; ngược lại thì tô trắng.
18 Lời giải và bình luận đề thi chọn đội tuyển IMO 2018

Khi đó, mỗi lần đi từ giao lộ này sang giao lộ kia, màu sẽ được thay đổi. Để đi qua hết các giao
lộ, mỗi giao lộ một lần rồi quay về đúng A thì rõ ràng, màu phải được đổi chẵn lần. Suy ra
. m C 1 / . n C 1/ chẵn hay ít nhất một trong hai số m ; n lẻ.
Ở câu b), ta cần phải đánh giá cẩn thận hơn về quan hệ giữa số lần đi ngang và đi dọc cũng như
vai trò tính chẵn lẻ của số m ; n :
Một số bài toán tương tự:
Bài toán 1 (Tạp chí Crux). Một con chuột ăn một miếng phô mai hình lập phương kích
thước 3  3  3 gồm 27 miếng nhỏ 1  1  1 .nó chỉ ăn phần ruột bên trong mỗi miếng/:
Hỏi nếu nó xuất phát từ một miếng ở góc thì có thể đi qua được 27 miếng và kết thúc tại
miếng ở giữa không, biết rằng nó chỉ được di chuyển giữa hai miếng có chung mặt với nhau
và không quay về miếng đã đi qua‹

f
af
st
Bài toán 2. Cho bảng ô vuông a  b với a b D 4036 và a ; b > 1: Chứng minh rằng có thể
n
điền các số nguyên dương từ 1 đến 4036 lên bảng sao cho có một đường đi xuất phát từ một ô
nào đó đi qua hết tất cả các ô trên bảng, mỗi ô đi qua đúng một lần rồi quay trở lại ô ban đầu,
biết rằng chỉ được di chuyển giữa hai ô có số chênh lệch nhau đúng 2017 hoặc 2019 đơn vị.
s ilo
Ep
Lời giải và bình luận đề thi chọn đội tuyển IMO 2018 19

Bài 6 (7.0 điểm). Cho tam giác AB C nhọn nội tiếp . O / và . J / là đường tròn bàng tiếp
góc A của tam giác. Gọi D ; E ; F lần lượt là tiếp điểm của . J / với B C ; C A và AB :

a) Gọi L là trung điểm của B C : Đường tròn đường kính LJ cắt các đường thẳng
DE ; DF lần lượt tại K và H : Chứng minh rằng các đường tròn .BDK / và
. C DH / cắt nhau trên đường tròn .J /:

b) Giả sử đường thẳng EF cắt đường thẳng B C tại G và GJ cắt các đường thẳng
A B ; AC lần lượt tại M ; N : Gọi P và Q là các điểm trên các đường thẳng JB ; J C
sao cho ∠ PAB D ∠ QAC D 9 0 ı : Gọi T là giao điểm của hai đường thẳng
P M ; QN và S là điểm chính giữa cung lớn B C của đường tròn . O /: Gọi I là tâm
đường tròn nội tiếp tam giác AB C : Chứng minh rằng đường thẳng S I cắt đường
thẳng AT tại một điểm thuộc đường tròn . O /:

f
Lời giải. a) Gọi D 0 và F 0 lần lượt là tiếp điểm của đường tròn .I / nội tiếp tam giác AB C

af
với B C ; BA : Dựng đường kính D W của đường tròn .J /: Ta có
AI IF 0 ID0
D D
AJ JF JW
nên ba điểm A; D 0 và W thẳng hàng.
A
st
n
D1

F0
ilo

I O

L D
B C
D0
E
s
Ep

X J U

Y
U0

W
20 Lời giải và bình luận đề thi chọn đội tuyển IMO 2018

Gọi D1 là giao điểm của các đường thẳng ID 0 và AD: Ta có JD D J W nên ID 0 D ID1 : Mà
BD 0 D CD nên LD D LD 0 ; từ đó suy ra IL k DD1 :
Gọi X là trung điểm của D 0 W thì tứ giác DLXJ là hình chữ nhật. Suy ra ∠XHD D 90ı ; hay
XH k JB: Từ đó XH ? BI:
Dựng hình chữ nhật C D J U: Ta có J U k C D và J U D C D nên J U k BD 0 và
J U D BD 0 ; suy ra tứ giác BD 0 U J là hình bình hành. Từ đây, ta thu được D 0 U k XH :
Gọi Y ; V và Z lần lượt là giao điểm của đường thẳng XH với các đường thẳng D J ; U W ;
U C : Khi đó, ta có V là trung điểm của U W ; mà U Z k Y W nên V là trung điểm của Y Z :
Xét tứ giác C E U J nội tiếp có C E D C D D U J nên U E k C J hay U E ? DE : Mà
E W ? ED nên ba điểm W ; U và E thẳng hàng.
Xét các tam giác B I C và Y V W ; ta có

f
∠ Y W V D ∠ C ED D ∠ I C B

af

∠ W Y V D ∠X Y J D ∠ B JD D ∠ I B C
nên 4B I C  4Y V W (g-g). Mà V là trung điểm của Y Z và L là trung điểm của B C nên ta

st
có 4B I L  4Y Z W (c-g-c). Do đó ∠ Y W Z D ∠ B LI D ∠ BDA D 90 ı ∠ W D U 0 :
Từ đó suy ra ∠ D U 0 W D 90 ı với U 0 là giao điểm của các đường thẳng W Z và AD: Mặt
khác, ta lại có ∠ D U 0 W D ∠D U 0 Z D 90 ı nên U 0 thuộc các đường tròn .C DH / và .J /:
Chứng minh tương tự, đường tròn .BDK / cũng đi qua U 0 : Từ đó, ta có điều phải chứng minh.
n
b) (Hình vẽ xem trang sau) Gọi S 0 và R lần lượt là giao điểm thứ hai của các đường thẳng
A I ; S I và đường tròn .O /: Ta có S 0 I 2 D S 0 L  S 0 S nên ∠ S 0 I L D ∠ S 0 S I D ∠ SAR :
Mà I L k AD nên ∠ S 0 I L D ∠ S 0 AD ; từ đó suy ra ∠ S 0 AR D ∠ S 0 AD hay
ilo

∠BAR D ∠C AD: .1/


Kẻ DH 0 ? J G .H 0 2 J G /: Ta có GH 0  GJ D GD 2 D GE  GF nên tứ giác EF J H 0
nội tiếp. Lại có tứ giác AE J F nội tiếp nên các điểm A; E ; F ; J và H 0 đồng viên, từ đó suy
ra ∠ A H 0 J D 90 ı hay ba điểm A; D và H 0 thẳng hàng.
s

Mặt khác, ta có ∠ PAJ D ∠ QAJ D 90 ı C 21 ∠ BAC D ∠ B I C và ∠ I B C D ∠ I J C ;


Ep

∠ I C B D ∠ I JB nên 4I B C  4AP J  4AJ Q (g-g).


Gọi P 0 ; Q 0 lần lượt là trung điểm của JP và J Q thì 4 I B L  4AP P 0  4AJ Q 0 và
4I C L  4AQ Q 0 : Suy ra tứ giác AP 0 J Q 0 nội tiếp.
Từ ∠ B I L D ∠JAQ 0 D ∠ JP 0 Q 0 D ∠BP Q ; ta suy ra tứ giác P B I L 0 nội tiếp (L 0 là
giao điểm của các đường thẳng LI và P Q). Mà ∠ P B I D 90 ı nên I L ? P Q: Lại có
I L k A D và AD ? M N nên M N k P Q:
Qua M kẻ đường thẳng song song với đường thẳng AP cắt đường thẳng AT tại A 0 : Ta có

TA0 TM TN
D D ;
TA TP TQ
suy ra NA 0 k AQ và ∠ AM A 0 D ∠ ANA 0 D 90 ı : Từ đó
∠ BAT D 90 ı ∠ AA 0 M D 90 ı ∠ AN M D ∠ C AD: .2/
Từ (1) và (2), ta có ∠BAT D ∠ BAR nên ba điểm A; R và T thẳng hàng.
Lời giải và bình luận đề thi chọn đội tuyển IMO 2018 21

L0

A S

Q0

f
O

af
I
P0

L D C G
B

F
R S0

H0
E
st
N

J
n
M

A0
s ilo
Ep

Bình luận. Đây là bài toán khá khó. Hai ý của bài toán không liên quan mấy tới nhau. Ý a) có
thể tổng quát hơn (và đơn giản hơn) thành bài toán sau:
Bài toán 1. Cho tam giác AB C và điểm P bất kỳ trên mặt phẳng. Gọi DEF là tam giác
pedal của điểm P : Đường tròn .K / đi qua P và D cắt lại các đường thẳng DE ; DF lần
lượt tại M và N : Chứng minh rằng các đường tròn .DB M /; .D C N / và .DEF / có một
điểm chung khác D:
22 Lời giải và bình luận đề thi chọn đội tuyển IMO 2018

Tính chất này còn có thể mở rộng hơn nữa bằng cách sử dụng điểm Miquel:
Bài toán 2. Cho tam giác AB C có các điểm D ; E ; F bất kỳ nằm trên cạnh B C ; C A; A B :
Các đường tròn .AEF /; .C F D /; .C DE / có một điểm chung là M : Một đường tròn .K /
đi qua P và D cắt lại các đường thẳng DE ; DF lần lượt tại Q và R : Chứng minh rằng các
đường tròn .DBQ /; .D C R / và .DEF / có một điểm chung khác D:
Lời giải hai bài toán tổng quát trên ngoài phương pháp cộng góc thông thường thì ta cũng có
thể sử dụng phép nghịch đảo cực D:
Ý b) của bài TST cũng là bài toán khá thách thức đòi hỏi học sinh phải có nhiều kỹ năng và biết
một số bổ đề mới có thể giải trọn vẹn. Ý này có thể tổng quát cho tam giác pedal bất kỳ như sau:
Bài toán 3. Cho tam giác AB C và điểm P bất kỳ trên mặt phẳng. Gọi DEF là tam giác
pedal của điểm P : Đường thẳng EF cắt đường thẳng B C tại G: Đường thẳng GP cắt các
đường thẳng C A và AB lần lượt tại Q ; R : Các đường thẳng qua A vuông góc AB và AC cắt

f
các đường thẳng P B ; P C tại M ; N : Đường thẳng N Q cắt đường thẳng M R tại L : Đường

af
tròn . DEF / cắt lại đường thẳng B C tại K : Chứng minh rằng ∠ K AB D ∠LAC :
Khi P trùng với tâm đường tròn bàng tiếp góc A của tam giác AB C thì ta thu được ý b) của
bài TST. Đây là một mở rộng theo kiểu đẳng giác có ý nghĩa và khá phức tạp. Sơ lược các bước
chứng minh cho bài toán tổng quát này như sau:
st
 Chứng minh M N song song với GP bằng cách sử dụng tỉ số kép, tức là chứng minh

P .B C ; D G / D P .M N ; D 1 /:
n
 Chứng minh hai tam giác K QR và AM N thấu xạ tâm T bằng định lý Desargues, cùng
với các định lý dùng để biến đổi tỉ số là Thales, Menelaus.
ilo

 Cuối cùng là chỉ ra hai đường thẳng AL và AT đẳng giác với góc ∠ M AN (cũng là đẳng
giác trong góc ∠ BAC ) thông qua bổ đề: Cho hình thang AB C D có hai đáy là AB và
C D: Đường thẳng AD cắt đường thẳng B C tại E : Các đường chéo AC và BD cắt
nhau tại F : Nếu một điểm M thỏa mãn M A; M B đẳng giác với góc ∠C M D thì M A;
s

M B cũng đẳng giác với góc ∠ E M F :


Ep

Có thể tham khảo thêm các mở rộng tại liên kết: http://analgeomatica.blogspot.
com/2018/04/cac-bai-toan-hinh-hoc-hang-tuan-tuan-1.html.

You might also like